ch. 18 PAP Test

Posted: April 8, 2013 in Gynocology

The Papanicolaou test (also called Pap smear, Pap test, cervical smear, or smear test) is a screening test used to detect potentially pre-cancerous and cancerous processes in the endocervical canal (transformation zone) of the female reproductive system.

Pap test

Pap test

Pap test

Pap test

cervicalgraphic

Types of PAP Test :

  1. Conventional Pap—In a conventional Pap smear, samples are smeared directly onto a microscope slide after collection.
  2. Liquid based cytology—The Pap smear sample is put in a bottle of preservative for transport to the laboratory, where it is then smeared on the slide.

The patient may also be referred for HPV DNA testing, which can serve as an adjunct to Pap testing.

Summary of pap test indications
woman’s characteristic indication rationale
never had sexual contact no test HPV usually transmitted by sexual contact
under age 21, regardless of sexual history no test more harms than benefits
age 20–25 until age 50–60 test every 3–5 years if results normal broad recommendation
over age 65; history of normal tests no further testing recommendation of USPSTFACOG,ACS and ASCP;
had total hysterectomy for non-cancer disease – cervixremoved no further testing harms of screening after hysterectomy outweigh the benefits
had partial hysterectomy – cervix remains continue testing as normal  
has received HPV vaccine continue testing as normal vaccine does not cover all cancer-causing types of HPV
 
post-operative transgender woman no test the neo-vagina does not contain a cervix and cannot be evaluated with a pap smear

 

ch. 17 Leopold’s maneuvers

Posted: April 2, 2013 in Gynocology

Maternal abdominal examination of the fetus by Leopold’s maneuvers is a common and systematic way to determine the position of a fetus inside the woman’s uterus.

• The lie is the relationship between the longitudinal axis of fetus and mother: longitudinal, oblique, and transverse.

19162 


Presentation is the relationship between the leading fetal part and the pelvic inlet: cephalic, or breech. The presenting part may be the fetal heard or the breech. Ordinarily, the head is flexed sharply so that the chin is in contact with the thorax. In this circumstance, the occipital fortanel is the presenting part, and such a presentation is usually referred to as a vertex or occiput presentation.

271_272_1
Position is the relationship between definite part of the fetus (back) to the right or left side of maternal pelvis. With each presentation there may be two position: right (II-second) or left (I-first). About two thirds of all vertex presentations are in the left occiput position and the one third in the right.

Fetus Back Presentations Examples. Step 1 (Right) Step 2 (Left)

Fetus Back Presentations Examples. Step 1 (Right) Step 2 (Left)


Variety is the relation of the back of fetus to the anterior and or posterior side of the mother’s pelvis. There may be two variety anterior or posterior. 
Engagement – the fetal is engaged if the widest leading part (typically the widest circumference of the head) is negotiating the inlet. 
With the mother lying comfortably on her back, the examiner faces the patient for the first 3 steps, and faces towards her feet for the fourth.

leopold_maneuver_l

1. FIRST maneuver. Having established the height of the fundus, the fundus itself is gently palpated with the fingers of both hands, in order to discover which pole of the fetus (breech or head) is present. The head feels hard and round, and is easily movable and ballotable. The breech feels soft, triangular and continuous with the body. A professional can also determine the level of the uterine fundal height (but not in cm, on the levels; on the level of umbilicus, ribs, xiphoid process).

2. SECOND maneuver. The palms hands are now placed on the sides of the abdomen. On one side there is the smooth, firm curve of the back of the fetus, and on the other side are fetal extremities (arms, legs) that are felt like small irregularities and protrusions. It is often difficult to feel the fetus well when the patient is obese, when there is a lot of liquor or when the uterus is tight, as in some primigravidas.

3. THIRD maneuver. The examiner grasps the lower portion of the abdomen, just above the symphysis pubis, between the thumb and fingers of one hand. The objective is to feel for the presenting part of the fetus and to decide whether the presenting part is loose above the pelvis or fixed in the pelvis. If the head is loose above the pelvis, it can be easily moved and balloted. The head and breech are differentiated in the same way as in the first step.

4. FOURTH maneuver. The objective of the step is to determine the amount of head palpable above the pelvic brim, if there is a cephalic presentation. The examiner faces the patient’s feet, and with the tips of the middle 3 fingers palpates deeply in the pelvic inlet. In this way the head can usually be readily palpated, unless it is already deeply in the pelvis. The amount of the head palpable above the pelvic brim can also be determined.

Krok 2. Medicine – 2011

Posted: March 28, 2013 in Krok II
Tags: , , ,

Krok 2. Medicine-2011

 1 A children’s health camp received a party of tinned food. External examination of the tins revealed that they had deep dents, could  be easily concaved when pressed and wouldn’t immediately return to the initial state; rust was absent; the tins were greased with  inedible fat. Specify the bloat type:  

A. Physical*

B. Physicochemical

C. Combined  

D. Biological

E. Chemical

 

 2 During a surgery on a 30-year-old patient a dark ileo-ileal conglomerate was discovered, the intussusceptum intestine was  considered to be unviable. The intussuscipiens intestine was dilated to 7-8 cm, swollen, full of intestinal contents and gases. What  pathology led to the surgery?  

A. Strangulation obstruction  

B. Obturation obstruction  

C. Paralytic obstruction  

D. Spastic obstruction  

E. Invagination (combined) obstruction*  

 

 3 A primigravida is 22 years ol d. She has Rh(-), her husband has Rh(+). Antibodies to Rh weren’t found at 32 weeks of pregnancy.  Redetermination of antibodies to Rh didn’t reveal them at 35 weeks of pregnancy as well. How often should the antibodies be  determined hereafter?  

A. Montly  

B. Once in three weeks  

C. Once in two weeks  

D. There is no need in further checks  

E. Once a week*  

 

 4 A 7-year-old boy has been managed for a month. Immediately after hospitalization there were apparent edemata, proteinuria – 7,1  g/l, daily urine protein – 4,2 g. Biochemical blood test shows persistent hypoproteinemia (43,2 g/l), hypercholesterolemia (9,2  millimole/l). The patient is most likely have the following type of glomerulonephritis:  

A. Hematuric  

B. Nephritic  

C. Nephrotic*  

D. Combined  

E. Isolated urinary

preview008  

 

 5 On the 2nd day after a surgery for toxic mixed goiter IV a 35-year-old patient complains of heart pain. ECG shows prolonged QT  intervals. Chvostek’s and Trousseau symptoms cannot be clearly defined. The patient is provisionally diagnosed with latent  tetany. What study will allow to confirm the diagnosis?

A. Determination of thyroid hormones  

B. Determination of sodium  

C. Determination of thyrotropic hormone  

D. Determination of potassim  

E. Determination of blood calcium and phosphor*  

Trousseau Sign

Trousseau Sign

chvostek's sign

chvostek’s sign

 

 6 In autumn a 25-year-old patient developed stomach ache arising 1,5-2 hours after having meals and at night. He complains of  pyrosis and constipation. The pain is getting worse after consuming spicy, salty and sour food, it can be relieved by means of  soda and hot-water bag. The patient has been suffering from this disease for a year. Objectively: furred moist tongue. Abdomen  palpation reveals epigastrial pain on the right, resistance of abdominal muscles in the same region. What is the most likely  diagnosis?

A. Duodenal ulcer*  

B. Diaphragmatic hernia  

C. Stomach ulcer  

D. Chronic cholecystitis  

E. Chronic pancreatitis

    • NB!   Difference Between a Duodenal Ulcer and Gastric Ulcer
  • Both types of ulcer cause pain, particularly gastric pain, climbing up to the esophagus. However, with a gastric ulcer, pain cannot be relieved by eating foods. In a duodenal ulcer, it can be relieved by eating. In a duodenal ulcer, there is bleeding in the stool called melena. In gastric ulcers, there is blood when the patient vomits called hematemesis. In gastric ulcers, pain occurs 1-2 hours after eating. In duodenal ulcers, pain occurs 3-4 hours after eating. 7 A 42-year-old female patient suffers from micronodular cryptogenic cirrhosis. Over the last week her condition has deteriorated:  there appeared convulsions, mental confusion, increased jaundice. What study may explain such aggravation?  

A. Determination of cholesterol ethers  

B. Determination of serum ammonia*

C. Determination of alkaline phosphatase rate  

D. Determination of ALAT and ASAT  

E. Determination of alpha-phetoprotein rate  

NB! Cryptogenic cirrhosis (Idiopathic Cirrhosis) is a type of liver disease for which there is no easily identifiable cause.It’s possible that another type of liver disease known as nonalcoholic steatohepatitis (NASH), which is a buildup of fat in the liver, may lead to cryptogenic cirrhosis.

Most ammonia in the body forms when protein is broken down by bacteria in the intestines. Ammonia levels in the blood rise when the liver is not able to convert ammonia to urea. This may be caused by cirrhosis or severe hepatitis.

 

 8 A parturient woman is 23 years old. Vaginal obstetric examination reveals full cervical dilatation. There is no fetal bladder. Fetal  head is in the plane of pelvic outlet. Sagittal suture is in mesatipellic pelvis, anterior fontanel is closer to pubes. The fetal head  diameter in such presentation will be:  

A. Suboccipito-bregmaticus*  

B. Suboccipitio-frontalis  

C. Biparietal  

D. Fronto-occipitalis recta  

E. Mento-occipitalis  

suboccipitobregmatic1336654161747

 

 9 A 49-year-old patient complains of dyspnea, cough. There are no sputum discharges. He has repeatedly used salbutamol and intal  but with no effect. Objectively: he is only able to sit while leaning on the table. Cyanosis of face, acrocyanosis are present.  Breathing is shallow, laboured, in some parts it cannot be auscultated; there are diffuse rales, expiration is significantly prolonged.  Heart sounds are muffled, tachycardia is present. Ps – 112/min., AP – 110/70 mm Hg. Liver is located near the costal arch. There  are no peripheral edemat a. What is your provisional diagnosis?  

A. Status asthmaticus*  

B. Cardiac asthma  

C. Chronic obstructive bronchitis  

D. Bronchiale asthma, moderate gravity  

E. Foreign object aspiration  

NB!  Acute severe asthma = status asthmaticus = Severe Asthma attacks. Cardiac asthma is a medical diagnosis of wheezing, coughing or shortness of breath due to congestive heart failure. It is known as cardiac asthma because the symptoms mimic ordinary asthma.

 

 10 A patient who had eaten mushrooms in the morning was delivered to the infectious diseases hospital at night. The disease  development was rapi d. The patient presented with stomach pain, frequent diarrhea, intractable vomiting, burning thirst, headache  and dizziness. He died on the third day. What mushrooms are most likely to have caused mycetismus?

A. Russules  

B. Morels  

C. Deadly amanita*  

D. Fly agarics  

E. Sulfur-tufts  

Deadly amanita

Deadly amanita

 

 11 The institutions which take part in carrying out medical examinations can be prevention and treatment facilities, medical board of  Ministry of Defense, medical board of Ministry of Home Affairs, medico-social expert commissions, forensic medical boards etc.  What institutions are responsible for temporary disability examination?

A. Medical boards of Ministry of Defense  

B. Prevention and treatment facilities*  

C. Medical boards of Ministry of Home Affairs  

D. Medico-social expert commissions  

E. Sanitary-and-prophylactic institutions  

 

 12 A 26-year-old patient with left lower lobe pneumonia experiences an acute chest pain on the left during coughing. Objectively:  diffuse cyanosis, widening of the left half of chest. Percussion reveals high tympanitis. Auscultation reveals no respiratory  murmurs above the left half of chest. There is a deviation of the right cardiac border towards the midclavicular lin e. What  examination will be the most informative?  

A. X-Ray*  

B. Spirography  

C. Bronchography  

D. Pneumotachometry  

E. Bronchoscopy  

 

 13 A 54-year-old female patient consulted a gynaecologist about bloody discharges from the vagina for 1 month. Last menstruation  was 5 years ago. Gynaecological examination revealed no pathological changes. What is the tactics of choice?  

A. Cytosmear  

B. Colposcopy  

C. Symptomatic therapy  

D. Diagnostic fractional curettage of uterine cavity*  

E. USI  

7071

 

 14 A child was born at a gestational age of 34 weeks in grave condition. The leading symptoms were respiratory distress symptoms,  namely sonorous and prolonged expiration, involving additional muscles into respiratory process. The Silverman score at birth  was 0 points, in 3 hours it was 3 points with clinical findings. Which diagnostic study will allow to diagnose the form of  pneumopathy?  

A. Immunoassay  

B. Proteinogram  

C. X-ray of chest *

D. Determination of blood gas composition  

E. Clinical blood test

Silverman Score (Silverman-Anderson Index)

Silverman Score (Silverman-Anderson Index)

Silverman Scoring for Respiratory Distress

Silverman Scoring for Respiratory Distress

 

 

 15 It is required to analyze the level of daylight illumination in a ward of therapeutics unit. What device should be applied to estimate  the level of daylight illumination?

A. Anemometer  

B. Illuminometer*

C. Actinometer  

D. Psychrometer  

E. Katathermometer

Digital Illuminometer

Digital Illuminometer

 

 

 16 A 38-year-old male patient has been taking alcohol for 3 years. 3 days after a regular drinking period he felt anxiety and fear. It  appeared to him that he was surrounded by spiders and worms, pursued by some “condemnatory voices”. His behaviour became  aggressiv e. The patient demonstrated correct self-awareness but impairment of temporal and spatial orientation. What is the most  likely diagnosis?  

A. Delirium alcoholicum*  

B. Pathologic intoxication  

C. Alcoholic encephalopathy  

D. Alcoholic hallucinosis  

E. Alcoholic paranoia  

 

 17 A pregnant woman was delivered to the gynecological unit with complaints of pain in the lower abdomen and insignificant bloody  discharges from the genital tracts for 3 hours. Last menstruation was 3 months ago. Vaginal examination showed that body of  womb was in the 10th week of gestation, a fingertip could be inserted into the external orifice of uterus, bloody discharges were  insignificant. USI showed small vesicles in the uterine cavity. What is the most likely diagnosis?

A. Threat of spontaneous abortion  

B. Grape mole*  

C. Abortion in progress  

D. Incipient abortion  

E. Incomplete abortion

molar-pregnancy-1 

 

 18 A 58-year-old patient complains about sensation of numbness, sudden paleness of II-IV fingers, muscle rigidness, intermittent  pulse. The patient presents also with polyarthralgia, dysphagia, constipations. The patient’s face is masklike, solid edema of hands  is present. The heart is enlarged; auscultation revealed dry rales in lungs. In blood: ESR – 20 mm/h, crude protein – 85/l,  g-globulines – 25%. What is the most likely diagnosis?  

A. Systemic lupus erythematosus  

B. Dermatomyositis  

C. Raynaud’s disease  

D. Systemic scleroderma*  

E. Rheumatoid arthritis  

Systemic Sclerosis (scleroderma)

Systemic Sclerosis (scleroderma)

systemic scleroderma - Musk Face

systemic scleroderma – Musk Face

 

 19 A 40-year-old patient, the forester, complains of severe headache, body temperature rise up to 39,5oC, trembling limbs. From  the patient’s history we know that he had seriously cut his hand during the dissection of a killed fox. Objectively: depressed mood.  The patient asks not to turn on the light or open the door. Any noise causes apparent motor excitation. When he saw a carafe of  water, he developed convulsive throat spasms. What tactics should an emergency doctor choose?  

A. Deliver the patient to the infectious disease hospital*

B. Deliver the patient to the neurological department  

C. Let him stay at home and consult a psychiatrist  

D. Deliver the patient to the psychiatric hospital  

E. Deliver the patient to the resuscitation department

 

 

 20 While asessing the health status of graduates of a secondary school, the doctor found one of them to have grade 3 tonsillar  hypertrophy, chronic rhinitis and vegetative-vascular dystonia. The organism functionality is reduced. This student belongs to the  following health group:  

A. II  

B. V

C. I

D. IV

E. III*

 

 21 A 32-year-old patient consulted a doctor about being inable to get pregnant for 5-6 years. 5 ago the primipregnancy ended in  artificial abortion. After the vaginal examination and USI the patient was diagnosed with endometrioid cyst of the right ovary. What  is the optimal treatment method?

A. Hormonal therapy with androgenic hormones  

B. Anti-inflammatory therapy  

C. Conservative therapy with estrogen-gestagenic drugs  

D. Sanatorium-and-spa treatment  

E. Surgical laparoscopy*  

 Surgical laparoscopy

Surgical laparoscopy

 

 22 A patient is being prepared for the operation on account of varix dilatation of lower extremities veins. Examination of the patient’s  soles revealed flour-like desquamation along the skin folds. All the toenails are greyish-yellow, thickened and partially decayed.  What dermatosis should be suspected?

A. Microsporia  

B. Candidosis  

C. Pityriasis versicolor  

D. Microbial eczema  

E. Rubromycosis*

Rubromycosis

Rubromycosis

 23 A 3-year-old child has been taken to a pediatrician. He has no recent history of any diseases. Objective examination revealed no  pathology of the internal organs. The child needs the routine immunization against the following disease:

A. Type B hepatitis

B. Measles, rubella, parotitis  

C. Diphtheria and tetanus  

D. Poliomyelitis*  

E. Pertussis  

Picture-10

 

 24 While staying in a stuffy room a 19-year-old emotionally labile girl developed severe weakness, dizziness, blackout, nausea and  loss of consciousness without convulsions. Objectively: the patient is unconscious, the skin is pale, extremities are col d. AP –  90/60 mm Hg, Ps- 96/min, deficient, breathing is shallow. Pupillary and tendon reflexes are present. There are no pathological  signs. What is the most likely diagnosis?  

A. Syncope *

B. Epileptic attack

C. Transient ischemic attack  

D. Hysterical neurosis  

E. Vegetovascular paroxysm

Syncope

Syncope

 

 25 A factory worker has ARD complicated by acute bronchitis. He receives treatment in the outpatient setting. The attending doctor  has issued him a medical certificate for 5 days and then extended its duration by 5 more days. Patient can’t get down to work  because of his health status. Who should extend the duration of medical certificate for this patient?  

A. Deputy medical superintendent in charge of temporary disability examination

B. Medical advisory commission  

C. Medical superintendent  

D. A department chief *

E. Deputy medical superintendent in charge of medical treatment  

 

 26 Head circumference of a 1-month-old boy with signs of excitement is 37 cm, prefontanel is 2×2 cm large. After feeding the child  regurgitates small portions of milk; stool is normal in respect of its volume and composition. Muscle tonus is within norm. What is  the most likely diagnosis?  

A. Meningitis  

B. Pylorospasm *

C. Pylorostenosis  

D. Craniostenosis  

E. Microcephaly  

Congenital pyloric stenosis. The abnormal narrowing of the opening of the pylorus causes episodes of projectile vomiting.

Congenital pyloric stenosis. The abnormal narrowing of the opening of the pylorus causes episodes of projectile vomiting.

 

 27 A 32-year-old male patient has been suffering from pain in the sacrum and coxofemoral joints, painfulness and stiffness in the  lumbar spine for a year. ESR – 56 mm/h. Roentgenography revealed symptoms of bilateral sacroileitis. The patient is the carrier of  HLA B27 antigen. What is the most likely diagnosis?  

A. Ankylosing spondylitis*  

B. Rheumatoid arthritis  

C. Reiter’s disease

D. Coxarthrosis  

E. Spondylosis  

as1

NB! Human Leukocyte Antigen (HLA) B27 (subtypes B*2701-2759) is a class I surface antigen encoded by the B locus in the major histocompatibility complex (MHC) on chromosome 6 and presents antigenic peptides (derived from self and non-self antigens) to T cells. HLA-B27 is strongly associated with ankylosing spondylitis (AS), and other associated inflammatory diseases referred to “spondyloarthropathies”. 

 28 A 14-year-old girl has been presenting with irritability and tearfulness for about a year. A year ago she was also found to have  diffuse enlargement of the thyroid gland (II grade). This condition was regarded as a pubertal manifestation, the girl didn’t undergo  any treatment. The girl’s irritability gradually gave place to a complete apathy. The girl got puffy face, soft tissues pastosity,  bradycardia, constipations. Skin pallor and gland density progressed, the skin became of a  waxen hue. What disease may be  suspected?  

A. Autoimmune thyroiditis*  

B. Thyroid carcinoma  

C. Subacute thyroiditis  

D. Diffuse toxic goiter  

E. Juvenile basophilism

Autoimmune thyroiditis

Autoimmune thyroiditis

 

 

 29 A 38-year-old patient complains of inertness, subfebrile temperature, enlargement of lymph nodes, nasal haemorrhages, bone  pain. Objectively: the patient’s skin and mucous membranes are pale, palpation revealed enlarged painless lymph nodes;  sternalgia; liver was enlarged by 2 cm, spleen – by 5 cm, painless. In blood: erythrocytes – 2,7*1012/l, Hb- 84 g/l,  leukocytes – 58*109/l, eosinophils – 1%, stab neutrophils – 2%, segmented neutrophils – 12%, lymphocytes – 83%,  lymphoblasts – 2%, smudge cells; ESR- 57 mm/h. What is the most likely diagnosis?  

A. Lymphogranulomatosis

B. Chronic myeloleukemia

C. Acute myeloleukemia

D. Acute lymphatic leukemia

E. Chronic lymphatic leukemia*

NB! Chronic lymphocytic leukemia is cancer of a type of white blood cells called lymphocytes. Chronic lymphocytic leukemia (also called CLL) is a blood and bone marrow disease that usually gets worse slowly. CLL is the second most common type of leukemia in adults. It often occurs during or after middle age.

Symptoms_of_Leukemia-pd

 

 30 On the 5th day after a surgery for colon injury a patient complains of bursting pain in the postoperative wound, weakness,  drowsiness, headache, fever up to 40oC. Objectively: the skin around the wound is swollen, there is gas crepitation. The  wound discharges are scarce foul-smelling, of dark-gray colorl. What is the most likely diagnosis?

A. Postoperative wound infection  

B. Abscess  

C. Anaerobic clostridial wound infection*

D. Erysipelas

E. Phlegmon

Alternate Names : Tissue infection - Clostridial, Gangrene - gas, Myonecrosis, Clostridial infection of tissues

Alternate Names : Tissue infection – Clostridial, Gangrene – gas, Myonecrosis, Clostridial infection of tissues

 

 31 A patient is 50 years old, works as a builder with 20 years of service recor d. He was admitted to the hospital for chest pain, dry  cough, minor dyspnea. Objectively: sallow skin, acrocyanosis, asbestos warts on the hands. In lungs – rough respiration, diffuse  dry rales. The x-ray picture shows intensification of pulmonary pattern, signs of pulmonary emphysema. What is the most likely  diagnosis? 

A. Lung cancer  

B. Chronic obstructive bronchitis  

C. Tuberculosis  

D. Asbestosis*  

E. Pneumonia  

chest_xray_asbestosis

 

 32 A 68-year-old patient consulted a doctor about a tumour in her left breast. Objectively: in the upper internal quadrant of the left  breast there is a neoplasm up to 2,5 cm in diameter, dense, uneven, painless on palpation. Regional lymph nodes are not enlarged.  What is the most likely diagnosis? 

A. Cyst  

B. Mastopathy  

C. Lipoma  

D. Fibroadenoma  

E. Cancer*  

SignOfBreastCancer

 

 33 A 50-year-old male suburbanite underwent treatment in rural outpatient clinic for pneumonia. The treatment didn’t have effect and  the disease got complicated by exudative pleuritis. What prevention and treatment facility should the patient be referred to for  further aid?

A. Phthisio-pulmonological dispensary  

B. Tuberculosis dispensary

C. Central district hospital*  

D. Municipal hospital  

E. Regional hospital  

 

 34 A woman consulted a doctor on the 14th day after labour about sudden pain, hyperemy and induration of the left mammary gland,  body temperature rise up to 39oC, headache, indisposition. Objectively: fissure of nipple, enlargement of the left mammary  gland, pain on palpation. What pathology would you think about in this case?

A. Lactational mastitis*

B. Fibrous adenoma of the left mammary gland

C. Phlegmon of mammary gland  

D. Lacteal cyst with suppuration

E. Breast cancer

mastitis-2

 

 35 A 45-year-old man has been exhibiting high activity for the last 2 weeks, he is talkative, euphoric, has little sleep, claims being able  “to save the humanity and solve the problem of cancer and AIDS”, gives money to starangers. What is the most likely diagnosis?  

A. Agitated depression

B. Schizo-affective disorder

C. Panic disorder  

D. Catatonic excitation

E. Maniacal onset*  (insane)

Maniacal onset (insane)

Maniacal onset (insane)

 

 36 A 43-year-old female patient complains of unstable defecation with frequent constipations, abdominal swelling, headache, sleep  disturbance. Body weight is unchanged. What disease are these clinical presentations typical for?  

A. Chronic pancreatitis  

B. Chronic atrophic gastritis  

C. Colorectal cancer  

D. Irritable colon syndrome*

E. Chronic enteritis  

Symptoms and signs of Irritable Bowel Syndrome.

Symptoms and signs of Irritable Bowel Syndrome.

ibs

 

 37 An 8-year-old boy suffering from haemophilia was undergoing transfusion of packed red cells. Suddenly he felt pain behind the  breastbone and in the lumbar area, dyspnea, cold sweat. Objectively: pale skin, heart rate – 100/min, AP – 60/40 mm Hg; oliguria,  brown urine. For the treatment of this complication the following drug should be administered:  

A. Aminophylline  

B. Analgine  

C. Prednisolone*

D. Lasix  

E. Adrenaline  

PrednisolonePackShot 

 38 A 45-year-old patient, a sailor, was hospitalized on the 2nd day of the disease. A week ago he returned from India. Complains of  body temperature of 41oC, severe headache, dyspnea, cough with frothy rusty sputum. Objectively: the patient is pale,  mucous membranes are cyanotic, breathing rate – 24/min, tachycardia is present. In lungs: diminished breath sounds, moist rales  over both lungs, crepitation. What is the most likely diagnosis?

A. Pneumonic plaque*

B. Sepsis  

C. Influenza  

D. Ornithosis  

E. Miliary tuberculosis

1-s2.0-S0169409X01001661-gr1 

 

 39 A 47-year-old patient complains of insomnia, heaviness over his entire body, constantly depressed mood. He considers himself  good-for-nothing, inadequate. Believes that he is a burden to his family, wants to die. The patient is depressed, inactive, has a  hypomimic face with sorrowful expression. He speaks quietly and monotonely,gives short answers. What is the most likely  diagnosis?  

A. Initial stage of Alzheimer’s disease  

B. Atherosclerotic depression  

C. Major depressive disorder*  

D. Late-onset schizophrenia  

E. Neurotic depression  

 

 40 A 30-year-old patient complains of paroxysmal abdominal pain, frequent liquid stools up to 10 times a day. Throughout the first 3  days he had a fever, since the 2nd day of disease there were scant liquid stools mixed with mucus. On palpation: tenderness of  all colon segments. Sigmoid colon was found spastic. What is your provisional diagnosis?  

A. Balantidiasis  

B. Acute dysentery*  

C. Salmonellosis  

D. Intestinal amebiasis  

E. Cholera

Dysentery  

 

 41 A 32-year-old gravida complains of episodes of unconsciousness, spontaneous syncopes that are quickly over after a change of  body position. A syncope can be accompanied by quickly elapsing bradycardia. There are no other complications of gestation.  What is the most likely reason for such condition?  

A. Postcava compresseion by the gravid uterus*  

B. Pressure fall in the veins of extremities  

C. Psychosomatic disorders

D. Vegetative-vascular dystonia (cardiac type)  

E. Pressure rise in the veins of extremities

Gravid uterus

Gravid uterus

 

 

 42 A 58-year-old female patient came to the antenatal clinic with complaints of bloody light-red discharges from the genital tracts.  Menopause is 12 years. Gynaecological examination found externalia and vagina to have age involution; uterine cervix was  unchanged, there were scant bloody discharges from uterine cervix, uterus was of normal size; uterine appendages were not  palpable; parametria were free. What is the most likely diagnosis?  

A. Granulosa cell tumor of ovary

B. Cervical carcinoma

C. Abnormalities of menstrual cycle with climacteric character  

D. Uterine carcinoma*  

E. Atrophic colpitis  

Uterine cancer (endometrial cancer)

Uterine cancer (endometrial cancer)

 

 43 A 43-year-old female patient was delivered to the hospital in grave condition. She has a history of Addison’s disease. The patient  had been regularly taking prednisolone but a week before she stopped taking this drug. Objectively: sopor, skin and visible  mucous membranes are pigmented, skin and muscle turgor is decreased. Heart sounds are muffled, rapi d. AP- 60/40 mm Hg, heart  rate – 96/min. In blood: Na – 120 millimole/l, K – 5,8 millimole/l. Development of this complication is primarily caused by the deficit of  the following hormone:  

A. Adrostendion

B. Corticotropin (ACTH)  

C. Adrenaline  

D. Noradrenaline

E. Cortisol*

Addison's disease

Addison’s disease

 

 44 3 hours ago a 65-year-old female patient felt sharp abdominal pain irradiating to the right scapula, there was a single vomiting. She  has a history of rheumatoid arthritis. Objectively: pale skin, AP- 100/60 mm Hg, Ps- 60/min. Abdomen is significantly painful and  tense in the epigastrium and right subcostal areat, there are positive symptoms of parietal peritoneum irritation over the right costal  arch, that is tympanitis. What is the tactics of an emergency doctor?  

A. To take the patient to the rheumatological department  

B. To inject pain-killers and watch the dynamics  

C. To perform gastric lavage  

D. To inject spasmolytics  

E. To take the patient to the surgical hospital*  

 

 45 While lifting a heavy load a 39-year-old patient suddenly felt a severe headache, pain in the interscapular region,and started  vomiting. Objectively: the pulse is rhythmic, 60/min., AP – 180/100 mm Hg. The patient is agitated. He presents with photophobia,  hyperacusis. There are positive Kernig’s and Brudzinski’s signs on both sides. In blood: WBCs – 10*109/l. CSF is bloody,  cytosis is 240/3. What is the most likely diagnosis?

A. Sympathoadrenal crisis  

B. Subarachnoid haemorrhage *

C. Ischemic stroke  

D. Acute hypertonic encephalopathy  

E. Meningococcal meningitis

 Subarachnoid haemorrhage

Subarachnoid haemorrhage

Rupture of a berry aneurysm at the bifurcation of large arteries at the base of the brain leads to escape of blood under pressure into the subarachnoid space, effectively coating the brain surface with clotted blood. Cerebral blood flow and brain glucose are reduced. Small ischemic lesions produced by microemboli set off recurring waves of CSDs, which, due to dysfunction of neurovascular coupling, are associated with paradoxical vasoconstriction and tissue hypoxia. These ischemic CSDs, which impose additional metabolic stress on the energy-depleted brain, worsen oxygen and glucose deficits, promoting the expansion of the lesions.

Rupture of a berry aneurysm at the bifurcation of large arteries at the base of the brain leads to escape of blood under pressure into the subarachnoid space, effectively coating the brain surface with clotted blood. Cerebral blood flow and brain glucose are reduced. Small ischemic lesions produced by microemboli set off recurring waves of CSDs, which, due to dysfunction of neurovascular coupling, are associated with paradoxical vasoconstriction and tissue hypoxia. These ischemic CSDs, which impose additional metabolic stress on the energy-depleted brain, worsen oxygen and glucose deficits, promoting the expansion of the lesions.

 

 

 46 Examination of a 13-year-old boy reveals that his body length is 147 сm (+2), body weight – 38 kg (+1,5), circumference of chest –  72 cm (+0,2). Estimate the harmonicity of the child’s physical development:

A. Above the average  

B. Sharply disharmonious  

C. Disharmonious*

D. Harmonious  

E. Supernormal  

 

 47 An ambulance delivered a 21-year-old woman to the gynaecological department with complaints of colicky abdominal pain and  bloody discharges from the genital tracts. Bimanual examination revealed that uterus was soft, enlarged to the size of 6 weeks of  gestation, a gestational sac was palpated in the cervical canal. Uterine appendages weren’t palpable. Fornices are free, deep and  painless. Discharges from the genital tracts are bloody and profuse. What is the most likely diagnosis?  

A. Incipient abortion  

B. Cervical pregnancy  

C. Interrupted fallopian pregnancy  

D. Threat of abortion  

E. Abortion in progress*

6WEEKS

 48 HIV displays the highest tropism towards the following blood cells:  

A. Erythrocytes  

B. Thrombocytes  

C. T-helpers*  

D. T-killers  

E. T-suppressors  

 

 49 A 20 year-old patient complains of nosebleeds, numbness of the lower extremities. Objectively: hyperaemia of face, on the upper  extremities AP is160/90 mm Hg, and 80/50 mm Hg on the lower ones. Pulse on the popliteal and pedal arteries is of poor volume,  there is systolic murmur over the carotid arteries. What is the most likely diagnosis?  

A. Atrial septal defect  

B. Ventricular septal defect  

C. Aorta coarctation*  

D. Aortopulmonary window  

E. Dissecting aortic aneurysm  

Coarctation-of-the-Aorta

 

 50 The Carpathian region is characterized by constant high humidity of atmospheric air (over 80%). Inhabitants of this region feel  severe cold in corresponding season at a medium low temperatur e. It’s caused by heat emission by:

A. Vaporization  

B.

C. Convection*  

D. Radiation  

E. Conduction

Convection

Convection

 

 

 51 A 53-year-old woman complained of cardiac pain and rhythm intermissions. She had experienced these presentations since  childhood. The patient’s father had a history of cardiac arrhythmias. Objectively: the patient was in grave condition, Ps – 220 bpm,  AP – 80/60 mm Hg. ECG: heart rate – 215/min, widening and deformation of QRS complex accompanied by atrioventricular  dissociation; positive P wave. Some time later heart rate reduced down to 45/min, there was a complete dissociation of P  wave and QRST complex. Which of the following will be the most effective treatment?  

A. Cardiac glycosides  

B. Calcium antagonists  

C. Cholinolytics  

D. b-adrenoreceptor blocking agents  

E. Implantation of the artificial pacemaker*  

artificial_pacemaker

19566_20826_5

A 77-year-old man with permanent artificial pacemaker. This is his EKG.

A 77-year-old man with permanent artificial pacemaker. This is his EKG.

 52 The correlation between the service record and eosinophil concentration in blood has been studied in female workers of dyeing  shops of textile factories. What index will be the most informative for the analysis of this data?  

A. Correlation factor *

B. Standardized index

C. Fitting criterion  

D. Sign index

E. Student’s criterion

 

 53 The patient complains of a painful swelling in the chin region, malaise, headache. Examination reveals an acutely inflamed  cone-shaped dense node. The skin over it is tense, red. In the center of the node there is an ulcer with overhanging edges and a  necrotic core of a dirty-green colour. Submandibular lymph nodes on the right are enlarged and painful. What is the most likely  diagnosis?

A. Parasitic sycosis  

B. Tertiary syphilis (gummatous form)  

C. Furuncle*  

D. Tuberculosis  

E. Carbuncle  

 Furuncle on the neck

Furuncle on the neck

folliculitis_skin_diagram

 Carbuncle on the neck

Carbuncle on the neck

 

 54 A 58-year-old patient has a 3-year history diabetes mellitus type II. He has been keeping to a diet and regularly taking glyburide. He  has been delivered to a hospital on an emergency basis for acute abdomen. Objectively: the patient is of supernutrition type. The  skin is dry. In the lungs vesicular breathing can be auscultated. Heart sounds are regular, 90/min. AP- 130/70 mm Hg. The symptom  of “wooden belly” is visible. Blood sugar – 9,8 millimole/l. The patients has indication for laparotomy. What is the most appropriate  way of further treatment of diabetes?  

A. To administer Semilong to be taken in the morning and insulin – in the evening  

B. To administer 1 tablet of Glurenorm three times a day  

C. To continue taking glyburide  

D. To administer short insulin*  

E. To administer 1 tablet of Maninil three times a day  

 

 55 A 16-year-old patient who has a history of intense bleedings from minor cuts and sores needs to have the roots of teeth  extracted. Examination reveals an increase in volume of the right knee joint, limitation of its mobility. There are no other changes.  Blood analysis shows an inclination to anaemia (Hb- 120 g/l). Before the dental intervention it is required to prevent the bleeding by  means of:  

A. Dried blood plasma  

B. Fibrinogen  

C. Calcium chloride  

D. Cryoprecipitate* (Frozen Blood)  

E. Epsilon-aminocapronic acid  

Cryoprecipitate-675x332

 

 56 A 30-year-old patient was hospitalized with a diagnosis: intestinal obstruction. During the surgery it was revealed that the  obstruction of the small intestine had been caused by a mass of helminths. What helminths are these?

A. Pinworms  

B. Ascarids*

C. Filarial worms

D. Cysticerci  

E. Guinea worms  

ascarids

 

 57 A 23-year-old patient complains of a dull ache, sensation of heaviness and distention in the epigastrium immediately after meals,  foul-smelling eructation; dry mouth, empty stomach nausea, diarrhea. Objectively: the skin is pale, the patient is of thin build.  Abdomen is soft on palpation, there is epigastric pain. The liver does not extend beyond the costal arch. In  blood: Hb – 110 g/l,  RBCs – 3,4*1012/l, WBC count is normal. ESR – 16 mm/h. What is the most informative study that will allow make a  diagnosis?

A. Duodenal probing  

B. X-ray of digestion organs  

C. Study of gastric juice  

D. pH-metry  

E. Esophageal gastroduodenoscopy*

ei_2578 

 

 58 A 22-year-old vegetarian patient with signs of malnutrition consulted a doctor about smell and taste distortion, angular stomatitis.  Objectively: marked blue sclerae. The patient was diagnosed with iron deficiency anemia. What is the dominating clinical  syndrome?  

A. Haemolytic  

B. Anaemic  

C. Myelodysplastic

D. Sideropenic*  ( iron deficiency in the blood serum)

E. Haemologic  

 

 59 On the 5th day of a respiratory disease accompanied by fever a 24-year-old man developed progressing headaches, systemic  dizziness, double vision, facial muscles paresis on the right, choking from swallowing. He was diagnosed with acute viral  encephalitis. Identify the main tendency of the emergency treatment:  

A. Ceftriaxone  

B. Zovirax*  

C. Haemodezum

D. Lasix  

E. Glucocorticoids

Zovirax-Aciclovir 

 

 60 A 60-year-old patient has been admitted to a hospital with complaints of dyspnea, tightness in the right subcostal area, abdomen  enlargement. These presentations have been progressing for a year. Heart auscultation reveals presystolic gallop rhythm.  Objectively: swelling of the neck veins, ascites, palpable liver and spleen. What disease requires differential diagnostics?  

A. Hepatocirrhosis  

B. Pulmonary embolism  

C. Chronic pulmonary heart  

D. Lung cancer with invasion to the pleura  

E. Constrictive pericarditis*  

ascites-precox-ivc-pericardial-reflection-constrictive-pericarditis

 

Present when a fibrotic, thickened, and adherent pericardium restricts diastolic filling of the heart.This slowly progresses to a chronic stage consisting of fibrous scarring and thickening of the pericardium with obliteration of the pericardial space

Present when a fibrotic, thickened, and adherent pericardium restricts diastolic filling of the heart.This slowly progresses to a chronic stage consisting of fibrous scarring and thickening of the pericardium with obliteration of the pericardial space

Pericardial Calcifications (TB Patient)  VSConstrictive Pericarditis

Pericardial Calcifications (TB Patient) 

 61 A 43-year-old man who often contacts with ethyl gasoline was admitted to a hospital with complaints of general weakness,  dizziness, memory impairment, sleepiness at daytime and insomnia at night, sensation of a hair in the mouth, colicky pains in the  right subcostal region. What is the most likely diagnosis?  

A. Chronic mercury intoxication  

B. Chronic manganese intoxication  

C. Chronic tetraethyl lead intoxication*  

D. Alcoholic delirium  

E. Chronic lead intoxication  

NB! Tetraethyllead (common name tetraethyl lead), abbreviated TEL. It was mixed with gasoline (petrol) beginning in the 1920s as an inexpensive octane booster which allowed engine compression to be raised substantially, which in turn increased vehicle performance and fuel economy.

TEL was phased out starting in the US in the mid-1970s because of its neurotoxicity and its damaging effect on catalytic converters.

ad-1928-ethyl-gasoline

 

 62 2 weeks after recovering from angina a 29-year-old patient noticed face edemata, weakness, decreased work performance.  There was gradual progress of dyspnea, edemata of the lower extremities, lumbar spine. Objectively: pale skin, weakening of the  heart sounds, anasarca. AP- 160/100 mm Hg. In urine: the relative density – 1021, protein – 5 g/l, erythrocytes – 20-30 in the field of  vision, hyaline cylinders – 4-6 in the field of vision. What is the most likely diagnosis?  

A. Essential hypertension  

B. Acute pyelonephritis  

C. Myxedema  

D. Acute glomerulonephritis*

E. Infectious allergic myocarditis  

 

 63 A 22-year-old patient is a clerk. His working day runs in a conditioned room. In summer he was taken by an acute disease with the  following symptoms: fever, dyspnea, dry cough, pleural pain, myalgia, arthralgia. Objectively: moist rales on the right, pleural  friction rub. X-ray picture showed infiltration of the inferior lobe. In blood: WBC – 11*109/l, stab neutrophils – 6%,  segmented neutrophils – 70%, lymphocytes – 8%, ESR – 42 mm/h. What is the ethiological factor of pneumonia?  

A. Staphylococcus  

B. Legionella*

C. Mycoplasm

D. Streptococcus  

E. Pneumococcus

NB! Legionella is a pathogenic Gram negative bacterium, including species that cause legionellosis (Legionnaires’ disease), most notably L. pneumophila. Legionella is water-borne and lives withinamoebae in the natural environment. 

The most common sources of Legionella and Legionnaires’ disease outbreaks are cooling towers (used in industrial cooling water systems), domestic hot water systems,and spas. Additional sources include large central air conditioning systems, fountains, domestic cold water, swimming pools (especially in Scandinavian countries and northern Ireland) and similar disseminators that draw upon a public water supply. Natural sources include freshwater ponds and creeks.

Legionella pneumophila

Legionella pneumophila

 

 64 10 minutes after delivery a woman discharged placenta with a tissue defect 5х6 cm large. Discharges from the genital tracts  were profuse and bloody. Uterus tonus was low, fundus of uterus was located below the navel. Examination of genital tracts  revealed that the uterine cervix, vaginal walls, perineum were intact. There was uterine bleeding with following blood coagulation.  Your actions to stop the bleeding:

A. To put an ice pack on the lower abdomen  

B. To make manual examination of uterine cavity*  

C. To administer uterotonics  

 

D. To apply hemostatic forceps upon the uterine cervix  

E. To introduce an ether-soaked tampon into the posterior fornix  

 

 65 A 38-year-old woman experiences episodic increases in arterial pressure up to 240/120 mm Hg, which is accompanied by  nausea, vomiting, tachycardia, increased sweating, hyperglycemia. The attack is usually followed by the excessive urination.  Renal sonography reveals an additional formation adjacent to the upper pole of the right kidney and possibly belonging to the  adrenal gland. What laboratory test will allow to clarify the diagnosis?  

A. Blood test for thyroxine and thyrotrophic hormone  

B. Blood test for renin level  

C. Estimation of glomerular filtration rate by measuring endogenous creatinine clearance  

D. Blood test for insulin and C-peptide  

E. Determination of urinary excretion of catecholamines and vanillylmandelic acid*  (Vanillyl mandelic acid (VMA) is an end-stage metabolite of the catecholamines epinephrine and norepinephrine. )

 

 66 A 24-year-old patient complains about putting on weight, limosis. Objectively: the patient’s constitution is of hypersthenic type,  body weight index is 33,2 kg/m2, waist circumference is 100 cm. Correlation of waist circumference to the thigh  circumference is 0,95. What is the most likely diagnosis?  

A. Hypothalamic Itsenko-Cushing obesity of the I stage, abdominal type

B. Alimentary constitutional obesity of the II stage, abdominal type

C. Alimentary constitutional obesity of the I stage, abdominal type*

D. Alimentary constitutional obesity of the III stage, gynoid type  

E. Hypothalamic Itsenko-Cushing obesity of the II stage, gynoid type  

abdominal

 

 67 A 65-year-old patient complains of pain in the lumbar spine, moderate disuria. He has been suffering from these presentations for  about half a year. Prostate volume is 45 cm3 (there are hypoechogenic nodes in both lobes, capsule invasion). The rate of  prostate-specific antigen is 60 ng/l. Prostate biopsy revealed an adenocarcinoma. Which of the supplemental examination methods  will allow to determine the stage of neoplastic process in this patient?  

A. Bone scintigraphy

B. Computer tomography of pelvis*  

 

C. Excretory urography  

D. Roentgenography of chest  

E. Roentgenography of lumbar spine  

Computer tomography of pelvis

Computer tomography of pelvis

 

 

 68 A 49-year-old male patient complains of dyspnea of combined nature, cough, shin edemata, abdomen enlargement due to ascites.  He has a 20-year history of chronic bronchitis. For the last 3 years he has been disabled (group II) because of cardiac changes.  Objectively: mixed cyanosis, edemata. Ps – 92/min, rhythmic, AP – 120/70 mm Hg, respiration rate – 24/min. There is accentuation of  the second sound above the pulmonary artery. Auscultation reveals the box resonance above the lungs. There are also dry rales  over the entire surface of lungs. What is the mechanism of heart changes development in this patient?

A. Kitaev’s reflex  

B. Bainbridge reflex  

C. Respiratory reflex  

D. Euler-Liljestrand reflex*

E. Cardiovascular reflex  

NB! The Euler–Liljestrand mechanism describes the connection between ventilation and blood circulation (perfusion) of the lung. If the ventilation in a part of the lung decreases it leads to local hypoxia. The local hypoxia leads to pulmonary vasoconstriction. This adaptive mechanism is beneficial, because it diminishes the amount of blood that passes the lung without being oxygenated.

 

 69 A child is 7 months old. Birth weight was 3450, the child is breastfed. Supplemental feeding was introduced on time. Determine the  daily protein requirements for the child:  

A. 2,5 g/kg  

B. 4,0 g/kg  

C. 3,0 g/kg*

D. 2,0 g/kg  

E. 3,5 g/kg  

 

 70 On the 21 day after appearance of vesiculous chickenpox rash a 7-year-old child developed ataxia, nystagmus, intention tremor,  muscle hypotonia. Liquor analysis shows a low-grade lymphocytic pleocytosis, slightly increased protein rate. What complication  is it?  

A. Acute nephritis  

B. Purulent meningitis  

C. Pneumonitis

D. Encephalitis*  

E. Postherpetic neuralgia

  1.  Pleocytosis = It is often defined specifically as an increased WBC count in CSF.
  2. Increased white blood cell count in the blood is called Leukocytosis. 
Cerebellar Ataxia

Cerebellar Ataxia

Nystagmus

Nystagmus

tremor

Tremor

Hypotonia

Hypotonia In Infants

 

 71 A 60-year-old female patient was admitted to a hospital for acute transmural infarction. An hour later the patient’s contition got  worse. She developed progressing dyspnea, dry cough. Respiratory rate – 30/min, heart rate – 130/min, AP- 90/60 mm Hg. Heart  sounds were muffled, there was also diastolic shock on the pulmonary artery. The patient presented with medium moist rales in  the lower parts of lungs on the right and on the left. Body temperature – 36,4oC. What drug should be given in the first place?  

A. Heparin  

B. Digoxin  

C. Promedol *

D. Aminophylline  

E. Dopamine  

636008_xlarge

Ischemic 04.004

NB! Promedol, a medicinal preparation that belongs to the group of narcotic analgesics. It exerts a pain-relieving and spasmolytic effect on smooth musculature. Promedol is used in the same instances as preparations of the morphine group.

 

 72 A 7-year-old child was brought to a doctor for a check. The child has a 4-year history of bronchial asthma, asthma attacks occur  mainly in spring and summer. Allergy tests revealed hypersensitivity to poplar seed tufts, field herbs. What recommendation should  be given?  

A. Physiotherapy  

B. Phytotherapy  

C. Treatment at a health resort  

D. Needle reflexotherapy  

E. Specific hyposensitization*  

 

 73 4 hours after having meals a patient with signs of malnutrition and steatorrhea experiences stomach pain, especially above navel  and to the left of it. Diarrheas take turns with constipation lasting up to 3-5 days. Palpation reveals moderate painfulness in the  choledochopancreatic region. The amylase rate in blood is stable. X-ray reveals some calcifications located above navel. What is  the most likely diagnosis?

A. Zollinger-Ellison syndrome  

B. Chronic pancreatitis*  

C. Chronic calculous cholecystitis  

D. Chronic gastroduodenitis  

E. Duodenal ulcer

Chronic pancreatitis

Chronic pancreatitis

Chronic pancreatitis

Chronic pancreatitis

 

 

 74 On the 3rd day of life a baby presented with haemorrhagic rash, bloody vomit, black stool. Examination revealed anaemia,  extended coagulation time, hypoprothrombinemia, normal thrombocyte rate. What is the optimal therapeutic tactics?  

A. Fibrinogen  

B. Vitamin K*  

C. Sodium ethamsylate  

D. Calcium gluconate  

E. Epsilon-aminocapronic acid  

 

 75 A 58-year-old female patient complains about periodical headache, dizziness and ear noise. She has been suffering from diabetes  mellitus for 15 years. Objectively: heart sounds are rhythmic, heart rate is 76/min, there is diastolic shock above aorta, AP is  180/110 mm Hg. In urine: OD- 1,014. Daily loss of protein with urine is 1,5 g. What drug should be chosen for treatment of arterial  hypertension?  

A. a-blocker

B. Calcium channel antagonist  

C. Thiazide diuretic

D. Ihibitor of angiotensin converting enzyme*

E. b-blocker

 

 76 After a serious nervous stress a 35-year-old patient has developed on the dorsal surface of hands redness and swelling that  were later replaced by small inflammatory nodules, vesicles and following erosion with a significant serous discharge. The  process is accompanied by severe itching. What is the most likely diagnosis?  

A. Simple contact dermatitis  

B. Toxicoderma

C. True eczema*  

D. Microbal eczema  

E. Allergic dermatitis

Hand_Eczema_Examples

 

 

 77 A 45-year-old patient complains of some painless nodular elements tending to peripheral growth and fusion. He has a 2-year  history of this disease. Aggravation takes place mainly in spring. In anamnesis: the patient’s father had similar skin lesions.  Objectively: pathological elements looke like guttate and nummular nodules, plaques covered with white scales. What is your  provisional diagnosis?  

A. Lichen ruber planus  

B. Pityriasis rosea  

C. Seborrheic eczema  

D. Psoriasis*  

E. Neurodermitis

Guttate Psoriasis

Guttate Psoriasis

Nummular Psoriasis

Nummular Psoriasis

mangosteen-for-eczema-and-psoriasis_1_1

psoriasis

 

 

 78 A 30-year-old gravida consulted a gynecologist about bright red bloody discharges from the vagina in the 32 week of gestation.  She was hospitalized with a suspicion of placental presentation. Under what conditions is it rational to conduct the internal  examination in order to make a diagnosis?  

A. In the delivery room keeping to all the aseptics regulations  

B. In the admission ward of maternity hospital  

C. In the operating room prepared for the operation*

D. In the examination room of antenatal clinic  

E. The examination is not to be conducted because of risk of profuse haemorrhage

criticalperiodshumandevelopment 

 

 79 A schizophrenic patient considers himself to be “an outstanding scientist, a brilliant composer and an unrivalled artist”. He  complains that “family and friends are always jealous of him and want to poison him“. Determine the psychopathological  syndrome:

A. Manic  

B. Paratrophic  

C. Hebephrenic  

D. Paranoid  

E. Paranoiac*

Paranoia FAIL

Paranoia FAIL

 

 80 A child is 9 months old. The patient’s body temperature is 36,7oC, the skin is pale, humid, there is pain in leg muscles. There is  no extremities mobility, sensitivity is present. The child has been diagnosed with poliomyelitis. The causative agent of this disease  relates to the following family:  

A. Adenovirus  

B. Picornavirus*  

C. Rotavirus  

D. Tohovirus  

E. Paramyxovirus  

pol10

effectspico

 

 81 A student analyzes noise level of cold-pressing process. What device should be applied for this hygienic study?  

A. Sound tester  

B. Pyranometer  

C. Actinometer  

D. Noise analyzer  

E. Noise and vibration analyzer*  

Noise and vibration analyzer

Noise and vibration analyzer

 

 82 A 28-year-old patient complains of periodic compressing heart pain. His brother died at the age of 34 from a cardiac disease with  similar symptoms. Objectively: the patients skin is pale. Heart borders display no significant deviations. Heart sounds are loud,  there is a systolic murmur above all the points with a peak above the aorta. Echocardioscopy reveals thickening of the  interventricular septum in the basal parts, reduction of left ventricular cavity. What drug should be administered in order to prevent  the disease progression?  

A. Metoprolol*

B. Furosemide  

C. Captopril  

D. Nitroglycerin  

E. Digoxin

metoprolol-25-mg 

 

 83 A 24-years-old patient has chronic glomerulonephritis.Urine test reveals the following: the relative density is 1010, protein – 1,65  g/l, RBCs – 5-7 in the field of vision, WBCs – 2-3 in the field of vision. Blood creatinine – 0,350 millimole/l. Serum sodium – 148  millimole/l. What is the main reason for hyperazotemia in this patient?

A. Reduction of renal blood flow  

B. Increased proteinuria  

C. Reduction of tubular reabsorption rate  

D. Sodium retention in the organism  

E. Reduction of glomerular filtration rate*  

img009

 

 84 A 47-year-old patient complains about cough with purulent sputum, pain in the lower part of the left chest, periodical body  temperature rise. She has been suffering from these presentations for about 10 years. Objectively: “drumstick” distal phalanges.  What examination would be the most informative for making a diagnosis?

A. Survey radiograph of lungs  

B. Pleural puncture  

C. Bacteriological analysis of sputum  

D. Bronchography*  

E. Bronchoscopy  

Drumstick Distal PhalangesHands of patient with lung cancer, demonstrating clubbing with increased nail base angle to 180 degrees and fusiform enlargement of the distal digit (so-called drumstick fingers).

Drumstick Distal Phalanges
Hands of patient with lung cancer, demonstrating clubbing with increased nail base angle to 180 degrees and fusiform enlargement of the distal digit (so-called drumstick fingers).

Drumstick

Drumstick

Bronchography COPD Chronic obstructive lung diseases

Bronchography
COPD Chronic obstructive lung diseases

 

 85 A 9-year-old boy fell from a tree and hit the occipital region, there was a momentary loss of consciousness. Objectively: the  child’s condition is satisfactory, he complains of the headache and dizziness. The X-ray of skull reveals a comminuted depressed  fracture of occipital bone in the region of inion. What treatment is indicated for the patient?  

A. Surgical intervention*  

B. Anti-inflammatory therapy  

C. Complex conservative treatment  

D. Therapeutic lumbar punctures  

E. Hemostatic therapy  

 Occipital fracture

Occipital fracture

 

 86 A 60-year-old patient had eaten too much fatty food, which resulted in sudden pain in the right subcostal area, nausea, bilious  vomiting, strong sensation of bitterness in the mouth. Two days later the patient presented with jaundice, dark urine. Objectively:  sclera and skin are icteric, abdomen is swollen, liver is increased by 3 cm, soft, painful on palpation, Ortner’s, Kehr’s, Murphy’s,  Zakharyin’s, Mayo-Robson’s symptoms are positive. Which method should be applied for diagnosis in the first place?

A. X-ray of abdominal organs  

B. Fibrogastroduodenoscopy  

C. Radionuclide scanning of liver and gallbladder  

D. USI of gallbladder and biliary duct*  

E. Diagnostic laparotomy  

  1. Kehr’s sign is the occurrence of acute pain in the tip of the shoulder due to the presence of blood or other irritants in the peritoneal cavity when a person is lying down and the legs are elevated.
  2. Ortner’s syndrome is a rare cardiovocal syndrome and refers to recurrent laryngeal nerve palsy from cardiovascular disease.
  3. Murphy’s sign refers to a maneuver during a physical examination as part of the abdominal examination and a finding elicited in ultrasonography. It is useful for differentiating pain in the right upper quadrant. Typically, it is positive in cholecystitis.

 

 87 The air of a foundry worker’s working zone contains condensation aerosol with dust particles sized 2 nm (90%), 2-5 nm (2%),  over 5 nm(6%), below 2 nm (about 2%). Characterize the dust dispersivity:  

A. Mist

B. Ultrafine-dispersed  

C. Coarsely dispersed  

D. Fine-dispersed*  

E. Median-dispersed

 

 88 A 17-year-old patient complains of pain in the area of the left knee joint. Soft tissues of thigh in the affected region are infiltrated,  joint function is limited. X-ray picture of the distal metaepiphysis of the left femur shows a destruction focus with periosteum  detachment and Codman’s triangle found at the defect border in the bone cortex. X-ray of chest reveals multiple small focal  metastases. What treatment is indicated?

A. Radioiodine therapy  

B. Disarticulation of the lower extremity  

C. Distance gamma-ray therapy  

D. Palliative chemotherapy*  

E. Amputation of the lower extremity  

18013

 

 89 3 days ago a 29-year-old patient presented with pulsating pain in the region of rectum, fever, general weakness. Objectively: local  tenderness in the anal region in the 6 o’clock position. Digital investigation of rectum revealed a painful infiltration reaching the  pectinate line. What is the most likely diagnosis?

A. Acute haemorrhoids  

B. Acute anal fissure  

C. Acute periproctitis*  

D. Acute prostatitis  

E. Rectum tumour

272182  

 

 90 A 30-year-old female patient has been delivered to the gynaecological department with complaints of acute pain in the lower  abdomen and body temperature 38,8oC. In history: sexual life out of wedlock and two artificial abortions. Gynaecological  examination reveals no changes of uterine. The appendages are enlarged and painful on both sides. Vaginal discharges are  purulent and profuse. What study is required to confirm a diagnosis?

A. Hysteroscopy  

B. Colposcopy  

C. Laparoscopy  

D. Bacteriological and bacterioscopic analysis*  

E. Curettage of uterine cavity  

 

 91 350 workers of a metalurgical plant had to undergo a yearly preventive examination. A territorial polyclinic carried out preventive  examination of 325 workers. As a result of it, 1 worker was recognized as temporarily disabled, 15 workers underwent further  rehabilitation at an after-work sanatorium, 10 workers were provided with diet meal. What index characterizing the preventive  work of the polyclinic should be applied in this case?  

A. Frequency of case detection during examinations  

B. Percentage of people who underwent rehabilitation at an after-work sanatorium 

C. Percentage of people who were provided with diet meal

D. Percentage of temporarily disabled people  

E. Coverage of preventive medical examinations*  

 Metalurgical Industry

Metalurgical Industry

 

 92 A patient suddenly felt an acute chest pain irradiating to the left arm. Objectively: the patient is excited, with pale skin. Breathing  rate – 38/min, AP – 180/110 mm Hg. Later the patient lost consciousness and fell down. Pulse on the great vessels was absent,  the pupils were equally dilated. What is the most likely diagnosis? 

A. Heart attack  

B. Clinical death*  

C. Coma  

D. Agonal state  

E. Disorder of the cerebral circulation  

NB! 

Clinical Death:

1. Stoppage of heart beat, pulse and breathing is called clinical death.

2. Most organs (eye, kidney) remain alive after clinical death.

3. These organs are used for transplantation.

Biological Death:

1. The death caused by degeneration of tissues in brain and other part is called biological death.

2. Most organs become dead after biological death.

3. These organs can not be used for organ transplantation.

 

93 A hospital has admitted a 52-year-old patient with disseminated pulmonary tuberculosis, complaints of acute pain in the right half  of chest, that developed after going upstairs to the 3rd floor; cough, dyspnea at rest, marked cyanosis. What kind of complication  should suspected first of all?  

A. Acute myocardial infarction  

B. Pleuritis  

C. Cardiac failure  

D. Pulmonary failure  

E. Spontaneous pneumothorax*  

 NB! Miliary tuberculosis (also known as “disseminated tuberculosis”) is a form of tuberculosis that is characterized by a wide dissemination into the human body and by the tiny size of the lesions (1–5 mm). Its name comes from a distinctive pattern seen on a chest X-ray of many tiny spots distributed throughout the lung fields with the appearance similar to millet seeds—thus the term “miliary” tuberculosis. Miliary TB may infect any number of organs, including the lungs, liver, and spleen. It is a complication of 1–3% of all TB cases.

Miliary tuberculosis

Miliary tuberculosis

Miliary tuberculosis

Miliary tuberculosis

Spontaneous Tension Pneumothorax

Spontaneous Tension Pneumothorax

Spontaneous Tension Pneumothorax

Spontaneous Tension Pneumothorax

 94 Environmental pollution is prevented by mechanical separation of nontoxic solid domestic waste. Specify the method which can be  used for mechanical utilization of these wastes:

A. Burial of wastes  

B. Burning as power-plant fuel  

C. Waste neutralization in biothermal boxes  

D. Compressing of wastes into building blocks*  

E. Hydrolysis  

 

 95 On the fifth day after a casual sexual contact a 25-year-old female patient consulted a doctor about purulent discharges from the  genital tracts and itch. Vaginal examination showed that vaginal part of uterine cervix was hyperemic and edematic. There was an  erosive area around the external orifice of uterus. There were mucopurulent profuse discharges from the cervical canal, uterine  body and appendages exhibited no changes. Bacterioscopic examination revealed bean-shaped diplococci that became red after  Gram’s staining. What is the most likely diagnosis?  

A. Acute gonorrheal endocervicitis*  

B. Candidal vulvovaginitis  

C. Clamydial endocervicitis  

D. Trichomonal colpitis  

E. Bacterial vaginism

gonorrhea

Gonorrhea_In_Women-3

 

 96 A 62-year-old patient has been delivered to the surgical department with complaints of sudden pain in the umbilical region  irradiating to the back and groin, which was accompanied by a syncope. Objectively: the patient is in grave condition, umbilical  region is tender on palpation, bowel sounds are diminished. AP drop is present. USI reveals: free fluid in the abdomen, thickening  of the wall of the abdominal aorta. The most likely diagnosis is:  

A. Rupture of abdominal aortic aneurism*  

B. Stomach ulcer perforation  

C. Peritonitis  

D. Acute appendicitis  

E. Acute pancreatitis  

Abdominal-Aortic-Aneurysm-Picture

 

 97 A 40-year-old patient underwent an operation for a lumbar phlegmon. Body temparature rose again up to 38oC, he got  intoxication symptoms, there was an increase of leukocyte number in blood. The wound that was nearly free from necrotic  tissues and full of granulations started to discharge pus, the granulations turned pale. What complication dveloped in this patient? 

A. Putrid phlegmon  

B. Sepsis*  

C. Erysipelas  

D. Erysipeloid  

E. Allergic reaction  

318009_379889645400468_1349572165_n

sepsis

 IEC Hand Washing A simple and Effective Method for Prevention of Nosocomial Sepsis-971

 98 A farmer hurt his right foot during working in a field and came to the emergency station. He doesn’t remember when he got last  vaccination and he has never served in the army. Examination of his right foot revealed a contaminated wound up to 5-6 cm long  with uneven edges. The further treatment tactics will be:

A. To make an injection of tetanus anatoxin  

B. To administer an antibiotic  

C. To make an injection of tetanus anatoxin and antitetanus serum*  

D. To make an injection of antitetanus serum  

E. Surgical d-bridement only  

Rusty nails Can cause Tetanus

Rusty nails Can cause Tetanus

Tetanus

Tetanus

Tetanus

Tetanus

 

 99 Working conditions of a building company worker are characterized by cooling microclimate effect, silica-containing dust, caustic  alkali (quicklime) and noise. What medical expert should be the chief of the commission that periodically examines the workers of  the mentioned category?  

A. Neurologist  

B. Therapeutist*  

C. Otolaryngologist

D. Dermatologist  

E. Ophthalmologist  

 

 100 A surgeon examined a 42-year-old patient and diagnosed him with right forearm furuncle at the purulo-necrotic stage. The  furuncle was lanced. At the hydration stage the wound dressing should enclose the following medication:  

A. Vishnevsky ointment  

B. Dimexide  

C. Hypertonic solution*  

D. Chloramine  

E. Ichthyol ointment  

hypertonic

 

 101 After lifting a load a patient felt undurable pain in the loin. He was diagnosed with acute lumbosacral radiculitis. Which of the  following is contraindicated for this patient?  

A. Vitamins of B group  

B. Intravenous injection of aminophylline  

C. Dehydrating drugs  

D. Analgetics  

E. Warming procedures* 

NB! Radicular pain, or radiculitis, is pain “radiated” along the dermatome (sensory distribution) of a nerve due to inflammation or other irritation of the nerve root (radiculopathy) at its connection to the spinal column. A common form of radiculitis is sciatica – radicular pain that radiates along the sciatic nerve from the lower spine to the lower back, gluteal muscles, back of the upper thigh, calf, and foot as often secondary to nerve root irritation from a spinal disc herniation or from osteophytes in the lumbar region of the spine.

 lumbarradicular1

Disk Herniation

Disk Herniation

Sciatic-Nerve-Pain-Relief-111 

102 A 49-year-old patient complains of deglutition problems, especially with solid food, hiccups, voice hoarseness, nausea,  regurgitation, significant weight loss (15 kg within 2,5 months). Objectively: body weight is reduced. Skin is pale and dry. In lungs:  vesicular breathing, heart sounds are loud enough, heart activity is rhythmic. The abdomen is soft, painless on palpation. Liver is  not enlarged. What study is required to make a diagnosis?

A. Esophageal duodenoscopy along with biopsy*  

B. Study of gastric secretion  

C. X-ray in Trendelenburg’s position  

D. Clinical blood test  

E. X-ray of digestive tract organs  

swallow

egd

 

 103 A 26-year-old woman complains of having bloody discharges from the genitals for the last 14 days, abdominal pain, general  fatiguability, weakness, weight loss, fever, chest pain, obstructed respiration. 5 weeks ago she underwent an induced abortion in  the 6-7 week of gestation. Objectively: the patient is pale and inert. Bimanual examination revealed that the uterus was enlarged  up to 8-9 weeks of gestation. In blood: Hb – 72 g/l. Urine test for chorionic gonadotropin gave the apparently positive result. What  is the most likely diagnosis?

A. Uterine fibromyoma  

B. Metroendometritis  

C. Uterus perforation  

D. Chorioepithelioma*  

E. Uterine carcinoma  

NB! Chorioepithelioma is a malignant, fast-growing tumor that develops from trophoblastic cells (cells that help an embryo attach to the uterus and help form the placenta). Almost all chorioepitheliomas form in the uterus after fertilization of an egg by a sperm, but a small number form in a testis or an ovary. Chorioepitheliomas spread through the blood to other organs, especially the lungs. They are a type of gestational trophoblastic disease. Also called chorioblastoma, choriocarcinoma, and chorionic carcinoma.

 Complete hydatidiform moles (Molar pregnancy)  have a higher risk of developing into choriocarcinoma — a malignant tumor of trophoblast cells — than do partial moles.

hydatidiformmolelabel

 

 104 After a 10-year-old child had been bitten by a bee, he was delivered to a hospital. There were lip, face and neck edemata. The  patient felt hot and short of breath. Objectively: breathing was laboured and noisy. There were foamy discharges from the mouth,  cough. The skin was pale and cold. There was bradypnoea. Heart sounds were muffled and arrhythmic. Thready pulse was  present. What diagnosis was made by the expert in resuscitation?  

A. Acute cardiovascular collapse  

B. Cerebral coma  

C. Anaphylactic shock*  

D. Quincke’s edema  

E. Bronchial asthma  

anaphylactic-shock-source

 

 105 After birth a child was pale and had arrhythmical breathing. Oxygen therapy didn’t have any effect. Pulse was weak and rapid. It  was difficult to measure arterial pressure accurately. There were no edemata. What is the most likely reason for these  symptoms?

A. Congestive heart failure  

B. Intracranial haematoma  

C. Asphyxia*  

D. Congenital pneumonia  

E. Intrauterine sepsis  

 

 106 A parturient woman is 25 years old, it is her second day of postpartum period. It was her first full-term uncomplicated labour. The  lochia should be:

A. Bloody*  

B. Purulent  

C. Mucous

D. Sanguino-serous

E. Serous

NB !  Lochia is the vaginal discharge for the first fortnight of puerperium (after birth), containing blood, mucus, and placental tissue. Lochia discharge typically continues for 4 to 6 weeks after childbirth.

It is sterile for the first 2-3 days, but not so by the third or fourth day, as the uterus begins to be colonized by vaginal commensals such as non-hemolytic streptococci and E. coli.

lochia1347311163147

 107 A full-term baby (the 1st uncomplicated pregnancy, difficult labour) had a cephalohematoma. On the 2nd day there was jaundice,  on the third the following changes in neurological status appeared: nystagmus, Graefe syndrome. Urine was yellow, feces were  of golden-yellow colour. Mother’s blood group is A (II) Rh, the baby’s one – A (II) Rh+. On the third day the child’s Hb was  200g/l, RBCs – 6,1*1012/l, blood bilirubin – 58 micromole/l at the expense of unbound fraction. What caused the jaundice in  the child?  

A. Fetal hepatitis  

B. Neonatal anaemia  

C. Biliary atresia  

D. Physiological jaundice  

E. Craniocerebral birth trauma*  

Cephalohematoma

Cephalohematoma

 

 108 A 25-year-old female patient complains about having amenorrhea for 3 years. She associates it with difficult labour complicated  by massive hemorrhage. She also complains of loss of weight, hair fragility and loss, lack of appetite and depression. Objective  examination reveals no pathological changes of uterus and its appendages. What is the desease pathogenesis?

A. Hypoproduction of progesterone

B. Hyperproduction of androgens  

C. Hyperproduction of prolactin

D. Hypoproduction of gonadotropin*  

E. Hyperproduction of estrogens  

 

 109 On the 1st day of life a full-term girl (2nd labour) weighing 3500g, with Apgar score of 8 points, presented with jaundice. Indirect  bilirubin of blood – was 80 micromole/l, 6 hours later – 160 micromole/l. What is the optimal method of treatment?  

A. Enterosorbents  

B. Exchange blood transfusion*  

C. Phenobarbital treatment  

D. Phototherapy  

E. Infusion therapy  

 

 110 A 14-year-old girl complains of pain in vaginal area and lower abdomen that last for 3-4 days and have been observed for 3  months about the same time. Each time pain is getting worse. Objectively: mammary glands are developed, hairiness corresponds  to the age. The virginal membrane is intact, cyanotic and protruded. She has never had menstruation. She has been diagnosed  with primary amenorrhea. What is the reason of amenorrhea?  

A. Turner’s syndrome  

B. Sexual development delay  

C. Babinski-Frohlich syndrome  

D. Pregnancy  

E. Hymen atresia*

v4c08a 

 

 111 A 42-year-old patient complains of back pain, darkened urine, general weakness, dizziness that occurred after treating a cold  with aspirin and ampicillin. Objectively: the patient is pale, with subicteric sclerae. HR – 98 bpm. Liver – +2 cm, spleen – +3 cm. In  blood: RBCs – 2,6*1012/l, Hb – 60 g/l, CI – 0,9, WBCs – 9,4*109/l, basophils – 0,5%, eosinophils – 3%, stab  neutrophils – 6% segmented neutrophils – 58%, lymphocytes – 25%, monocytes – 7%, ESR – 38 mm/hour, reticulocytes – 24%.  Total bilirubin – 38 millimole/l. What complication occurred in the patient?  

A. Cholelithiasis  

B. Agranulocytosis  

C. Paroxysmal nocturnal hemoglobinuria  

D. Acquired hemolytic anemia*  

E. Toxic hepatitis  

Fig2-HereditaryAnemia

 

 

 112 A child is 1 year old. Ater the recent introduction of complementary feeding the child has presented with loss of appetite, diarrhea  with large amounts of feces and occasional vomiting, body temperature is normal. Objectively: body weight is 7 kg, the child is  very pale, there are edemata of both legs, abdomen is significantly enlarged. Coprogram shows many fatty acids and soaps. The  child has been diagnosed with celiac disease and administered the gluten-free diet. What is to be excluded from the ration?  

A. High digestible carbohydrates  

B. Milk and dairy products  

C. Fruit  

D. Cereals – wheat and oats*  

E. Animal protein  

Celiac disease is an increasingly diagnosed chronic inflammatory disorder of the intestine. Damage to the intestine is due to a specific immune response triggered by certain peptides derived from gluten proteins in wheat, rye and barley, and sometimes oats.

Celiac disease is an increasingly diagnosed chronic inflammatory disorder of the intestine. Damage to the intestine is due to a specific immune response triggered by certain peptides derived from gluten proteins in wheat, rye and barley, and sometimes oats.

 

 113 15 minutes after the second vaccination with DTP vaccine a 4-month-old boy exhibited the symptoms of Quincke’s edema. What  medication should be given for emergency aid?  

A. Adrenalin

B. Heparin

C. Seduxen

D. Prednisolone*  

E. Furosemide

 Quincke's edema (Angioedema)

Quincke’s edema (Angioedema)

 

 114 A 10-year-old boy periodically has short states (up to 10-15 seconds) that can be characterized as a “sudden blackout” and are  accompanied by gaze fixation in the upright position, absent-minded and vacant face expression, lack of movements and following  amnesia. Specify this state:  

A. Thought block  

B. Fugue  

C. Absence*  

D. Trance  

E. Obnubilation  

 

 115 Examination of the corpse of a man who died from hanging reveals: death spots disappear when pressed upon and restore after  50 seconds, rigor mortis is moderately expressed only in the masticatory muscles as well as neck and finger muscles, body  temperature is of 31oC. The time of death:  

A. 6-7 hours ago*  

B. 10-18 hours ago  

C. 16-24 hours ago  

D. 1-2 hours ago  

E. 8-10 hours ago  

 

 116 A 42-year-old woman has had hyperpolymenorrhea and progressing algodismenorrhea for the last 10 years. Gynaecological  examination revealed no changes of uterine cervix; discharges are moderate, of chocolate colour, uterus is slightly enlarged and  painful, appendages are not palpable, the fornices are deep and painless. What is the most likely diagnosis?  

A. Uterine carcinoma  

B. Uterine endometriosis*  

C. Endomyometritis  

D. Subserous uterine fibromyoma  

E. Adnexal endmetriosis  

1

 

 117 A municipal hospital reported on the number of operated patients including fatal outcomes following the operations. Which index of  hospital work can be calculated on the ground of this data?  

A.

B. Standardized lethality  

C. Total lethality  

D. Postoperative lethality*  

E. Index of late hospitalization since a disease incursion  

 

 118 A therapeutist needs to analyze adult health in the area of service. Which groups of indicators will be included into this analysis?  

A. Demographic, sickness rates, disability*  

B. Demographic, sickness rates, physical development  

C. Sickness rates, disability, death rates  

D. Birth rates, sickness rates, disability  

E. Sickness rates, death rates, physical development  

 

 119 A 54-year-old male patient complains of aching pain in the lumbar region, that is getting worse after standing in an upright position,  physical exercise, supercooling. The patient also reports of experiencing weakness in the afternoon. Pain in the lumbar region,  said about 10 years old. Objectively: pale skin, to– 37,2oC, AP- 180/100 mm Hg, minor costovertebral angle tenderness  (Pasternatsky symptom). In blood: RBCs – 3,5*1012/l, WBCs – 6,5*109/l, ESR – 22 mm/h. In urine: the relative  density – 1010, leukocytes – 12-15 in the field of vision, erythrocytes – 2-3 in the field of vision. Urine bacterial count – 100000 in 1  ml. What is the most likely diagnosis?  

A. Chronic glomerulonephritis  

B. Polycystic renal disease  

C. Amyloidosis  

D. Nephrolithiasis

E. Chronic pyelonephritis*  

utisrx

 

 120 An 8-year-old girl has been admitted to the cardiology department. Objectively: there is a skin lesion over the extensor surfaces of  joints with atrophic cicatrices, depigmentation, symmetrical affection of skeletal muscles (weakness, edema, hypotrophy). What  disease are these changes most typical for?  

A. Reiter’s disease  

B. Systemic scleroderma  

C. Dermatomyositis*

D. Nodular periarteritis  

E. Systemic lupus erythematosus

Dermatomyositis (in Child)

Dermatomyositis (in Child)

Dermatomyositis at Fingernail

Dermatomyositis at Fingernail

 

 

 121 14 days ago a 49-year-old patient was operated on for perforated appendicitis and disseminated fibrinopurulent peritonitis. The  postoperative period was uneventful. On the 9th day the patient presented with low-grade fever, abdominal pain, frequent liquid  stools. USI of the left mesogastrium reveals a fluid formation 9×10 cm large. In blood: leukocytosis with the left shift. What is your  provisional diagnosis?  

A. Left kidney cyst  

B. Interloop abscess*  

C. Abdominal cavity tumour  

D. Liver abscess  

E. Spleen abscess

Interloop abscess = Surgically removed. if the abscess drains and recurs, there is probably some residual bacteria within the wall and interior of the abscess cavity. in that case, it makes sense to completely remove the abscess after the inflammation has calmed down, including the entire wall, to prevent recurrence. 

 

 122 A 3-year-old child has been delivered to a hospital in soporose state with considerable amyotonia, inhibition of tendon and  periosteal reflexes. Miosis and asthenocoria are also present. Corneal reflexes are preserved. Pulse is rapid and weak. AP- 80/50  mm Hg. The parents suspect the child of accidental taking some tablets. Such clinical presentations are typical for intoxication  with the following tableted drugs:

A. Barbiturates  

B. Antihypertensive drugs  

C. Beta-2-adrenoceptor agonists  

D. Antropine drugs  

E. Tranquilizers*  

Tranquilizers

 

 123 Topographic percussion of lungs in a patient who got a serious job-related barotrauma revealed that the lower lungs borders  were located one rib below normal, there was a significant increase in both lungs height and Kronig’s isthmus. What disease  should be suspected in the first place?  

A. Bronchial asthma  

B. Chronic bronchitis  

C. Pneumothorax  

D. Exudative pleuritis  

E. Pulmonary emphysema*

Kronig’s Isthmus = The narrow straplike portion of the resonant field that extends over the shoulder, connecting the larger areas of resonance over the pulmonary apex in front and behind.

h9991437_001

Signs of hyperinflation can be seen in emphysema, chronic bronchitis and asthma. We can call it emphysema only when hyperinflation is associated with blebs and paucity of vascular markings in the outer third of the film.

Signs of hyperinflation can be seen in emphysema, chronic bronchitis and asthma. We can call it emphysema only when hyperinflation is associated with blebs and paucity of vascular markings in the outer third of the film.

 

 124 A 69-year-old female patient complains of temperature rise up to 38,3oC, haematuria. ESR – 55 mm/h. Antibacterial therapy  turned out to be ineffective. What diagnosis might be suspected?

A. Urolithiasis  

B. Chronic glomerulonephritis  

C. Renal amyloidosis  

D. Renal cancer*

E. Polycystic renal disease  

 

 125 A 62-year-old male has been hospitalized in the intensive care unit with a continuous attack of retrosternal pain that cannot be  relieved by nitroglycerin. Objectively: AP- 80/60 mm Hg, heart rate – 106/min, breathing rate – 22/min. Heart sounds are muffled, a  gallop rhythm is present. How would you explain the AP drop?  

A. Reduction in peripheral resistance  

B. Internal haemorrhage  

C. Reduction in cardiac output*  

D. Blood depositing in the abdominal cavity  

E. Adrenergic receptor block  

 

 126 As a result of lifting a load a 62-year-old female felt acute pain in the lumbar region, in a buttock, posterolateral surface of her right  thigh, external surface of the right shin and dorsal surface of foot. Objectively: weakness of the anterior tibial muscle, long  extensor muscle of the right toes, short extensor muscle of the right toes. Low Achilles reflex on the right. Positive Lasegue’s  sign. What examination method would be the most effective for specification of the diagnosis of discogenic compression of L5  root?  

A. Magnetic resonance scan*  

B. Spinal column X-ray  

C. Angiography  

D. Electromyography  

E. Lumbar puncture  

 

 127 A children’s surgical unit admitted a 1-month-old boy who had been prenatally diagnosed with the left-sided pyelectasis. Such  studies as drip infusion urography, cystography and USI allowed to reveal initial hydronephrosis. There is no information  confirming the secondary pyelonephritis. What tactics of this patient management is most advisable?

A. There is no need in further surveillance and treatment  

B. Antibacterial therapy  

C. Urgent nephrostomy  

D. Anderson-Hynes operation  

E. 6-month surveillance*  

 

 

 128 A 44-year-old patient has been admitted to a hospital with complaints of dull, aching pain in the left lumbar region, the admixture of  pus in the urine. Examination revealed a grade II staghorn calculus on the left. What method of treatment is indicated for this  patient?  

A. Surgery*  

B. Conservative therapy  

C. Ascending litholysis  

D. Contact lithotripsy  

E. Distance lithotripsy  

 

 129 A 72-year-old patient complains of pain and bleeding during defecation. Digital rectal investigation revealed a tumour of the anal  canal. After verification of the diagnosis the patient was diagnosed with squamous cell carcinoma. The secondary (metastatic)  tumour will be most probably found in:  

A. Brain

B. Liver

C. Mediastinum

D. Pelvic bones

E. Lungs*

 

 130 A welder at work got the first-degree burns of the middle third of his right shin. 5 days later the skin around the burn became  edematic and itchy. Objectively: on a background of a well-defined erythema there is polymorphic rash in form of papules,  vesicles, pustules, erosions with serous discharge. What is the most likely diagnosis?  

A. Streptococcal impetigo  

B. True eczema  

C. Toxicoderma  

D. Occupational eczema  

E. Microbal eczema*  

 

 131 An emergency team deliverd a 83-year-old patient complaining of inability of her right leg to support the body after falling on her  right side. Objectively: the patient lies on a gurney, her right leg is rotated outwards, the outside edge of foot touches the bed.  There is positive straight leg raising sign. What is your provisional diagnosis?  

 

A. Hip joint contusion

B. Femoral diaphysis fracture  

C. Hip dislocation  

D. Cotyloid cavity fracture  

E. Femoral neck fracture*  

 

 132 After contusion of the right eye a patient complains of sudden loss of vision with remaining light perception. Objectively: the eye is  not irritated. The cornea is transparent. Pupil reacts to light. The pupil area is black. The fundus reflex is absent. What is the most  likely cause of vision loss?

A. Acute occlusion of retinal vessels  

B. Traumatic cataract  

C. Optic nerve avulsion  

D. Retinal detachment  

E. Hemophthalmia*  

 

 133 A patient has an over a year-old history of fast progressive rheumatoid arthritis. X-raying confirms presence of marginal erosions.  What basic drug would be the most appropriate in this case?  

A. Chloroquine  

B. Prednisolone  

C. Diclofenac sodium  

D. Aspirin  

E. Methotrexate*  

 

 134 At first appointment with an obstetrician-gynaecologist a pregnant woman is referred to other medical specialists. She must be  obligatory examined by the following specialists:  

A. Therapeutist and endocrinologist  

B. ENT and ophthalmologist

C. Therapeutist and dentist*  

D. Dentist and phthisiatrician  

E. Dentist and cardiologist  

 

 135 A 27-year-old patient complains of nasal haemorrhages, multiple bruises on the anterior surface of the trunk and extremities,  sudden weakness. In blood: Hb- 74 g/l, reticulocytes – 16%, RBCs – 2,5*1012/l, platelets – 30*109/l, ESR- 25  mm/h. What is the most effective measure for the treatment of thrombocytopenia?

A. Splenectomy*  

B. Iron preparations  

C. Cytostatics  

D. Hemotransfusion  

E. Vitamin B12

 

 136 A 47-year-old patient came to see a doctor on the 7th day of disease. The disease developed very fast: after the chill body  temperature rose to 40oC and lasted up to 7 hours, then dropped abruptly, which caused profuse sweat. There were three  such attacks occuringonce in two days. Two days ago the patient arrived from Africa. Objectively: pale skin, subicteric sclera,  significantly enlarged liver and spleen. What is the cause of fever attacks in this disease?  

A. Exotoxin of a causative agent  

B. Endotoxin of a causative agent  

C. Tissue schizogony  

D. Erythrocytic schizogony*  

E. Gametocytes  

 

 137 According to the report of water quality control, drinking city water has the following characteristics: turbidity – 1,5 mg/m3,  odour – 3 points, metallic taste – 2 points, pale yellow colour, colour index – 20o, temperature – 12o. Which of these factors  doesn’t comply with hygienic requirements?  

A. Taste  

B. Turbidity  

C. Odour*  

D. Colour index  

E. Temperature  

 

 138 Half an hour after a 30-year-old woman had had some custard cake, she experienced lancinating abdominal pain, nausea,  vomiting. Objectively: body temperature – 36,0oC, pale skin, breathing rate – 20/min, Ps- 100/min. AP- 95/65 mm Hg, loud cardiac  sounds. Dry tongue. Abdomen was painful in its epigastrial part, there were no signs of peritoneum irritation. What is the first  measure to be taken?  

A. Injection of Cerucal  

B. Antibiotic therapy  

C. Intravenous rehydratation  

D. Administration of an enterosorbent  

E. Gastric lavage*  

 

 139 A patient has a stab wound on his right foot. On the fourth day after injury the patient’s body temperature rose up to 38oC,  inguinal lymph nodes became enlarged and painful, skin over them reddened. What complication might be suspected?  

A. Lymphangitis

B. Phlegmon

C. Tetanus

D. Erysipelas

E. Lymphadenitis*  

 

 140 A teacher of a secondary school was diagnosed with pulmonary tuberculosis. What is the maximum duration of his medical  certificate?  

A. Four months  

B. Two months  

C. Ten months*  

D. A month  

E. Five months

 

 141 A 39-year-old patient complains of a tumour on the anterior surface of her neck. The tumour has been observed for 2 years. It is  nonmobile and has enlarged recently. The patient has a changed tone of voice, a sense of pressure. Objectively: in the left lobe of  the thyroid gland a 3 cm node is palpable; it is very dense, tuberous, painless. Cervical lymph nodes are enlarged. Functional  status of the thyroid gland is unchanged. What is the most likely diagnosis?

A. Thyroid gland cancer*  

B. Nodular euthyroid goiter  

C. Nodular hyperthyroid goiter  

D. Chronic fibrous Riedel’s thyroiditis  

E. Chronic lymphomatous Hashimoto’s thyroiditis  

 

 142 A child is 2 years old. The child complains of hoarse voice, dyspnea with obstructed inspiration. The disease started 3 days ago  from dry cough and nose stuffiness. Objectively: general condition is unbalanced, stridor is present. The child’s skin is pale. Body  temperature is 37,7oC. The palatine arches are hyperemic. There is no deposit. Heart sounds are rhythmic. Auscultation of  lungs reveals rough breathing sounds, crepitation is absent. Parainfluenza virus has been detected in nasopharynx lavage. What  is the most likely diagnosis?

A. Epiglottitis  

B. Foreign body  

C. Acute laryngotracheitis*  

D. Laryngospasm  

E. Diphtheria  

 

 143 A 58-year-old patient complains of a headache in the occipital region, nausea, choking, opplotentes. The presentations appeared  after a physical exertion. Objectively: the patient is excited. Face is hyperemic. Skin is pale. Heart sounds are regular, the 2nd  aortic sound is accentuated. AP- 240/120 mm Hg, HR- 92/min. Auscultation reveals some fine moisr rales in the lower parts of the  lungs. Liver is not enlarged. ECG shows signs of hypertrophy and left ventricular overload. What is the most likely diagnosis?

A. Uncomplicated hypertensic crisis  

B. Community-acquired pneumonia  

C. Bronchial asthma exacerbation  

D. Acute myocardial infarction, pulmonary edema  

E. Complicated hypertensic crisis*  

 

 144 A 28-year-old woman has a 12-year history of chronic glomerulonephritis with latent course. Over the past six months she has  developed general weakness, loss of appetite, low work performance, nausea. The patient complains of headache, pain in the  joints. On examination: anemia, blood urea – 34,5 millimole/l, blood creatinine – 0,766 millimole/l, hyperkalemia. What complication has  developed?  

A. Chronic renal insufficiency*

B. Acute renal insufficiency

C. Nephrotic syndrome

D. Renal amyloidosis  

E. Pyelonephritis  

 

 145 A 43-year-old patient had been admitted to a hospital with clinical presentations of ischiorectal periproctitis. On the 12th day of  treatment the patient’s condition deteriorated: there was an increase in the rate of intoxication and hepatic failure, the body  temperature became hectic, AP was 100/60 mm Hg. USI of liver revealed a hydrophilic formation. In blood: WBCs –  19,6*109/l, RBCs.- 3,0*1012/l, Hb- 98 g/l. What complication was developed?

A. Liver abscess*  

B. Pylephlebitis  

C. Hepatic cyst  

D. Budd-Chiari syndrome  

E. Liver necrosis  

 

 146 Examination of a 9-month-old girl revealed skin pallor, cyanosis during excitement. Percussion revealed transverse dilatation of  cardiac borders. Auscultation revealed continuous systolic murmur to the left of the breastbone in the 3-4 intercostal space. This  murmur is conducted above the whole cardiac region to the back. What congenital cardiac pathology can be suspected?  

A. Defect of interatrial septum  

B. Pulmonary artery stenosis  

C. Coarctation of aorta

D. Defect of interventricular septum*

E. Fallot’s tetrad

 

 147 An 8-month-old baby has had problems with nasal breathing and muco-purulent discharge from the nose for a week. Examination  reveals a rhinedema, muco-purulent discharges from the middle nasal meatus as well as on the back of pharynx. What disease  are these symptoms most typical for?

A. Hemisinusitis  

B. Sphenoiditis  

C. Ethmoiditis*  

D. Maxillary sinusitis  

E. Frontitis  

 

 148 A 22-year-old female patient complains of dull pain in her right iliac area that she has been experiencing for a week, morning  sickness and gustatory change. She has a histrory of menstruation delay for 3 weeks. Objectively: AP- 80/50 mm Hg, pulse is 78 bpm, body temperature is 37oC. Bimanual examination reveals that uterus is enlarged, soft, mobile and painless. Uterine  appendages are palpable on the right, there is a dense, elastic and moderately painful formation 3×4 cm large. What is the most  likely diagnosis?

A. Uterogestation  

B. Interrupted fallopian pregnancy  

C. Acute appendicitis  

D. Progressing fallopian pregnancy*  

E. Right ovarian cyst  

 

 149 2 days ago a patient presented with acute pain in the left half of chest, general weakness, fever and headache. Objectively:  between the 4 and 5 rib on the left the skin is erythematous, there are multiple groups of vesicles 2-4 mm in diameter filled with  transparent liquid. What diease are these symptoms typical for?  

A. Herpes zoster* 

B. Herpetiform Duhring’s dermatosis  

C. Pemphigus  

D. Streptococcal impetigo  

E. Herpes simplex  

 

 150 A 62-year-old patient complaining of enlargement of cervical, supraclavicular and axillary lymph nodes, subfebrile temperature for  the last 3 months has been admitted to a hospital. In blood: WBCs – 64109/l, lymphocytes – 72%. What method of study  should be used to specify the diagnosis?  

A. Thermography  

B. Lymphoscintigraphy  

C. Myelogram*  

D. Lymphography  

E. X-rays  

 

 151 A patient is 30 years old, works as a carpenter. Six months ago there appeared some behavioural  changes: he got interested in  philosophy, began writing a treatise on the purpose of his human existence, quitted his job, stopped caring about his children,  went  out carelessly dressed, heard “voices in his head” that guided his behaviour. The patient claimed sure that he was an  Ambassador of God on Earth and was constantly feeling His influence. He is not critical about his disease. What diagnosis can be  assumed?  

A. Alcocholic psychosis  

B. Reactive psychosis  

C. Organic psychosis  

D. Schizophrenia*  

E. Somatogenic psychosis  

 

 152 Examination of a 38-year-old patient who had been hit with a blunt object on the left side of chest revealed a fracture of the X rib  with fragments displacement, parietal pneumothorax. The patient complains of pain in the left subcostal area. Objectively: the  patient is pale, AP- 80/40 mm Hg, Ps- 138/min, of poor volume. USI reveals fluid in the left abdomen. Splenic rupture is present.  What treatment tactics should be chosen?  

A. Anti-schock actions and laparotomy following the arterial pressure rise  

B. Drainage of the left pleural cavity and laparotomy*  

C. Left-sided thoracotomy and immediately following laparotomy  

D. Immediate laparotomy and alcohol-novocaine block of the X rib  

E. Immediate upper middle laparotomy and following drainage of the left pleural cavity  

 

 153 A 3-year-old girl presents with pertussis-like cough with thick sputum. There have been persistent changes in lungs since the age  of 6 months when she was first diagnosed with acute pneumonia. Chloride concentration in the perspiration is 112 mEq/l. The  child has been diagnosed with mucoviscidosis. What is the basis for autosomal recessive disease – mucoviscidosis?  

A. Inadequate transport of sodium and chloride ions*  

B. Deposition of calcium triphosphates and carbotates in the alveoles  

C. Pulmonary cysts  

D. a1-antitrypsin deficiency  

E. Pulmonary artery hypoplasia  

 

 154 A 47-year-old woman underwent a thyroid gland resection on ccount of nodular euthyroid goiter. What preparations are most  likely to prevent the disease recurrence?  

A. Thyroid hormones*  

B. Antistruminum (potassium iodide)  

C. Mercazolil  

D. Thyrotropin  

E. Radioactive iodine  

 

 155 After a car accident a 37-year-old patient has an acute pain and bleeding in the middle third of his right thigh. Objectively: there is a  wound on the anterior surface of the right thigh with massive bleeding, abnormal mobility at the level of the middle third of the  thigh. The first aid is to be started with:  

A. Injection of pain-killers  

B. Immobilization of the extremity with a transportation splint  

C. Digital occlusion of the femoral artery*  

D. Venipuncture and intravenous infusion of polyglycine  

E. Tourniquet application  

 

 156 A 28-years-old woman complains of nausea and vomiting about 10 times per day. She has been found to have body weight loss  and xerodermia. The pulse is 100 bpm. Body temperature is 37,2oC. Diuresis is low. USI shows 5-6 weeks of pregnancy.  What is the most likely diagnosis?  

A. Premature abortion  

B. I degree preeclampsia  

C. Food poisoning  

D. Moderate vomiting of pregnancy*  

E. Mild vomiting of pregnancy  

 

 157 Development of chronic venous insufficiency of lower extremities depends on the functional condition of so-called  musculovenous pump. This term refers to the following group of muscles:

A. Foot muscles  

B. Abdominal wall muscles  

C. Thigh muscles  

D. Shin muscles*  

E. Buttock region muscles  

 

 158 On the 2 nd day of illness a 27-year-old patient complains of the unbearable headache, repeated vomiting. Objectively: the patient  is in a grave condition. He is conscious but adynamic. Lies in a forced position with his head thrown back. There is no skin rash.  Occipital muscles are evidently rigid, there are Kernig’s and Brudzinski’s signs. to– 39,5oC, Ps- 120/min., AP- 130/80 mm Hg.  What is the reason for the leading syndrome of this disease?

A. Liquor hypotension  

B. Liquor hypertension*  

C. Hyperthermy  

D. Haemorrhages into the adrenal glands  

E. Affection of the cranial nerve nuclei  

 

 159 A full-term baby was born with body weight of 3200 g, body length of 50 cm, Apgar score – 8-10 points. What is the optimum time  for the first breast-feeding?  

A. First 48 hours  

B. First 6 hours  

C. First 24 hours  

D. After 48 hours 

E. First 30 minutes*  

 

 160 A 40-year-old female patient complain of headache, dizziness, muscle weakness, sometimes – cramps in the extremities. She has  been taking antihypertensive medications for 10 years. AP- 180/100 mm Hg. Blood potassium – 1,8 millimole/l, sodium – 4,8  millimole/l. In urine: alkaline reaction, the relative density – 1012, protein and sugar are not found, WBCs – 3-4 in the field of vision,  RBCs – 1-2 in the field of vision. Conn’s syndrome is suspected. Which drug should be chosen for the treatment of arterial  hypertension?  

A. Propanolol  

B. Hydrochlorothiazide  

C. Clonidine  

D. Enalapril  

E. Spironolactone*  

 

 161 Condition of a patient with purulent otitis has abruptly deteriorated: he presents with headache, vomiting, febrile temperature,  general hyperesthesia. There are meningeal signs, papilledemas. Focal symptoms are absent. Cerebrospinal fluid is turbid,  pressure is high, there is albuminocytologic dissociation with neutrophil predominance. What disease can be suspected?

A. Secondary purulent meningitis*  

B. Meningoencephalitis  

C. Serous meningitis

D. Subarachnoid haemorrhage  

E. Primary purulent meningitis

 

 162 On the 6th day of life a child got multiple vesicles filled with seropurulent fluid in the region of occiput, neck and buttocks. General  condition of the child is normal. What disease should be suspected?

A. Miliaria

B. Vesiculopustulosis*

C. Impetigo neonatorum

D. Epidermolysis bullosa

E. Impetigo

 

 163 A secundipara has regular birth activity. Three years ago she had cesarean section for the reason of acute intrauterine hypoxia.  During parodynia she complains of extended pain in the area of postsurgical scar. Objectively: fetus pulse is rhythmic – 140 bpm.  Vaginal examination shows 5 cm cervical dilatation. Fetal bladder is intact. What is the tactics of choice?  

A. Waiting tactics of labor management  

B. Cesarean section*  

C. Vaginal delivery  

D. Augmentation of labour  

E. Obstetrical forceps  

 

 164 A 43-year-old female patiet complains of eruption on her right leg skin, pain, weakness, body temperature rise up to 38oC. The  disease is acute. Objectively: there is an edema on the right leg skin in the region of foot, a well-defined bright red spot in form of  flame tips which feels hot. There are isolated vesicles in focus. What is your provisional diagnosis?

A. Haemorrhagic vasculitis

B. Erysipelas*  

C. Toxicoderma  

D. Contact dermatitis  

E. Microbial eczema  

 

 165 A 56-year-old patient with diffuse toxic goiter has ciliary arrhythmia with pulse rate of 110 bpm, arterial hypertension, AP- 165/90  mm Hg. What preparation should be administered along with mercazolil?  

A. Verapamil  

B. Procaine hydrochloride  

C. Propranolol*

D. Corinfar  

E. Radioactive iodine  

 

 166 A hospital admitted a patient with coarse breathing (obstructed inspiration), skin cyanosis, tachycardia and arterial hypertension.  He has a histrory of bronchial asthma. An hour ago he was having salbutamol inhalation and forgot to remove a cap that was  aspired while taking a deep breath. What measures should the doctor take?

A. Perform conicotomy immediately  

B. Make a subcutaneous injection of dexamethasone  

C. Perform the Heimlich manoever*  

D. Send for an anesthesiologist and wait for him

E. Use an inhalation of b2-adrenoceptor agonist  

 

 167 Forensic medical expertise of corpse of a newborn revealed: body weight 3500 g, body length 50 cm, the umbilical cord was  smooth, moist, glossy, without any signs of drying. Hydrostatic tests were positive. The test results are the evidence of:

A. Stillbirth  

B. Live birth*  

C. Hyaline membrane disease  

D. Primary atelectasis  

E. Secondary atelectasis  

 

 168 After an accident a patient complains of pain in the hip joint. Objectively: the leg is in the position of flexion, adduction and internal  rotation, significantly contracted. There is elastic resistance to passive adduction or abduction of the extremity. Major trochanter is  located high above the Roser-Nelaton line. A significant lordosis is present. What  is your provisional diagnosis?

A. Femoral neck fracture with a displacement  

B. Iliac dislocation of hip*  

C. Pertrochanteric fracture of hip  

D. Inferoposterior dislocation of hip  

E. Cotyloid cavity fracture with a central dislocation of hip  

 

 169 A 6-year-old girl drank some coloured fizzy drink which gave her a feeling of pressure in the throat. 30 minutes later the child’s  lips got swollen, then edema gradually spread over the whole face, laryngeal breathing became difficult. The child is excited. Ps-  120/min, breathing rate – 28/min, breathing is noisy, indrawing of intercostal spaces is observed. What basic aid is most  appropriate for the restoration of laryngeal breathing?

A. Conicotomy  

B. Sedative drugs  

C. Tracheostomy  

D. Corticosteroids*  

E. Antibacterial drugs  

 

 170 A 30-year-old male patient consulted a family doctor 2 months after he had been operated for an open fracture of brachial bone.  Objectively: the patient’s condition is satisfactory, in the region of the operative wound there is a fistula with some purulent  discharge, redness, fluctuation. X-ray picture shows brachial bone destruction with sequestra. What complication arose in the  postoperative period?  

A. Suture sinus  

B. Hematogenic osteomyelitis  

C. Posttraumatic osteomyelitis*  

D. Wound abscess  

E. Posttraumatic phlegmon  

 

 171 A 35-year-old patient has been in the intensive care unit for acute renal failure due to crush for 4 days. Objectively: the patient is  inadequate. Breathing rate – 32/min. Over the last 3 hours individual moist rales can be auscultated in lungs. ECG shows high T  waves, right ventricular extrasystoles. CVP – 159 mm Hg. In blood: the residual nitrogen – 62 millimole/l, K+– 7,1 millimole/l, Cl– 78 millimole/l, Na+– 120 millimole/l, Ht – 0,32, Hb – 100 g/l, blood creatinine – 0,9 millimole/l. The most appropriate method of  treatment would be:

A. Plasma sorption  

B. Hemodialysis*  

C. Ultrafiltration  

D. Plasma filtration

E. Hemosorption  

 

 

 172 A 46-year-old patient complains of sudden palpitation, which is accompanied by pulsation in the neck and head, fear, nausea. The  palpitation lasts for 15-20 minutes and is over after straining when holding her breath. What kind of cardiac disorder may be  suspected?  

A. An attack of atrial flutter  

B. An attack of ciliary arrhythmia  

C. An attack of ventricular paroxysmal tachycardia  

D. An attack of supraventricular paroxysmal tachycardia*  

E. An attack of extrasystolic arrhythmia  

 

 173 A 32-year-old patient has a 3-year history of asthma attacks, that can be hardly stopped with berotec. Over a few last months he  has experienced pain in the joints and sensitivity disorder of legs and feet skin. Ps – 80/min, AP – 210/100 mm Hg. In blood:  eosinophilia at the rate of 15%. What disease can be suspected in this case?  

A. Periarteritis nodosa*  

B. Wegener’s disease  

C. Dermatomyositis  

D. Systemic lupus erythematosus  

E. Systemic scleroderma  

 

 174 A 3-year-old child was playing in a playpen when he suddenly developed paroxysmal cough and shortness of breath. Objectively:  dry cough, mixed dyspnea. Lung auscultation revealed some wheezes. Breathing sounds on the right are diminished. The child  doesn’t mix with other children. Immunization is age-appropriate. What pathological condition can be suspected?  

A. Foreign body in the respiratory tracts*  

B. Pneumonia  

C. Bronchial asthma  

D. Pertussis  

E. Acute respiratory viral infection  

 

 175 Bacterial analysis of air in a living space in winter period by means of Krotov’s apparatus revealed that total number of  microorganisms in 1m3 of air was 7200. What is the permissible number of microorganisms for the air to be characterized as  “pure”?  

A. Up to 3500

B. Up to 5500

C. Up to 2500

D. Up to 7500

E. Up to 4500*  

 

 176 A 24-year-old female patient complains of acute pain in the lower abdomen that turned up after a physical stress. She presents  with nausea, vomiting, dry mouth and body temperature 36,6oC. She has a right ovarian cyst in history. Bimanual examination  reveals that uterus is dense, painless, of normal size. The left fornix is deep, uterine appendages aren’t palpable, the right fornix is  contracted. There is a painful formation on the right of uterus. It’s round, elastic and mobile. It is 7х8 cm large. In blood:  leukocytosis with the left shit. What is the most likely diagnosis?  

A. Ovarian cyst with pedicle torsion*  

B. Acute metritis  

C. Extrauterine pregnancy  

D. Subserous fibromyoma of uterus  

E. Right-sided pyosalpinx  

 

 177 A 10-year-old child has been folowed-up for the dilated cardiomyopathy. The child presents with dyspnea, cardialgia. There are  dense, nonmobile edemata on the lower extremities and sacrum. Ps- 120/min. The cardiac borders are extended transversely.  Heart sounds are muffled, there is blowing systolic murmur at the apex and over the xiphoid process. Liver is 3 cm enlarged, urine  output is reduced. The blood total protein – 58.6 g/l. In urine: protein – 0,025 g/l, WBCs – 2-4 in the field of vision, RBCs – 2-3 in the  field of vision. What is the main mechanism of edema syndrome development:

A. Venous congestion of lesser circulation  

B. Venous congestion of greater circulation*  

C. Hypoproteinemia  

D. Peripheral circulation disorder  

E. Secondary nephropathy development  

 

 178 A 25-year-old victim of a road accient complains of chest pain, dyspnea. Objectively: the patient is in a grave condition, Ps-  120/min, AP- 90/70 mm Hg.There is pathological mobility of fragments of III-V ribs on the right. Percussion reveals a box sound  over the right lung, breathing sounds cannot be auscultated on the right. What examination should be administered in the first  place?  

A. Pleural puncture  

B. X-ray of chest organs*  

C. USI of chest organs  

D. Bronchoscopy  

E. Thoracoscopy  

 

 179 A 19-year-old student has been hospitalized on an emergency basis because of severe dyspnea, pain in the left side of chest.  The patient got sick 3 days ago. Objectively: body temperature – 38,8oC. BH – 42/min., hypopnoe. There is dullness of  percussion sound on the right of the scapula middle, breathing sounds cannot be auscultated. The left border of heart is displaced  outwards by 3 cm. Embryocardia is preent, HR – 110/min. The right hypochondrium is painful on palpation. What are the immediate  treatment measures in this situation? 

A. Urgent puncture of pleural cavity*

B. Administartion of antibiotics of penicillin group

C. Injection of cardiac glycosides

D. Injection of lasix

E. Transfer of the patient to the thoracic surgery department

 

 180 A 30-year-old man was injured in a fire and got thermic burns of III-A and III-B degree that affected 20% of the total skin. AP –  110/70 mm Hg, heart rate -120/min. What transfusion means should be used for blind infusion before transportation?  

A. Albumin  

B. 10% glucose solution  

C. Saline solutions*  

D. Fresh frozen plasma  

E. Polyglycine  

 

 181 After treating a field with pesticides a machine operator presents with great weakness, headache, nausea, vomiting, diarrhea,  visual impairment, watery eyes. Objectively: the patient is excited, hypersalivation, hyperhidrosis, muscle fibrillation of tongue and  eyelids are oberved. Pupils are narrowed, there is tachycardia, lung auscultation reveals moist small and medium bubbling rales. In  blood: changed level of cholinesterase activity. What is the most likely diagnosis?  

A. Intoxication with organomercurial pesticides  

B. Intoxication with carbamic acid derivatives  

C. Intoxication with arsenic-containing pesticides  

D. Intoxication with organochlorine pesticides  

E. Intoxication with organophosphorous pesticides*  

 

 

 182 A 3-year-old child has been diagnosed with type I diabetes mellitus, hyperosmolar coma. The laboratory confirmed the diagnosis.  Which laboratory findings are characteristic for such condition?

A. Hyperglycemia and high indicators of acid-base balance  

B. High hyperglycemia without ketonemia*  

C. Hyperglycemia and ketonemia  

D. Hyperglycemia and ketonuria

E. Hyperglycemia and glucosuria  

 

 183 A 44-year-old patient complains about difficult urination, sensation of incomplete urinary bladder emptying. Sonographic  examination of the urinary bladder near the urethra entrance revealed an oval well-defined hyperechogenic formation 2×3 cm  large that was changing its position during the examination. What conclusion can be made?

A. Prostate adenoma  

B. Urinary bladder polyp  

C. Primary ureter tumour  

D. Malignant tumour of the urinary bladder

E. Concrement*  

 

 184 A 54-year-old female patient consulted a doctor about bloody discharges from the genital tracts after 2 years of amenorrhea. USI  and bimanual examination revealed no genital pathology. What is the tactics of choice?  

A. Contracting drugs  

B. Estrogenic haemostasia  

C. Styptic drugs  

D. Fractional biopsy of lining of uterus and uterine mucous membranes*  

E. Hysterectomy  

 

 185 A department chief of an in-patient hospital is going to inspect resident doctors as to observation of medical-technological  standards of patient service. What documentation should be checked for this purpose?

A. Treatment sheets  

B. Registry of operative interventions  

C. Statistic cards of discharged patients

D. Annual report of a patient care institution  

E. Health cards of in-patients*  

 

 186 A 5-year-old child developed an acute disease starting from body temperature rise up to 38,5oC, running nose, cough and  conjunctivitis. On the 4th day the child presented with maculo-papular rash on face. Body temparature rose again up to  39,2oC. Over the next few days the rash spread over the whole body and extremities. Mucous membrane of palate was  hyperemic, there was whitish deposition on cheek mucous membrane next to molars. What is your provisional diagnosis?  

A. Rubella  

B. Acute viral respiratory infection  

C. Measles*  

D. Yersinia  

E. Enterovirus diseases  

 

 187 A 56-year-old scientist experiences constricting retrosternal pain several times a day while walking for 100-150 m. The pain lasts  for up to 10 minutes and can be relieved by nitroglycerine. Objectively: the patient is overweight, heart borders exhibit no  abnormalities, heart sounds are rhythmic, Ps- 78 bpm, AP- 130/80 mm Hg. ECG contains low amplitude of T wave in V4-5.  What disease might be suspected?

A. Stable FC I stenocardia  

B. Instable stenocardia  

C. Stable FC IV stenocardia  

D. Stable FC III stenocardia*  

E. Stable FC II stenocardia  

 

 188 In the current year general practitioners of the municipal polyclinic have referred 11 patients with coronary artery disease to the  in-patient hospital. In 3 cases the diagnosis wasn’t confirmed. What managerial decision shoud be made in such case?

A. Analysis of each case of diagnostic divergence*  

B. Analysis of material and technical basisof the polyclinic  

C. Analysis of medical check-up quality  

D. Analysis of doctors’ skill level  

E. Analysis of diagnostic examination quality  

 

 189 A 35-year-old patient has been admitted to a hospital for pain in the left sternoclavicular and knee joints, lumbar area. The disease  has an acute character and is accompanied by fever up to 38oC. Objectively: the left sternoclavicular and knee joints are  swollen and painful. In blood: WBCs – 9,5*109/l, ESR – 40 mm/h, CRP – 1,5 millimole/l, fibrinogen – 4,8 g/l, uric acid – 0,28  millimole/l. Examination of the urethra scrapings reveals chlamydia. What is the most likely diagnosis?  

A. Rheumatic arthritis  

B. Gout  

C. Reiter’s syndrome*  

D. Rheumatoid arthritis  

E. Bechterew’s disease  

 

 190 A 32-year-old patient lives in an area endemic for echinococcosis. In the last 6 months he reports of pain in the right subcostal  area, fever. He is suspected to have liver echinococcosis. What study would be the most informative in this case?  

A. Angiography  

B. USI*

C. Biochemical laboratory examination  

D. Liver scanning  

E. Survey radiography of abdominal cavity  

 

 191 A 48-year-old patient got a job-related injury of a hypodermic varicose vein on his shin that was accompanied by the intensive  phleborrhagia. Choose the optimal variant of first aid:  

A. Pressure bandage and limb strapping*  

B. Maximal limb flexion in knee joint  

C. Occlusion of femoral artery in a typical place  

D. Application of Esmarch’s tourniquet beneath the injury  

E. Application of Esmarch’s tourniquet above the injury  

 

 192 As a result of prolonged exposure to the sun a 20-year-old patient has developed low-grade fever, pain and swelling in the knee  and ankle joints, erythema on her face and nose bridge, leukopenia and accelerated ESR. She has been provisionally diagnosed  with systemic lupus erythematosus. What pathognomonic laboratory data may confirm this diagnosis?  

A. Lymphocytosis  

B. Antinuclear factor*  

C. Accelerated ESR  

D. C-reactive protein  

E. Anaemia  

 

 193 A full-term infant is 3 days old. On the different parts of skin there are erythemas, erosive spots, cracks, areas of epidermis  peeling. The infant has scalded skin syndrome. Nikolsky’s symptom is positive. General condition of the infant is grave. Anxiety,  hyperesthesia, febrile temperature are evident. What is the most probable diagnosis?  

A. Phlegmon of newborn

B. Impetigo neonatorum

C. Finger’s pseudofurunculosis

D. Mycotic erythema

E. Exfoliative dermatitis*

 

 194 A 40-year-old female patient has a history of rheumatism. She complains about acute pain in her left eye, especially at night, vision  impairment, photophobia, lacrimation. The patient cannot suggest any reasons for the disease. Objectively: weak pericorneal  injection, flattening of iris relief, iris discoloration. What is the most likely diagnosis?  

A. Iridocyclitis*  

B. Iritis  

C. Keratitis  

D. Choroiditis  

E. Acute attack of glaucoma  

 

 195 A patient with bilateral hydrothorax has repeatedly undergone pleural puncture on both sides. After a regular puncture the  patient’s condition has become worse: he presents with fever, chest pain. The next day, the attending physician performing  pleural puncture revealed some pus on the right. What is the mechanism of acute right-sided empyema development?

A. Aerial  

B. Lymphogenous  

C. Hematogenous  

D. Implantation  

E. Contact-and-aspiration*  

 

 196 A patient is on the sick leave for 4 months continuously from the date of injury. The treatment is going to last for 1-2 months. Who  has the right to extend the duration of medical certificate for this patient?

A. Medical advisory commission after medico-social expert commission examination*  

B. Medical advisory commission after inpatient treatment  

C. District doctor by agreement with a department chief  

D. Medical superintendent

E. Medico-social expert commission  

 

 197 A 10-year-old boy underwent treatment in cardiological department for rheumatism, I acute attack of rheumatic fever, active  phase, II degree. The patient was discharged in satisfactory condition. Which drug should be chosen for prevention of rheumatism  recurrence?  

A. Erythromycin  

B. Ampicillin  

C. Oxacillin  

D. Bicillinum-5*

E. Bicillinum-1  

 

 198 Educational rooms are illuminated with various lighting fittings. What type of lighting fittings is the most appropriate in respect of  hygienic norms?  

A. Direct light fittings  

B. Semi-reflected light fittings  

C. Combined light fittings  

D. Ambient light fittings  

E. Indirect light fittings*  

 

 199 A 54-year-old patient has an over 20-year history of femoral osteomyelitis. Over the last month she has developed progressing  edemata of the lower extremities. Urine test reveals: proteinuria at the rate of 6,6 g/l; in blood: dysproteinemia in form of  hypoalbuminemia, increase in a2– and g-globulin rate, ESR – 50 mm/h. What is the most likely diagnosis?

A. Systemic lupus erythematosus  

B. Acute glomerulonephritis  

C. Secondary renal amyloidosis*  

D. Chronic glomerulonephritis  

E. Myelomatosis  

 

 200 A 36-year-old patient complains of skin rash that appeared a week ago and doesn’t cause any subjective problems. Objectively:  palm and sole skin is covered with multiple lenticular disseminated papules not raised above the skin level. The papules are  reddish, dense on palpation and covered with keratinous squamae. What is the provisional diagnosis?

A. Palm and sole callosity  

B. Palmoplanar psoriasis  

C. Secondary syphilis *

D. Palmoplanar rubrophytosis  

E. Verrucosis  

 

 

Diabetes Mellitus

Posted: March 11, 2013 in Endocrinology
Tags: ,

– The term diabetes mellitus describes a metabolic disorder of multiple aetiology characterized by chronic hyperglycaemia with disturbances of carbohydrate, fat and protein metabolism resulting from defects in insulin secretion, insulin action, or both.

symptoms-of-diabetes

  1. Type 1 DM results from the body’s failure to produce insulin, and currently requires the person to inject insulin or wear an insulin pump. This form was previously referred to as “insulin-dependent diabetes mellitus” (IDDM) or “juvenile diabetes”.
  2. Type 2 DM results from insulin resistance, a condition in which cells fail to use insulin properly, sometimes combined with an absolute insulin deficiency. This form was previously referred to as non insulin-dependent diabetes mellitus (NIDDM) or “adult-onset diabetes”.
  3. Gestational diabetes (or gestational diabetes mellitus, GDM) is a condition in which women without previously diagnosed diabetes exhibit high blood glucose levels during pregnancy (especially during their third trimester).
  4. Misc Types of DM : Other specific types of diabetes result from specific genetic conditions (such as maturity-onset diabetes of youth), surgery, drugs, malnutrition, infections, and other illnesses.

Untitled-3Untitled-4

Untitled-5

Complications Diabetes

diabetes-complicationsdiabetes_map1

Krok 2. Medicine – 2009

Posted: January 28, 2013 in Krok II
Tags: , , ,

Krok 2. Medicine – 2009

 1 A 27 year old man complains of pains in epigastrium which are relieved by food intake. EGDFS shows antral erosive gastritis,  biopsy of antral mucous presents Hеlicobacter Pylori. Diagnosis is:   

A. Gastritis of type A   

B. Menetrier’s gastritis    

C. Reflux-gastritis   

D. Gastritis of type B*   

E. Rigid antral gastritis 

 

NB! Type A gastritis primarily affects the body/fundus of the stomach, and is more common with pernicious anemia.It is also sometimes called auto-immune gastritis. In this type of gastritis, the parietal cells which are responsible for the production of acid are attacked, which can in advanced cases lead to complete deterioration of the stomach lining. A particular form of anaemia can also result due to an interruption in the absorption of vitamin B12.

Type B gastritis (most common overall) primarily affects the antrum.This gastritis accounts for 80% of all gastritis cases. It is primarily caused by pathogens like bacteria, viruses or alcohol. In 90% of cases of type B gastritis, there is one primary cause: theHelicobacter pylori bacteria.

 

Helicobacter pylori.

Helicobacter pylori.

 

 2 Examination of placenta revealed a defect. An obstetrician performed manual investigation of uterine cavity, uterine massage.  Prophylaxis of endometritis in the postpartum period should involve following actions:    

A. Instrumental revision of uterine cavity   

B. Antibacterial therapy*   

C. Haemostatic therapy   

D. Contracting agents   

E. Intrauterine instillation of dioxine 

Endometriosis

Endometriosis

Endometriosis

Endometriosis

   

 

 3 A 67 year old female patient complains about edemata of face and legs, pain in the lumbar area that is getting worse at moving;  great weakness, sometimes nasal haemorrhages, rise of body temperature up to 38,4oC. Objectively: painfulness of vertebral  column and ribs on palpation. Laboratorial study revealed daily proteinuria of 4,2 g, ESR- 52 mm/h. What changes of laboratory  indices are to be expected?    

A. Haemoglobin – 165 g/l   

B. g-globulins – 14%   

C. Albumins – 65%   

D. Leukocytes – 15,3 g/l    

E. Whole protein of blood serum – 101 g/l*    

 NB! Serum total protein, also called plasma total protein or total protein, is a Biochemical test for measuring the total amount of protein in blood plasma or serum. Protein in the plasma is made up of albumin and globulin. The globulin in turn is made up of α1, α2, β, and γ globulins. The reference range for total protein is 60-85g/L. (It is also sometimes reported as “6-8.5g/dl”.).

  • Concentrations below the reference range usually reflect low albumin concentration, for instance in liver disease or acute infection.
  • Concentrations above the reference range are found in paraproteinaemia, Hodgkin’s lymphoma or leukaemia.

 

 

4 A 43 year old female patient was delivered to the hospital in grave condition. She suffers from Addison’s disease. The patient had  been regularly taking prednisolone but a week before she stopped taking this drug. Objectively: sopor, skin and visible mucous  membranes are pigmented, skin and muscle turgor is lowered. Heart sounds are muffled, rapid. AP- 60/40 mm Hg, heart rate –  96/min. In blood: Na- 120 millimole/l, K- 5,8 micromole/l. Development of this complication is primarily caused by the deficit of the  following hormone:    

A. Adrostendion   

B. Noradrenaline   

C. Adrenaline    

D. Cortisol*    

E. Corticotropin (ACTH)

 NB! Addison’s disease (also chronic adrenal insufficiency, hypocortisolism, and hypoadrenalism) is a rare, chronic endocrine disorder in which the adrenal glands do not produce sufficient steroid hormones (glucocorticoids and often mineralocorticoids). It is characterised by a number of relatively nonspecific symptoms, such as abdominal pain and weakness, but under certain circumstances, these may progress to Addisonian crisis, a severe illness which may include very low blood pressure and coma.In most cases Addison’s disease is an idiopathic autoimmune disease.


 5 A 52 year old patient was admitted to a hospital because of high hemorrhagic diathesis of mucous membranes, massive skin  haemorrhages in form of ecchymoses and spots, nasal and stomachal haemorrhages. After clinical examinations her illness was  diagnosed as thrombocytopenic purpura. What is the most probable cause of this disease?    

A. Iron deficit in blood serum, bone marrow and depot   

B. Deficit of the VIII factor of blood coagulation   

C. Generation of antithrombocytic antibodies*   

D. Disturbed hemostasis   

E. Inherited insufficiency of plasm factors of blood coagulation   

NB!  Bleeding diathesis (or bleeding tendency or predisposition or ha(e)morrhagic diathesis) is an unusual susceptibility to bleeding (hemorrhage) mostly due to hypocoagulability.

 

 6 A 37 year old patient applied to a local therapeutist. As a result of exacerbation of chronic obstructive bronchitis the patient had  been temporarily disabled for 117 days within 1 year. What tactics will be legally correct?    

A. The patient should be referred to the medicosocial expertise*   

B. The therapeutist should extend a medical certificate   

C. The patient shoul be referred to the sanatorium-and-spa treatment    

D. The therapeutist should issue a new medical certificate    

E. The patient should be referred to the medical consultation comission for extension of medical certificate 

   

 

 

 7 A patient applied to the traumatology cenre and complained about a trauma of the lower third of the volar forearm surface caused  by cut on a piece of galss. Objectively: flexion of the IV and V fingers is impaired, sensitivity of the interior dorsal and palmar  surface of hand as well as of the IV finger is reduced. What nerve is damaged?       

A. Musculoskeletal   

B. Radial   

C. Median   

D. Ulnar*   

E. Axillary    

 

 8 A parturient woman is 27 year old, it was her second labour, delivery was at term, normal course. On the 3rd day of postpartum  period body temperature is 36,8oC, Ps – 72/min, AP – 120/80 mm Hg. Mammary glands are moderately swollen, nipples are  clean. Abdomen is soft and painless. Fundus of uterus is 3 fingers below the umbilicus. Lochia are bloody, moderate. What is the  most probable diagnosis?   

A. Postpartum metroendometritis   

B. Lactostasis   

C. Remnants of placental tissue after labour   

D. Physiological course of postpartum period*   

E. Subinvolution of uterus   

NB! In the field of the obstetrics, lochia is the vaginal discharge for the first fortnight of puerperium (after birth), containing blood, mucus, and placental tissue. Lochia discharge typically continues for 4 to 6 weeks after childbirth.

 

 9 A 43 year old patient was admitted to the infectious diseases hospital with high body temperature and intense headache. The  iIlness has lasted for 2 days. Examination revealed a carbuncle on his forearm. The area around it was apparently edematic and  slightly painful. Regional lymphadenitis and hepatolienal syndrome were also present. It is known from the anamnesis that the  patient works at a cattle-breeding farm. What disease should be suspected in the first place?    

A. Skin cancer   

B. Erysipeloid   

C. Anthrax*   

D. Erysipelas   

E. Eczema   

NB!  Anthrax (Woolsorter’s disease; Ragpicker’s disease; Cutaneous anthrax; Gastrointestinal anthrax)

Anthrax is an infectious disease due to a type of bacteria called Bacillus anthracis. Infection in humans most often involves the skin, gastrointestinal tract, or lungs.

Untitled-3

fig4

19057

 

 

10 A 28 year old woman has bursting pain in the lower abdomen during menstruation; chocolate-like discharges from vagina. It is  known from the anamnesis that the patient suffers from chronic adnexitis. Bimanual examination revealed a tumour-like formation  of heterogenous consistency 7х7 cm large to the left from the uterus. The formation is restrictedly movable, painful when moved.  What is the most probable diagnosis?    

A. Fibromatous node   

B. Follicular cyst of the left ovary    

C. Endometrioid cyst of the left ovary*   

D. Tumour of sigmoid colon   

E. Exacerbation of chronic adnexitis   

sf_05endometriosis

NB! Unfortunately, increasingly more women, especially young women, have this pathology. Adnexitis is a inflammatory diseases that is generally affecting the fallopian tubes and ovaries, but can also affect the uterus. It is called metro-adnexitis (the uterus is affected). Inflammatory lesions occur most commonly in the fallopian tubes and ovaries and uterus are less damaged.

Statistical studies show that one in four women suffers from adnexistis and in 60% of cases the infection becomes chronic.

 

 11 A 68 year old patient complains about acute pain in his right foot, toe edema and darkening of skin of the IV toe. He has been  suffering from diabetes mellitus for 15 years, doesn’t receive regular treatment. What complication of diabetes mellitus is it?    

A. Haematoma   

B. Gangrene of the IV toe on the right foot*  

C. Panaritium   

D. Erysipelas   

E. Fracture of the IV toe on the right foot   

Untitled-4

 

 12 A 52 year old male patient complains about attacks of asphyxia, pain in his left side during respiration. These manifestations turned  up all of a sudden. It is known from his anamnesis that he had been treated for thrombophlebitis of the right leg for the last month.  In the admission ward the patient suddenly lost consciousness, there was a sudden attack of asphyxia and pain in his left side.  Objectively: heart rate – 102/min, respiratory rate – 28/min, AP- 90/70 mm Hg. Auscultation revealed diastolic shock above the  pulmonary artery, gallop rhythm, small bubbling rales above the lungs under the scapula on the right, pleural friction rub. What  examination method will be the most informative for a diagnosis?     

A. Echocardioscopy   

B. Coagulogram   

C. ECG   

D. Study of external respiration function   

E. Angiography of pulmonary vessels*   

 

 13 A 10 year old boy suffers from chronic viral hepatitis type B with maximal activity. What laboratory test can give the most precise  characteristic of cytolysis degree?    

A. Weltman’s coagulation test     

B. Test for whole protein   

C. Takata-Ara test   

D. Transaminase test*   

E. Prothrombin test   

Untitled-5

 

 14 An infant is full-term. Delivery was pathological, with breech presentation. Examination of the infant revealed limited abduction of  the right leg to 50o, positive “clicking” symptom on the right, asymmetric inguinal folds. What is the most probable diagnosis?    

A. Inborn dislocation of the right hip*   

B. Inborn dislocation of both hips   

C. Varus deformity of both femoral necks   

D. Right hip dysplasia   

E. Fracture of both femoral necks   

 

 15 On the 6th day of life a child got multiple vesicles filled with seropurulent fluid in the region of occiput, neck and buttocks. General  condition of the child is normal. What disease should be suspected?    

A. Epidermolysis bullosa   

B. Miliaria   

C. Impetigo neonatorum   

D. Vesiculopustulosis*   

E. Impetigo   

NB! A lesion of the epidermis consisting of a vesicle containing inflammatory cells.

bc6d155bed6e4698ab54449c338a813f1

 

 16 Estimation of physical development of a child involved dynamometry and estimation of body weight and length, annual gain in body  length, chest circumference, number of permanent teeth, secondary sexual characters, lung vital capacity. Which of the  mentioned indices relates to the physiometric ones?    

A. Body length and weight, chest circumference    

B. Lung vital capacity, dynamometry*    

C. Annual gain in body length   

D. Secondary sexual characters    

E. Number of permanent teeth   

 13-1

 17 A 37 year old male patient was admitted to the resuscitation department because of attacks of tonoclonic spasms repeating every  half an hour. Between the attacks the patient remains unconscious. AP is 120/90 mm Hg, Ps- 100 bpm. A day before the patient  was at wedding and consumed alcohol. 5 years ago he had a closed craniocerebral trauma and brain contusion that later caused  single convulsive attacks accompanied by loss of consciousness, but the patient didn’t undergo antiepileptic treatment. What drug  should be injected for emergency aid?    

A. Sodium oxybutyrate    

B. Diazepam*   

C. Aminazine   

D. Sodium thiopental    

E. Magnesium sulfate   

NB! What is diazepam? Diazepam is a benzodiazepine (ben-zoe-dye-AZE-eh-peens). It affects chemicals in the brain that may become unbalanced and cause anxiety.

Diazepam is used to treat anxiety disorders, alcohol withdrawal symptoms, or muscle spasms. Diazepam is sometimes used with other medications to treat seizures.

Generic Name: diazepam
Brand Name: Valium

valium-24190_1

 

 18 A patient has got pain in the axillary area, rise of temperature developed 10 hours ago. On examination: shaky gait is evident, the  tongue is coated with white deposit. The pulse is frequent. The painful lymphatic nodes are revealed in the axillary area. The skin  over the lymph nodes is erythematous and glistering. What is the most probable diagnosis?     

A. Tularemia    

B. Acute purulent lymphadenitis    

C. Bubonic plague*    

D. Lymphogranulomatosis    

E. Anthrax    

Symptoms_of_bubonic_plague

bubound

 

 19 A 14 year old child suffers from vegetovascular dystonia of pubertal period. He has got sympathoadrenal atack. What medicine  should be used for attack reduction?    

A. Corglicone    

B. Amysyl    

C. Aminophylline   

D. Obsidan*    

E. No-shpa

NB! Obsidan 40 mg is a beta-blocker. Obsidan 40 mg is used for high blood pressure (hypertension) diseases of the coronary arteries (coronary heart disease), heart rhythm disorders with increased stroke rate (tachycardia arrhythmia)  functional heart disorders (hyperkinetic heart syndrome) “tremors” unknown cause (essential tremor) preventive treatment of migraine (migraine prophylaxis) treating symptoms of an overactive thyroid (hyperthyroidism), or as an addition to the effect of specific interventions.   

obsidan-40-mg-33797_3 

 

 20 1,5 hour after start of gullet bougienage a 48 year old patient suffering from corrosive stricture felt acute abdominal pain.  Previously he had been suffering from duodenal ulcer. Examination revealed that abdomen was very tense and painful; Ps- 110  bpm, painful sialophagia, skin pallor. What is the most probable diagnosis?     

A. Thrombosis of mesenteric vessels   

B. Perforation of abdominal part of esophagus*   

C. Perforation of duodenal ulcer   

D. Acute myocardium infarction   

E. Strangulation of diaphragmal hernia   

 gerd-and-barrett-s-esophagus_1_1

 21 A 46 year old patient is to be prepared to the operation on account of stomach cancer. Preoperative preparation involves infusion  therapy. It was injected up to 3 l of solutions into his right lunar vein. On the next day he got tensive pain in the region of his right  shoulder. Examination of interior brachial surface revealed an oblong area of hyperemia, skin edema and painful cord. What  complication is it?    

A. Acute thrombophlebitis*    

B. Vein puncture and edema of paravenous cellular tissue   

C. Acute lymphangitis   

D. Phlegmon of paravenous cellular tissue   

E. Necrosis of paravenous cellular tissue   

2008122820481993517

 

 22 A 2 month old full-term child was born with weight 3500 g and was on the mixed feeding. Current weight is 4900 g. Evaluate the  current weight of the child:    

A. Paratrophy of the I grade    

B. 150 g less than necessary    

C. Hypotrophy of the I grade    

D. Corresponding to the age*   

E. Hypotrophy of the II grade    

 

 

 

 23 A child was born with body weight 3250 g and body length 52 cm. At the age of 1,5 month the actual weight is sufficient (4350 g),  psychophysical development corresponds with the age. The child is breast-fed, occasionally there are regurgitations. What is the  cause of regurgitations?     

A. Acute gastroenteritis   

B. Aerophagia*   

C. Pylorospasm   

D. Pylorostenosis   

E. Esophageal atresia   

 

 24 A 58 year old female patient complains about periodical headache, dizziness and ear noise. She has been suffering from diabetes  mellitus for 15 years. Objectively: heart sounds are rhythmic, heart rate is 76/min, there is diastolic shock above aorta, AP is  180/110 mm Hg. In urine: OD- 1,014. Daily loss of protein with urine is 1,5 g. What drug should be chosen for treatment of arterial  hypertension?    

A. Calcium channel antagonist    

B. a-blocker   

C. Thiazide diuretic   

D. b-blocker   

E. Ihibitor of angiotensin converting enzyme*   

 

 25 A 9 year old boy had acute respiratory viral infection. After it there appeared polydipsia, polyuria, weakness, nausea. Examination  revealed the following symptoms: mental confusion, dry skin, soft eyeballs, Kussmaul’s respiration, acetone smell from the mouth,  muffled heart sounds, soft and painless abdomen. Blood sugar was 19 millimole/l. What acute condition is it?    

A. Ketoacidotic coma*   

B. Hepatic coma   

C. Cerebral coma   

D. Acute renal insufficiency   

E. Hyperosmolar coma   

safe_image

11

 

 

 

 26 Which of the following symptoms would occur only if a total-body acute radiation exposure exceeded 5.000 rad (50 Gy)   

A. Diarrhea   

B. Bleeding gums   

C. Nausea and vomiting    

D. Hallucinations and impairment of vision*   

E. Epilation (hair loss)   

 

 27 A 42 year old man works in a boiler room. He complains about girdle headache and recurring vomiting. There was also short-term  consciousness loss. Objectively: increase of tendon reflexes, spontaneous myofibrillations. AP is 150/80 mm Hg, Ps- 104 bpm.  Visible mucous membranes and cutaneous surfaces have crimson colouring. What is the most probable diagnosis?    

A. Poisoning with anilin colouring agents   

B. Poisoning with hydrocyanic acid   

C. Poisoning with methane   

D. Poisoning with carbon monooxide*   

E. Poisoning with benzine   

carbon-monoxide-gas-safety

 

 28 A 32 year old patient complains about cardiac irregularities, dizziness, dyspnea at physical stress. He has never suffered from  this before. Objectively: Ps- 74 bpm, rhythmic. AP- 130/80 mm Hg. Auscultation revealed systolic murmur above aorta, the first  heart sound was normal. ECG showed hypertrophy of the left ventricle, signs of repolarization disturbance in the I, V5 and  V6 leads. Echocardiogram revealed that interventricular septum was 2 cm. What is the most probable diagnosis?    

A. Aortic stenosis   

B. Myocardium infarction   

C. Essential hypertension   

D. Coarctation of aorta   

E. Hypertrophic cardiomyopathy*   

Hypertrophic_Cardiomyopathy_Dia

 

 

 

 

 29 Examination of a 22 year old man suffering from polyarthralgia and high fever revealed right-sided exudative pleuritis. X-ray  picture showed a homogenous shadow below the IV rib on the right. In the II segment there were single dense focal shadows.  Mantoux test with 2 TU resulted in formation of a papula 16 mm largeE. Pleural liquid has increased protein concentration, Rivalta’s  reaction is positive, there was also increased number of leukocytes with prevailing lymphocytes. What is the most probable  etiology of pleuritis?    

A. Viral   

B. Tuberculous*   

C. Staphylococcal   

D. Autoimmune    

E. Cancerous  

NB! Rivalta test is used in order to differentiate a transudate from an exudate .Using a pH 4.0 acetic acid solution, 8 types of proteins were identified in Rivalta reaction-positive turbid precipitates: C-reactive protein (CRP), Alpha 1-antitrypsin (alpha1-AT), Orosomucoid ((Alpha-1-acid glycoprotein or AGP)), haptoglobin (Hp), transferrin (Tf), ceruloplasmin (Cp), fibrinogen (Fg), and hemopexin (Hpx). Since those are Acute-phase proteins, a positive Rivalta’s test may be suggestive of inflammation.

 

 30 A woman consulted a therapeutist about fatigability, significant weight loss, weakness, loss of appetite. She has had amenorrhea  for 8 months. A year ago she born a full-term child. Haemorrhage during labour made up 2 l. She got blood and blood substitute  transfusions. What is the most probable diagnosis?    

A. Homological blood syndrome   

B. Sheehan’s syndrome*   

C. Stein-Leventhal syndrome    

D. Shereshevsky-Turner’s syndrome    

E. Vegetovascular dystonia   

 NB! Sheehan syndrome (Alternate Names : Postpartum hypopituitarism, Postpartum pituitary insufficiency, Hypopituitarism Syndrome)

 

 31 A 36 year old patient was diagnosed with right-sided pneumothorax. What method of treatment is indicated to the patient?    

A. Antiinflammation therapy   

B. Thoracotomy   

C. Symptomatic therapy   

D. Pleural puncture   

E. Surgical treatment: drainage of the pleural cavity*   

 h9991550_001

 

 

 

 32 A patient consulted a venereologist about painful urination, reddening of the external opening of urethra, profuse purulent  discharges from the urethra. He considers himself to be ill for 3 days. He also associates the disease with a casual sexual  contact that took place for about a week ago. If provisional diagnosis “acute gonorrheal urethritis” will be confirmed, then  bacteriological study of urethral discharges will reveal:    

A. Mycoplasma   

B. Gram-positive diplococci   

C. Gram-negative diplococci*    

D. Proteus vulgaris   

E. Spirochaete   

 

 33 A 23 year old female patient complains about periodical chill and body temperature rise up to 40oC, sense of heat taking turns  with profuse sweating. The patient has had already 3 attacks that came once in two days and lasted 12 hours. She has lived in  Africa for the last 2 months. Liver and spleen are enlarged. In blood: erythrocytes – 2,5*1012/l. What is the most probable  diagnosis?     

A. Haemolytic anaemia   

B. Malaria*   

C. Sepsis   

D. Leptospirosis   

E. Spotted fever   

 

 34 A 26 year old woman had the second labour within the last 2 years with oxytocin application. The child’s weight is 4080 g. After  the placent birth there were massive bleeding, signs of hemorrhagic shock. Despite the injection of contractive agents, good  contraction of the uterus and absence of any cervical and vaginal disorders, the bleeding proceeds. Choose the most probable  cause of bleeding:     

A. Hypotonia of the uterus    

B. Injury of cervix of the uterus     

C. Delay of the part of placenta   

D. Hysterorrhexis   

E. Atony of the uterus*   

 

 

 35 A 44 year old man has been working in coke industry for 16 years. Dust concentration at his workplace is 5-10 times more than  maximum permissible concentration. Roentgenography of lungs revealed changes that are typical for pneumoconiosis. What is the  most probable type of pneumoconiosis in this case?     

A. Anthracosilicosis   

B. Asbestosis   

C. Siderosis   

D. Silicatosis    

E. Anthracosis*   

 

 36 It is suspected that a 34 year old patient has an abscess of Douglas pouches. What diagnostic method is to be chosen?    

A. Digital examination of rectum*   

B. Rectoromanoscopy    

C. Laparoscopy    

D. R-scopy of abdominal cavity    

E. Percussion and auscultation of stomach   

 

 37 Female 45 year old patient was admitted to the traumatological ward with the closed fracture of the medial malleolus with its  displacement up to 3 mm. The foot is to be fixed with a plaster cast in the following position:    

A. In position of supination  

B. At right angle with varus positioning of the foot*   

C. In position of dorsal flexion of foot   

D. In position of planter flexion of foot   

E. In position of pronation   

 

 38 In the 43rd week of gestation a long, thin infant was delivered. He is apneic, limp, pale, and covered with “pea soup” amniotic fluid.  The first step in the resuscitation of this infant at delivery should be:   

A. Artificial ventilation with bag and mask    

B. Artificial ventilation with endotracheal tube   

C. Catheterization of the umbilical vein   

D. Suction of the trachea under direct vision*   

E. Administration of 100% oxygen by mask    

 

 39 The average body lenth of newborn boys is 50,9 cm at a sigma 1,66; and average mass – 3432 at a sigma 5,00. What criterion is  necessary in order to compare degree of variability of these signs?    

A. Limit   

B. Sigma    

C. Amplitude   

D. Coefficient of association    

E. Coefficient of variation*   

 

 40 A 27 year old patient suffers from haemophilia. He was admitted to the hospital with melena and skin pallor. Objectively: Ps- 110  bpm, AP- 100/60 mm Hg. In blood: Hb- 80 g/l, erythrocytes – 2,8*1012/l. What medication should be administered in the first  place?    

A. Cryoprecipitate*   

B. Epsilon-aminocapronic acid   

C. Stored blood   

D. Packed red blood cells   

E. Dicinone   

 

 41 A 4 month old child fell seriously ill: body temperature rose up to 38,5oC, the child became inert and had a single vomiting. 10  hours later there appeared rash over the buttocks and lower limbs in form of petechiae, spots and papules. Some haemorrhagic  elements have necrosis in the centr E. What is the most probable disease?    

A. Scarlet fever    

B. Meningococcemia*   

C. Haemorrhagic vasculitis   

D. Influenza   

E. Rubella   

 

 

 42 A 39 year old patient complained about morning headache, appetite loss, nausea, morning vomiting, periodic nasal haemorrhages.  The patient had acute glomerulonephritis at the age of 15. Examination revealed rise of arterial pressure up to 220/130 mm Hg, skin  haemorrhages on his arms and legs, pallor of skin and mucous membranes. What biochemical index  has the greatest diagnostic  importance in this case?   

A. Fibrinogen   

B. Uric acid   

C. Blood bilirubin   

D. Blood creatinine*   

E. Blood sodium   

 

 43 A 12 year old child has the ulcer disease of stomach. What is the etiology of this disease?   

A. Influenza   

B. Salmonella   

C. Helicobacter pylory   

D. Lambliosis   

E. Intestinal bacillus*   

 

 44 A 10 year old boy complains about pain in his left eye and strong photophobia after he had injured his left eye with a pencil at  school. Left eye examination: blepharospasm, ciliary and conjunctival congestion, cornea is transparent, other parts of eyeball  have no changes. Visus 0,9. Right eye is healthy, Visus 1,0. What additional method would you choose first of all?    

A. Cornea sensation-test   

B. Gonioscopia   

C. Tonometria   

D. Staining test with 1% fluorescein*   

E. X-ray examination of orbit   

 

 45 A 63 year old patient was diagnosed with purulent mediastinitis. What of the below listed diseases are NOT the cause of purulent  mediastinitis?       

A. Iatrogenic injury of the trachea   

B. Perforation of the cervical part of the oesophagus   

C. Deep neck phlegmon   

D. Perforation of the thoracic part of the oesophagus    

E. Cervical lymphadenitis*    

 

 46 A 4 year old girl was playing with her toys and suddenly she got an attack of cough, dyspne A. Objectively: respiration rate –  45/min, heart rate – 130/min. Percussion revealed dullness of percutory sound on the right in the lower parts. Auscultation  revealed diminished breath sounds with bronchial resonance on the right. X-ray pictue showed shadowing of the lower part of  lungs on the right. Blood analysis revealed no signs of inflammation. The child was diagnosed with foreign body in the right  bronchus. What complication caused such clinical presentations?     

A. Atelectasis*   

B. Pneumonia   

C. Bronchitis   

D. Emphysema   

E. Pneumothorax   

 

 47 Five days after a total hip joint replacement a 72 year old woman becomes acutely short of breath, diaphoretic and hypotensiv E.  Both lung fields are clear to auscultation and percussion, but examination of the neck reveals mild jugular venous distension with  prominent A waves. Heart sounds are normal. ECG shows sinus tachycardia with a new right bundle branch block and minor  nonspecific ST-T wave changes. The most likely diagnosis is:    

A. Aspiration   

B. Pericarditis   

C. Pulmonary thromboembolism*   

D. Acute myocardial infarction   

E. Aortic dissection   

 

 48 A 3 year old child with weight deficiency suffers from permanent moist cough. In history there are some pneumonias with  obstruction. On examination: distended chest, dullness on percussion over the lower parts of lungs. On auscultation: a great  number of different rales. Level of sweat chloride is 80 millimol/l. What is the most probable diagnosis?    

A. Bronchiectasis    

B. Bronchial asthma    

C. Pulmonary hypoplasia    

D. Recurrent bronchitis    

E. Mucoviscidosis (cystic fibrosis)*   

 

 49 A 34 year old patient complains of profuse sweating at night, skin itching, weight loss (9 kg within the last 3 months). Examination  revealed malnutrition, skin pallor. Palpation of neck and inguinal areas revealed dense elastic lymph nodes for about 1 cm in  diameter, nonmobile, non-adhering to skin. What is the most probable diagnosis?     

A. Lymphosarcoma   

B. Lymphogranulomatosis*     

C. Chronic lymphadenitis    

D. Cancer metastases    

E. Burkitt’s lymphoma    

 

 50 A 47 year old male patient got a flame burn of trunk and upper extremities and was delivered to the hospital. The patient is in grave  condition, confused mental state, with fever. AP- 80/50 mm Hg, Ps- 118 bpm. It was locally stated that the patient got III B degree  burns with total area of 20%. What medical actions should be taken?    

A. Necrotomy of burn surface, haemotransfusion   

B. Primary surgical pocessing    

C. Antibacterial and detoxicating therapy   

D. Administration of detoxicating blood substitutes   

E. Injection of narcotic analgetics and powdered blood substitutes*    

 

 51 On the third day of life an infant’s skin got icteric colouring. The child was born with body weight of 3,200 kg, body length of 52  cm. The child is activ E. There is puerile respiration above the lungs. Respiratory rate is 36/min, heart sounds are rhythmic, heart  rate is 130/min. Abdomen is soft, liver comes out from the edge of costal arch by 2 cm, spleen is not palpabl E. Feces are in form of  meconium. What is the most probable diagnosis?    

A. Physiologic jaundice*   

B. Biliary tracts atresia   

C. Neonatal sepsis   

 

D. Minkowsky-Shauffard disease   

E. Hemolytic disease of newborn   

 52 What is the maximum duration of medical certificate in case of tuberculosis?   

A. 2 months*   

B. 2 weeks   

C. Month   

D. 10 months   

E. Week   

 

 53 A 36 year old man was delivered to the surgical department an hour after a road accident. His condition is getting worse:  respiratory insufficiency is progressing, there are cardiac abnormalities. Clinical and roentgenological investigations revealed  mediastinal displacement. What process has caused this complication?   

A. Closed pneumothorax    

B. Subcutaneous emphysema    

C. Valvular pneumothorax*   

D. Open pneumothorax   

E. Mediastinitis   

 

 54 A female patient consulted a doctor about gain in weight, chill, edemata, dry skin, sleepiness, problems with concentration.  Objectively: the patient’s height is 165 cm, weight is 90 kg, gynoid body proportions, to– 35,8oC, ESR- 58/min, AP- 105/60  mm Hg. Heart sounds are weakened, bradycardia is present. Other internal organs have no changes. Thyroid gland is not  palpable. Mammary glands ooze milk droplets. Hormonal study revealed rise of TSH and prolactin concentration, reduction of  T4. What factor caused obesity?    

A. Secondary hypothyroidism    

B. Primary hypothyroidism*    

C. Prolactinoma   

D. Adiposogenital dystrophy   

E. Hypopituitarism   

 

 55 A 22 year old female patient complains about frequent and painful urination, urge to urinate at night, enuresis, pain in the  suprapubic and lumbar area. Her urine often has beer colouring. She got married a month ago. Objectively: general state is  satisfactory. Lung examination revealed vesicular respiration. Heart sounds are rhythmic, heart rate is 78/min, AP- 128/68 mm Hg.  Abdomen is soft, painful in the suprapubic area. Urine contains 12-18 erythrocytes and 12-15 bacteria within eyeshot. What is the  most probable diagnosis?    

A. Infection of superior urinary tracts – pyelonephritis   

B. Infection of inferior urinary tracts – cystitis*   

C. Gonorrhoea   

D. Urolithiasis   

E. Primary syphilis   

 

 56 On the second day after preventive vaccination a 2 year old boy got abdominal pain without clear localization, body temperature  rose up to 38oC. On the third day the child got red papular haemorrhagic eruption on the extensor surfaces of limbs and  around the joints. Knee joints were edematic and slightly painful. Examination of other organs and systems revealed no  pathological changes. What is the most probable diagnosis?    

A. Haemorrhagic vesiculitis*   

B. Thrombocytopenic purpura   

C. Urticaria   

D. DIC syndrome   

E. Meningococcemia   

 

 57 15 minutes after the second vaccination with diphteria and tetanus toxoids and pertussis vaccine a 4 month old boy manifested  symptoms of Quincke’s edema. What medication should be applied for emergency aid?    

A. Furosemide   

B. Seduxen   

C. Heparin   

D. Adrenalin   

E. Prednisolone*   

 

 

 58 A child is 1 day old. During delivery there had been problems with extraction of shoulders. Body weight is 4300,0. Right arm hangs  down along the body, hand is pronated, movement in the arm is absent. “Scarf” symptom is positive. What is the most probable  diagnosis?    

A. Hemiparesis   

B. Tetraparesis   

C. Total right-sided obstetric paralysis*   

D. Distal right-sided obstetric paralysis   

E. Proximal right-sided obstetric paralysis   

 

 59 A 62 year old patient complains of rest dyspnea, heart pains. 3 years ago he had myocardial infarction. Physical examination:  orthopnea, acrocyanosis, swollen cervical veins. Ps – 92, total heart enlargement, the liver is enlarged by 7 cm, shin edema. What  is the stage of chronic heart failure (CHF)?   

A. CHF- 3   

B. CHF- 2 B*   

C. CHF- 2 А   

D. CHF- 1   

E. CHF- 0   

 

 60 A 3 year old boy has petechial eruption. Examination revealed no other pathological changes. Thrombocyte number is  20*109g/l; haemoglobin and leukocyte concentration is normal. What is the most probable diagnosis?    

A. Disseminated intravascular coagulopathy    

B. Schonlein-Henoch disease   

C. Acute lymphoblastic leukemia    

D. Systemic lupus erythematosus    

E. Immune thrombocytopenic purpura*   

 

 61 Which of the methods of examination is the most informative in the diagnostics of a tube infertility?    

A. Pertubation   

B. Laparoscopy with chromosalpingoscopy*   

C. Bicontrast pelviography    

D. Transvaginal echography   

E. Hysterosalpingography   

 

 62 A 13 year old patient is suffering from an acute disease with the following symptoms: thirst, polyuria, weakness. Objectively: his  general condition is satisfactory, there is no smell of acetone. Glucose concentration in blood on an empty stomach is 32  micromole/l, in urine – 6%, acetone +. What treatment should be administered?    

A. Diet   

B. Sulfonylurea    

C. Biguanides   

D. Long-acting insulin   

E. Short-acting insulin*    

 

 63 A patient has got acute macrofocal myocardial infarction complicated by cardiogenic shock. The latter is progresing under  conditions of weak general peripheric resistance and lowered cardiac output. What antihypotensive drug should be injected to the  patient in the first place?    

A. Noradrenaline   

B. Adrenaline   

C. Prednisolone   

D. Mesatonum   

E. Dopamine*   

 

 64 A 28 year old patient was admitted to the clinic with complaints of the temperature rise up to 39,0oC, headache, weakness,  constipation on the 9th day of the disease. On examination: single roseolas on the skin of the abdomen are present. The pulse rate  is 78 bpm. The liver is enlarged by 2 cm. What is the most probable diagnosis?    

A. Sepsis    

B. Brucellosis    

C. Typhoid fever*   

D. Leptospirosis   

E. Malaria    

 

 65 A 52 year old woman complains about face distortion. It turned up 2 days ago after supercooling. Objectively: body temperature is  38,2oC. Face asymmetry is present. Frontal folds are flattened. Left eye is wider than right one and doesn’t close. Left  nasolabial fold is flattened, mouth corner is lowered. Examination revealed no other pathology. Blood count: leukocytes –  10*109/l, ESR – 20 mm/h. What is the most probable diagnosis?   

A. Hemicrania (migraine)   

B. Ischemic stroke   

C. Facial neuritis*   

D. Trigeminus neuralgia    

E. Brain tumour   

 

 66 At year-end hospital administration has obtained the following data: annual number of treated patients and average annual number  of beds used for patients’ treatment. What index of hospital work can be calculated on the base of this data?     

A. Average bed idle time    

B. Average annual bed occupancy    

C. Average duration of patients’ presence in the hospital     

D. Bed turnover*     

E. Bed resources of the hospital     

 

 67 A male patient, 60 years old, tobacco smoker for 30 years, alcoholic, has dysphagia and weight loss since 4 months. Suggested  diagnosis?   

A. Esophagitis   

B. Hanter’s disease   

C. Esophageal achalasia   

D. Esophageal diverticulum   

E. Cancer of the esophagus*   

 

 68 A 54 year old female patient was admitted to the hospital with evident acrocyanosis, swollen cervical veins, enlarged liver,  ascites. Cardiac borders are dilated. Heart sounds cannot be auscultated, apical beat is undetectable. AP is 100/50 mm Hg. X-ray  picture of chest shows enlarged heart shadow in form of a trapezium. What pathology might have caused these symptoms?   

A. Complex heart defect   

B. Hiatal hernia   

C. Exudative pleuritis   

D. Acute cardiac insufficiency   

E. Cardiac tamponade*   

 

 69 A healthy 75 year old woman who leads a moderately active way of life went through a preventive examination that revealed  serum concentration of common cholesterol at the rate of 5,1 millimole/l and HDL (high-density lipoproteins) cholesterol at the rate  of 70 mg/dl. ECG reveals no pathology. What dietary recommendation is the most adequate?    

A. Decrease of cholesterol consumption   

B. Decrease of carbohydrates consumption    

C. Increase of cellulose consumption    

D. Decrease of saturated fats consumption     

E. Any dietary changes are necessary*    

 

 70 An 18 year old primigravida in her 27-28 week of pregnancy underwent an operation on account of acute phlegmonous  appendicitis. In the postoperative period it is necessary to take measures for prevention of the following pegnancy complication:    

A. Intestinal obstruction   

B. Late gestosis   

C. Fetus hypotrophy   

D. Noncarrying of pregnancy*   

E. Premature placenta detachment   

 

 71 The observed patient’s movements are retarded, she answers no questions. Sometimes she spontaneously stiffens in strange  postures. It is possible to set her body and limbs into different positions artificially. If the psychiatrist lifts her arm or leg, so that she  remains standing on the other leg, the patient can stay in such a position for quite a long time. Name the probable disorder:    

A. Psychogenic stupor, stress disorder    

B. Depressive stupor, bipolar disorder    

C. Apathetic stupor, schizophrenia    

D. Catatonic stupor, schizophrenia*    

E. Dissociative stupor, dissociative psychosis    

 

 72 The man, aged 42, applied to the therapeutist with complaints of pricking pains in scapulas area, dyspnea on physical exertion,  cough with discharge of small amount of sputum. During 10 years he works in coal mining. On percussion-box-note sound in the  lower parts, on auscultation- a harsh breathing. There were no changes in the heart. Possible diagnosis?   

A. Silicosis*   

B. Silicatosis   

C. Tuberculosis of lungs   

D. Bronchiectatic disease   

E. Chronic bronchitis   

 

 73 A woman complains of having slight dark bloody discharges and mild pains in the lower part of abdomen for several days. Last  menses were 7 weeks ago. The pregnancy test is positive. Bimanual investigation: the body of the uterus indicates for about 5-6  weeks of pregnancy, it is soft, painless. In the left appendage there is a retort-like formation, 7х5 cm large, mobile, painless. What  examination is necessary for detection of fetus localization?    

A. Hysteroscopy    

B. Colposcopy   

C. Cystoscopy   

D. Hromohydrotubation   

E. Ultrasound*   

 

 74 A 34 year old patient was delivered to the hospital because of follicular tonsillitis charactirized by high temperature. The patient  has been abusing alcohol for 12 years. In the evening on the day of hospitalization he became anxious, couldn’t stay in bed, left  his ward several times and applied to the staff on duty with different complaints. He reported about seeing alot of spiders and flies  in his ward as well as abou hearing threats from the corridor. He was exasperated by the fact that other patients didn’t hear  them. He lost also spatial orientation. What psychopathological syndrome is it?    

A. Twilight state   

B. Delirious*   

C. Amentive   

D. Asthenic confusion   

E. Oneiric   

 

 75 A patient complains about evaginations in the region of anus that appear during defecation and need to be replaced. Examination  with anoscope revealed 1×1 cm large evaginations of mucosa above the pectineal line. What is the most probable diagnosis?    

A. Acute paraproctitis   

B. –   

C. Anal fissure   

D. External hemorrhoids   

E. Internal hemorrhoids*   

 

 76 A 72 year old patient complains about pain and bleeding during defecation. Digital rectal investigation revealed a tumour of anal  canal. After verification of the diagnosis the patient was diagnosed with squamous cell carcinoma. The secondary (metastatic)  tumour will be most probably found in:   

A. Brain   

B. Liver   

C. Pelvic bones   

D. Lungs*   

E. Mediastinum   

 

 77 A 45 year old woman complains about unbearable pain attacks in the left part of face lasting 1-2 minutes. Such attacks are  provoked by mastication. These symptoms appeared 2 months ago after exposure to cold. Objectively:the pain is localized in the  ending points of trigeminus on the left. A touch near nose wing provokes another pain attack and tonic spasm of face muscles.  What is the most probable diagnosis?    

A. Facial migraine   

B. Maxillary sinusitis   

C. Arthritis of mandibular joint   

D. Neuralgia of trigeminus*   

E. Neuralgia of glossopharyngeal nerve   

 

 78 A 38 year old female patient complains about body stiffness in the morning, especially in the articulations of her upper and lower  limbs, that disappears 30-60 minutes later after active movements. She has also arthritis of metacarpophalangeal and proximal  phalangeal articulations, subfebrile temperature. ESR- 45 mm/h. Roentgenography revealed osteoporosis and erosion of articular  surface of small hand and foot articulations. What is the most probable diagnosis?    

A. Rheumatoid arthritis*   

B. Psoriatic arthropathy   

C. Systemic lupus erythematosus   

D. Osteoarthrosis deformans   

E. Reactive polyarthritis   

 

 79 In the woman of 24 years about earlier normal menstrual function, cycles became irregular, according to tests of function  diagnostics – anovulatory. The contents of prolactin in blood is boosted. Choose the most suitable investigation:   

A. USI of organs of small pelvis   

B. Progesterone assay   

C. Determination of the contents of testosteron-depotum in blood serum   

D. Determination of the level of gonadotropins    

E. Computer tomography of the head*   

 

 80 A 30 year old man complains of intense pain, reddening of skin, edema in the ankle-joint area, fever up to 39oC. There was an  acute onset of the illness. In the past there were similar attacks lasting 5-6 days without residual changes in the joint. The skin  over the joint is hyperemic and ill-defined, without infiltrative bank on the periphery. What is the most likely diagnosis?       

A. Erysipelatous inflammation   

B. Rheumatoid arthritis   

C. Gout*   

D. Infectious arthritis   

E. Osteoarthritis   

 

 

 81 A woman of a high-risk group (chronic pyelonephritis in anamnesis) had vaginal delivery. The day after labour she complained of  fever and loin pains, frequent urodynia. Specify the most probable complication:   

A. Apostasis of sutures after episiotomy    

B. Thrombophlebitis of veins of the pelvis   

C. Endometritis   

D. Infectious contamination of the urinary system*   

E. Infectious hematoma   

 

 82 A 3 month old infant suffering from acute segmental pneumonia has dyspnea (respiration rate – 80 per minute), paradoxical  breathing, tachycardia, total cyanosis. Respiration and pulse – ratio is 1:2. The heart dullness under normal size. Such signs  characterise:   

A. Respiratory failure of II degree   

B. Myocarditis    

C. Respiratory failure of I degree   

D. Congenital heart malformation   

E. Respiratory failure of III degree*   

 

 83 Indices that characterize population health include demographic indices. What environment is used for calculation of these  indices?     

A. Number of patients   

B. Population number*    

C. Number of population being liable to preventive examination    

D. Employment number    

E. Number of hospitalized people    

 

 84 Surgical department admitted a 37 year old patient with a big crushed wound of his left thigh 4 hours after he got this trauma.  What is the main provision for successful prevention of gaseous gangrene?    

A. Wound lavage with 6% solution of hydrogen peroxide    

B. Infiltration of soft tissues around the wound with antibiotic solution   

C. Removal of necrotic tissues and timely surgical processing of the wound*    

D. Injection of specific serum 3 000 U   

E. Injection of specific serum 30 000 U    

 

 85 A 50 year old locksmith was diagnosed with typhoid fever. The patient lives in a separate apartment with all facilities. Apart of him  there are also 2 adults in his family. What actions should be taken about persons communicating with the patient?    

A. Vaccination   

B. Bacteriological study*   

C. Antibiotic prophylaxis    

D. Dispensary observation   

E. Isolation   

 

 86 Three weeks after acute angina the patient is still weak, inert, subfebrile, his retromaxillary lymph nodes are enlarged. Tonsils are  flabby, stick together with arches, there are purulent plugs in lacunae. What is the most probable diagnosis?    

A. Acute lacunar tonsillitis   

B. Chronic tonsillitis*   

C. Chronic pharyngitis   

D. Paratonsillitis   

E. Tonsillar tumour   

 

 87 A 4 month old child was admitted to a surgical department 8 hours after the first attack of anxiety. The attacks happen every 10  minutes and last for 2-3 minutes, there was also one-time vomiting. Objectively: the child’s condition is grave. Abdomen is soft,  palpation reveals a tumour-like formation in the right iliac area. After rectal examination the doctor’s finger was stained with blood.  What is the most probable diagnosis?    

A. Gastrointestinal haemorrhage   

B. Helminthic invasion   

C. Ileocecal invagination*   

D. Pylorostenosis   

E. Wilm’s tumour   

 

 88 After objective clinical examination a 12 year old child was diagnosed with mitral valve prolapse. What complementary instrumental  method of examination should be applied for the diagnosis confirmation?    

A. Phonocardiography   

B. ECG   

C. Roentgenography of chest   

D. Echocardiography*   

E. Veloergometry   

 

 89 A female patient has been suffering from pain in the right subcostal area, bitter taste in the mouth, periodical bile vomiting for a  month. The patient put off 12 kg. Body temperature in the evening is 37,6oC. Sonography revealed that bile bladder was  5,5х2,7 cm large, its wall – 0,4 cm, choledochus – 0,8 cm in diameter. Anterior liver segment contains a roundish hypoechoic  formation up to 5 cm in diameter and another two up to 1,5 cm each, walls of these formations are up to 0,3 cm thick. What is the  most probable diagnosis?    

A. Alveolar echinococcus of liver*   

B. Liver cancer   

C. Cystous liver cancer   

D. Liver abscess   

E. Paravesical liver abscesses   

 

 90 On the 5th day after labor body temperature of a parturient suddenly rose up to 38,7oC. She complains about weakness,  headache, abdominal pain, irritability. Objectively: AP- 120/70 mm Hg, Ps- 92 bpm, to– 38,7oC. Bimanual examination  revealed that the uterus was enlarged up to 12 weeks of pregnancy, it was dense, slightly painful on palpation. Cervical canal lets  in 2 transverse fingers, discharges are moderate, turbid, with foul smell. In blood: skeocytosis, lymphopenia, ESR- 30 mm/h. What  is the most probable diagnosis?   

A. Parametritis   

B. Endometritis*   

C. Lochiometra   

D. Metrophlebitis    

E. Pelviperitonitis    

 

 91 A fitter of a metallurgic factory with occupational exposure to high concentrations of mercury fumes for 16 years presents  instability of pulse and blood pressure, general hyperhydrosis, asymmetric innervations of facial muscles and tongue, positive  subcortical reflexes, hand tremor on physical examination. A dentist revealed paradontosis and chronic stomatitis. What is the  most probable diagnosis?    

A. Acute mercury intoxication   

B. Mercury encephalopathy   

C. Parkinson syndrome   

D. Chronic mercury intoxication*   

E. Neuroinfection   

 

 92 A 40 year old female patient has been observing excessive menstruation accompanied by spasmodic pain in the lower abdomen  for a year. Bimanual examination performed during menstruation revealed a dense formation up to 5 cm in diameter in the cervical  canal. Uterus is enlarged up to 5-6 weeks of pregnancy, movable, painful, of normal consistency. Appendages are not palpabl E.  Bloody discharges are profus E. What is the most probable diagnosis?    

A. Abortion in progress   

B. Cervical myoma   

C. Algodismenorrhea   

D. Cervical carcinoma   

E. Nascent submucous fibromatous node*   

 

 93 Examination of a 26 year old female patient revealed a node in the right lobe of thyroid gland. The node appeared no earlier than 3  months ago. The patient associates this node with stress. She doesn’t complain either about pain or enlargement of the node.  Ultrasonic scanning revealed a 2×2,5 cm large node in the inferior part of the right lobe of thyroid gland. What treatment should be  administered?    

A. Dynamic observation   

B. No need for treatment    

C. –

D. Conservative therapy   

E. Surgical intervention*   

 

 

 94 A 2 year old girl has been ill for 3 days. Today she has low grade fever, severe catarrhal presentations, slight maculopapular rash  on her buttocks and enlarged occipital lymph nodes. What is your diagnosis?    

A. Rubella*    

B. Pseudotuberculosis   

C. Adenoviral infection   

D. Measles   

E. Scarlet fever   

 

 95 Administration of a plant producing red lead paint intends to form a group of medical specialists for periodical medical  examinations. What specialist must be obligatory included into this group?     

A. Otolaryngologyst   

B. Psychiatrist   

C. Neuropathologist*    

D. Gynaecologist    

E. Dermatologist   

 

 96 Study of morbidity rate in a city N revealed that population of different administrative districts differed in age structure. What  statistic method allows to eliminate influence of this factor upon morbidity indices?     

A. Correlative regressive analysis   

B. Analysis of dynamic series   

C. Standardization*    

D. Wilcoxon’s t-criterion    

E. Calculation of average values   

 

 97 A 33 year old patient has acute blood loss (erythrocytes – 2,2*1012/l, Hb- 55 g/l), blood group is A(II)Rh+. Accidentally  the patient got transfusion of donor packed red blood cells of AB(IV)Rh+ group. An hour later the patient became anxious, got  abdominal and lumbar pain. Ps- 134 bpm, AP- 100/65 mm Hg, body temperature – 38,6oC. After catheterization of urinary  bladder 12 ml/h of dark-brown urine were obtained. What complication is it?    

A. Toxic infectious shock   

B. Cardial shock   

C. Allergic reaction to the donor red blood cells   

D. Citrate intoxication     

E. Acute renal insufficiency*   

 

 98 A department chief of an in-patient hospital is going to examine resident doctors as to observation of medical-technological  standards of patient service. What documentation should be checked for this purpose?    

A. Annual report of a patient care institution    

B. Statistic cards of discharged patients   

C. Health cards of in-patients*    

D. Treatment sheets    

E. Registry of operative interventions    

 

 99 A boy is 1 year old. Previously he had purulent otitis. After that he started complaining about pain in the upper third of his left thigh,  body temperature rose up to 39oC. Objectively: the thigh is swollen in its upper third, inguinal fold is smoothed. Extremity is in  half-bent position. Active and passive movements are impossible because of acute pain. What is the most probable diagnosis?    

A. Osteosarcoma   

B. Brodie’s abscess   

C. Acute coxitis   

D. Intermuscular phlegmon   

E. Acute haematogenic osteomyelitis*   

 

 100 A woman consulted a doctor on the 14th day after labour about sudden pain, hyperemy and induration of the left mammary gland,  body temperature rise up to 39oC, headache, indisposition. Objectively: fissure of nipple, enlargement of the left mammary  gland, pain on palpation. What pathology would you think about in this case?    

A. Breast cancer   

B. Lactational mastitis*   

C. Lacteal cyst with suppuration   

D. Phlegmon of mammary gland   

E. Fibrous adenoma of the left mammary gland   

 

 101 The most available and informative diagnostic method for closed trauma of the urinary bladder is:    

A. Pelvic arteriography   

B. Sonography of the urinary bladder   

C. Palpation and percussion of abdomen   

D. Cystography   

E. Retrograde cystography*    

 

 102 The 25 year old patient was admitted on the 1st day of the disease with complaints of double vision in the eyes, difficult  respiration. The day before the patient ate home-made mushrooms. On objective examination: paleness, widened pupils, disorder  of swallowing, bradycardia, constipation are marked.  What is the diagnosis?    

A. Botulism*   

B. Leptospirosis   

C. Yersiniosis   

D. Salmonellosis, gastrointestinal form   

E. Lambliasis    

 

 103 Examination of a 12 year old child revealed diffuse thyroid enlargement of the II degree. Heart auscultation revealed dullness of  heart sounds, heart rate was 64/min. The child has frequent constipations, anemia. Concentration of thyreoglobulin antibodies is  increased. What disease might have caused such symptoms?    

A. Thyroid carcinoma   

B. Autoimmune thyroiditis*   

C. Thyroid hyperplasia   

D. Endemic goiter   

E. Diffuse toxic goiter   

 

 104 A 60 year old man with unstable angina pectoris fails to respond to heparin, nitroglycerin, beta adrenegic blockers and calcium  channel antagonist. The best management includes:   

A. Antihypertensive therapy   

B. Coronary artery bypass grafting*   

C. Intravenous streptokinase   

D. Excercise testing   

E. Oral aspirin   

 

 105 A 29 year old female patient complains about periodical right-sided headache that is usually provoked by strong smells or  excitement. The patient’s mother suffers from the same disease. Objectively: examination of internal organs revealed no  pathology. During the attack there are general hyperesthesia and nausea, at the end of the attack polyuria is observed. Palpation  of the right temporal artery during the attack revealed tension and painfulness of the artery. Complete blood count and urine  analysis reveale dno pecularities. What is the most probable diagnosis?    

A. Neuralgia of trigeminus   

B. Migraine*    

C. Neuritis of the facial nerve   

D. Meniere’s syndrome   

E. Epilepsy   

 

 106 A 59 year old female patient applied to a maternity welfare clinic and complained about bloody discharges from the genital tracts.  Postmenopause is 12 years. Vaginal examination revealed that external genital organs had signs of age involution, uterus cervix  was not erosive, small amount of bloody discharges came from the cervical canal. Uterus was of normal size, uterine  appendages were unpalpable. Fornices were deep and painless. What method should be applied for the diagnosis specification?     

A. Culdoscopy   

B. Puncture of abdominal cavity through posterior vaginal fornix   

C. Extensive colposcopy   

D. Separated diagnosic curretage*   

E. Laparoscopy   

 

 107 A 25 year old patient had pharyngitis 2 weeks ago. Now he complains about body temperature rise up to 38oC, general  weakness, dyspnea during walking, swelling and shifting pain in the articulations. Objectively: cyanosis of lips, rhythmic pulse of  poor volume – 100 bpm. Left cardiac border deviates outwards from the mediaclavicular line by 1 cm. The first heart sound is  weakened on the apex, auscultation revealed systolic souffle. What is the most probable aetiological factor that caused this  pathological process?    

A. Pneumococcus   

B. Virus   

C. Fungi   

D. b-haemolytic streptococcus*   

E. Staphylococcus   

 

 108 Study of morbidity with temporary disability among workers of a machine building plant revealed that average duration of a case  was 20 days. What diseases influenced upon the index value?     

A. Hard to determine    

B. Preexisting diseases    

C. Subacute    

D. Chronic*   

E. Acute    

 

 109 A 68 year old female patient complains about temperature rise up to 38,3oC, haematuria. ESR- 55 mm/h. Antibacterial therapy  turned out to be ineffective. What diagnosis might be suspected?    

A. Renal cancer*   

B. Renal amyloidosis    

C. Chronic glomerulonephritis    

D. Polycystic renal disease   

E. Urolithiasis    

 

 110 A pediatrician talked to a mother of a 7 month old breast-fed boy and found out that the child was fed 7 times a day. How many  times should the child of such age be fed?     

A. 7 times   

B. 3 times   

C. 5 times*   

D. 4 times   

E. 6 times  

 

 111 A 72 year old male patient complains about itch in his left shin, especially around a trophic ulcer. Skin is reddened and edematic,  there are some oozing lesions, single yellowish crusts. The focus of affection is well-defined. What is the most probable  diagnosis?    

A. Streptococcal impetigo   

B. Seborrheic eczema    

C. Microbial eczema*   

D. Allergic dermatitis   

E. Cutaneous tuberculosis    

 

 112 A man, aged 37, working on the collective farm on sowing, was admitted to the infectious hospital with the clinical symptoms:  miosis, labored breathing, sweating.What kind of poisoning is it and what is the first aid?   

A. Poisoning by lead. Treatment: tetacine Calcii   

B. –

C. Poisoning by POC. Treatment: atropine*   

D. Poisoning by the methylic alcohol. Treatment: ethylic alcohol   

E. Poisoning by vapours of mercury. Treatment: unithiol   

 

 113 A 25 year old woman applied to a maternity welfare clinic and complained about being unable to conceive within 3 years of  regular sexual life. Examination revealed weight gain, male pattern of hair distribution on the pubis, excessive pilosis of thighs.  Ovaries were dense and enlarged, basal temperature was monophase. What is the most probable diagnosis?     

A. Tubo-ovaritis   

B. Premenstrual syndrome   

C. Gonadal dysgenesis    

D. Adrenogenital syndrome   

E. Sclerocystosis of ovaries*   

 

 114 A 52 year old patient has hypervolaemic type of essential hypertension. Which of the following medications is to be prescribed  either as monotherapy or in complex with other antihypertensive drugs?    

A. Nifedipin   

B. Clonidine   

C. Dibazol   

D. Kapoten   

E. Hypothiazid*   

 

 115 A woman is 34 years old, it is her tenth labor at full term. It is known from the anamnesis that the labor started 11 hours ago, labor  was active, painful contractions started after discharge of waters and became continuous. Suddenly the parturient got knife-like  pain in the lower abdomen and labor activity stopped. Examination revealed positive symptoms of peritoneum irritation, ill-defined  uterus outlines. Fetus was easily palpable, movable. Fetal heartbeats wasn’t auscultable. What is the most probable diagnosis?    

A. Risk of uterus rupture   

B. Uterine inertia   

C. Discoordinated labor activity   

D. II labor period   

E. Rupture of uterus*   

 

 116 A patient complained about problems with pain and tactile sensitivity, pain in the nail bones at the end of the working day. He  works at a plant with mechanical devices. What pathology can be suspected?   

A. Vibration disease*   

B. Overwork symptoms   

C. Hypovitaminosis of B1   

D. Caisson disease   

E. Noise disease   

 

 

 

 

 117 A 14 year old girl complains of profuse bloody discharges from genital tracts during 10 days after suppresion of menses for 1,5  month. Similiar bleedings recur since 12 years on the background of disordered menstrual cycle. On rectal examination: no  pathology of the internal genitalia. In blood: Нb – 70 g/l, RBC- 2,3*1012/l, Ht – 20. What is the most probable diagnosis?    

A. Werlholf’s disease     

B. Polycyst ovarian syndrome      

C. Incomplete spontaneous abortion    

D. Hormonoproductive ovary tumor    

E. Juvenile bleeding, posthemorrhagic anemia*     

 

 118 A 26 year old manual worker complained of 3 weeks history of fevers and fatigue, weight loss with no other symptoms. Physical  findings: Temperature 37,6oC, Ps- 88 bpm, blood pressure 115/70 mm Hg, superficial lymph nodes (occipital, submental,  cervical, axillary) are enlarged, neither tender nor painful. Rubella-like rash on the trunk and extremities. Herpes simplex lesions on  the lips. Candidosis of oral cavity. What infectious disease would you suspect?    

A. HIV infection*    

B. Rubella   

C. Infectious mononucleosis    

D. Influenza   

E. Tuberculosis   

 

 119 Student В. lives in the canalized house in the flat with complete set of sanitary equipment (WC, bath, shower, local water heater).  How much water consumption has he got?   

A. 10-15 l   

B. 500 l   

C. 160-200 l*

D. 50-100 l   

E. 300-400 l   

 

 

 

 120 Examination of a 3-month-old child revealed scrotum growth on the right. This formation has elastic consistency, its size  decreases during sleep and increases when the child is crying. What examination will be helpful for making a correct diagnosis?    

A. Punction of the scrotum   

B. Palpation of the thickened cord crossing the pubical tubercule (sign of the silk glove)*    

C. Examination of the formation in Trendelenburg’s position   

D. Diaphanoscоpy   

E. Palpation of the external inguinal ring   

 

 121 A 48 year old female patient complains about contact haemorrhage. Speculum examination revealed hypertrophy of uterus cervix.  It resembles of cauliflower, it is dense and can be easily injured. Bimanual examination revealed that fornices were shortened,  uterine body was nonmobile. What is the most probable diagnosis?   

A. Cervical papillomatosis   

B. Endometriosis   

C. Metrofibroma   

D. Cervical carcinoma*   

E. Cervical pregnancy   

 

 122 Prophylactic photoroentgenography examination of a 25 year old man revealed focal shadowings of small and medium intensity  with irregular contours in the 1st and 2nd segments of the right lung. Which clinical form can be suspected?    

A. Tuberculoma   

B. Fibro-cavernous   

C. Disseminated   

D. Miliary   

E. Focal*   

 

 123 A 39 year old patient suffering from hypertension suddenly gotb intense headache in the region of occiput; there appeared  recurrent vomiting. These presentations has been lasting for 5 hours. Objectively: Ps – 88 bpm, AP – 205/100 mm Hg, painfulness  of occipital points, rigidity of occipital muscles are present. Kernig’s symptom is bilaterally positive. Subarachnoid haemorrhage is  also suspected. What diagnostic method will be of the greatest importance for confirmation of provisional diagnosis?    

A. Ultrasonic dopplerography   

B. Examination of eye fundus   

C. EEG   

D. Computer tomography   

E. Lumbar puncture*   

 

 124 A prematurely born girl is now 8 months old. She has dyspnea, tachycardia, hepatosplenomegaly, physical developmental lag, limb  cyanosis. There is also parasternal cardiac hump, auscultation revealed systolodiastolic murmur in the II intercostal space on the  left. AP is 90/0 mm Hg. What disease should be suspected?    

A. Nonclosure of interventricular septum   

B. Stenosis of aortal valve   

C. Coarctation of aorta   

D. Patent ductus arteriosus*   

E. Stenosis of pulmonary artery   

 

 125 A 32 year old welder complains of weakness and fever. His illness started as tonsillitis a month before. On exam, BT of  38,9oC, RR of 24/min, HR of 100/min, BP of 100/70 mm Hg, hemorrhages on the legs, enlargement of the lymph nodes. CBC  shows Hb of 70 g/l, RBC of 2,2*1012/l, WBC of 3,0*109/l with 32% of blasts, 1% of eosinophiles, 3% of bands,  36% of segments, 20% of lymphocytes, and 8% of monocytes, ESR of 47 mm/h. What is the cause of anemia?   

A. Vitamin B12 deficiency anemia   

B. Chronic lympholeukemia   

C. Acute leukemia*   

D. Chronic hemolytic anemia   

E. Aplastic anema   

 

 126 A 26 year old woman complains about edemata, swelling and painfulness of mammary glands, headache, tearfulness, irritability.  These signs turn up 5 days before menstruation and disappear after its start. What clinical syndrome is it?    

A. Premenstrual syndrome*   

B. Climacteric syndrome    

C. Postcastration syndrome   

D. Adrenogenital syndrome   

E. Stein-Leventhal syndrome    

 

 127 A 38 year old patient complains about inertness, subfebrile temperature, enlargement of lymph nodes, nasal haemorrhages, bone  pain. Objectively: the patient’s skin and mucous membranes are pale, palpation revealed enlarged painless lymph nodes;  sternalgia; liver was enlarged by 2 cm, spleen – by 5 cm, painless. In blood: erythrocytes – 2,7*1012/l, Hb- 84 g/l,  leukocytes – 58*109/l, eosinophils – 1%, stab neutrophils – 2%, segmented neutrophils – 12%, lymphocytes – 83%,  lymphoblasts – 2%, smudge cells; ESR- 57 mm/h. What is the most probable diagnosis?   

A. Lymphogranulomatosis   

B. Chronic lymphatic leukemia*   

C. Acute myeloleukemia   

D. Acute lymphatic leukemia   

E. Chronic myeloleukemia   

 

 128 A 32 year old patient suffering from chronic viral hepatitis complains about dull pain in the right subcostal area, nausea, dry mouth.  Objectively: liver dimensions are 13-21-11 cm (according to Kurlov), spleen is by 2 cm enlarged, aspartate aminotransferase is  3,2 micromole/l*h, alanine aminotransferase – 4,8 millimole/l*h. Serological study revealed HBeAg, high  concentration of DNA HBV. What drug should be chosen for treatment of this patient?     

A. a-interferon*   

B. Arabinoside monophosphate    

C. Acyclovir    

D. Essentiale-forte    

E. Remantadinum    

 

 129 A parturient complains about pain in the mammary gland. Palpation revealed a 3х4 cm large infiltration, soft in the centre. Body  temperature is 38,5oC. What is the most probable diagnosis?   

A. Retention of milk   

B. Birth trauma   

C. Acute purulent mastitis*   

D. Pneumonia   

E. Pleuritis   

 

 130 A 3 year old child has been suffering from fever, cough, coryza, conjunctivitis for 4 days. He has been taking sulfadimethoxine.  Today it has fever up to 39oC and maculopapular rash on its face. Except of rash the child’s skin has no changes. What is  your diagnosis?    

A. Measles*   

B. Pseudotuberculosis   

C. Scarlet fever   

D. Rubella   

E. Allergic rash   

 

 131 A 15 year old girl suddenly got arthralgia, headache, nausea, vomiting; pain and muscle tension in the lumbar area; body  temperature rose up to 38-39oC. Pasternatsky’s symptom was distinctly positive on the right. In the urine: bacteriuria, pyuria.  What is the most probable diagnosis?    

A. Renal colic   

B. Acute pyelonephritis*   

C. Pararenal abscess   

D. Cystitis   

E. Acute glomerulonephritis   

 

 132 A 38 year old man was delivered to the hospital in unconscious state. The symptoms of illness turned up a day before: headache,  nausea, vomiting, to– 38,5oC, dizziness, delusion. For the last 4 days he had been complaining of pain and hearing loss in  the left ear. Objectively: sopor, rigidity of occipital muscles, bilateral Kernig’s symptom, general hyperesthesia, purulent discharges  from the left ear. What is the most probable diagnosis?     

A. Secondary purulent meningitis*   

B. Subarachnoidal haemorrhage   

C. Parenchymatous subarachnoidal haemorrhage   

D. Tuberculous meningitis   

E. Primary purulent meningitis   

 

 133 During examination a patient is unconscious, his skin is dry and hot, face hyperemia is present. The patient has Kussmaul’s  respiration, there is also smell of acetone in the air. Symptoms of peritoneum irritation are positive. Blood sugar is at the rate of 33  millimole/l. What emergency actions should be taken?    

A. Intravenous infusion of short-acting insulin*   

B. Intravenous infusion of sodium chloride saline   

C. Intravenous infusion of glucose along with insulin   

D. Intravenous infusion of neohaemodesum along with glutamic acid   

E. Introduction of long-acting insulin   

 

 134 A 20 year old patient was delivered to the hospital in summer from the street with haemorrage from the brachial artery. First  medical aid involved aplication of a tourniquet for provisional arrest of bleeding. What is the maximal exposure of the tourniquet?    

A. 60 minutes   

B. 15 minutes   

C. 120 minutes*   

D. 180 minutes   

E. 30 minutes   

 

 135 A 19 year old boy was admitted to a hospital with closed abdominal trauma. In course of operation multiple ruptures of spleen and  small intestine were revealed. AP is falling rapidly, it is necessary to perform hemotransfusion. Who can specify the patient’s  blood group and rhesus compatibility?      

A. A traumatologist   

B. An anaesthesilogist   

C. A laboratory physician    

D. A doctor of any speciality*   

E. A surgeon   

 

 136 A 35 year old female patient suffering from cholelithiasis has broken her diet, and this caused an acute pain attack in the right  subcostal are. The pain eased off on the third day, but the patient got progressing jaundice. What non-invasive diagnostic method  should be applied?    

 

A. Infusive cholecystocholangiography   

B. Test for bilirubin    

C. Survey radiography of abdominal organs   

D. Endoscopic retrograde cholangiopancreatography*    

E. Duodenal probing   

 

 137 Examination of a 9 month old girl revealed skin pallor, cyanosis during excitement. Percussion revealed transverse dilatation of  cardiac borders. Auscultation revealed continuous systolic murmur on the left from the breastbone in the 3-4 intercostal space.  This murmur is conducted above the whole cardiac region to the back. What congenital cardiac pathology can be suspected?    

A. Fallot’s tetrad   

B. Coarctation of aorta   

C. Pulmonary artery stenosis    

D. Defect of interatrial septum    

E. Defect of interventricular septum*   

 

 138 A 26 year old woman who delivered a child 7 months ago has been suffering from nausea, morning vomiting, sleepiness for the  last 2 weeks. She suckles the child, menstruation is absent. She hasn’t applied any contraceptives. What method should be  applied in order to specify her diagnosis?    

A. Roentgenography of small pelvis organs   

B. Bimanual vaginal examination   

C. Palpation of mammary glands and pressing-out of colostrum   

D. Speculum examination   

E. Ultrasonic examination*   

 

 139 An 18 year old patient was admitted to a hospital with complaints of headache, weakness, high temperature, sore throat.  Objectively: enlargement of all groups of lymph nodes was revealed. The liver is enlarged by 3 cm, spleen – by 1 cm. In blood:  leukocytosis, atypical lymphocytes – 15%. What is the most probable diagnosis?    

A. Infectious mononucleosis *  

B. Diphtheria   

 

C. Acute lymphoid leukosis   

D. Adenoviral infection   

E. Angina   

 

 140 A 54 year old male patient complains about permanent dull pain in the mesogastral region, weight loss, dark blood admixtures in the  feces, constipations. He put off 10 kg within a year. In blood: erythrocytes: 3,5*1012/l, Hb- 87 g/l, leukocytes –  12,6*109/l, stab neutrophil shift, ESR- 43 mm/h. What is the most probable diagnosis?    

A. Cancer of transverse colon*    

B. Gastric ulcer   

C. Chronic pancreatitis   

D. Chronic colitis   

E. Stomach cancer   

 

 141 A patient who takes diuretics has developed arrhythmia as a result of cardiac glycoside overdose. What is the treatment tactics in  this case?     

A. Increased potassium concentration in blood*   

B. –   

C. Increased calcium concentration in blood    

D. Reduced magnesium concentration in blood   

E. Increased sodium consentration in blood   

 

 142 It is planned to make complete isolation boxes in the infectious department in order to prevent nosocomial airborne infections. The  boxes consist of a tambour, a ward and a lock chamber. What structure should be also included in a complete isolation box?    

A. Manipulation room   

B. Bathroom unit*   

C. Nursing room   

D. Patient’s examination room   

E. Doctor’s consulting room   

 

 143 A maternity hospital registered 616 live births, 1 stillbirth, 1 death on the 5th day of life over a 1 year period. What index allows the  most precise estimation of this situation?    

A. Crude mortality rate    

B. Neonatal mortality    

C. Perinatal mortality*    

D. Natural increase    

E. Natality    

 

 144 A 38 year old man, previously in good health, suddenly develops severe abdominal pain radiating from the left loin to groin and  accompanied by nausea, perspiration and the need for frequent urination. He is restless, tossing in bed but has no abnormal  findings. The most likely diagnosis is:   

A. Torsion of the left testicle   

B. Retroperitoneal haemorrhage   

C. Sigmoid diverticulitis   

D. Leftsided renal colic*   

E. Herpes zoster   

 

 145 Indicate the registration medical document for the patient, who 21.02. was addressed to the doctor with diagnosis ARVD for the  first time in this year:   

A. The statistical coupon is to be filled in, but a sign (+) is not necessary to be put in   

B. It is necessary to fill in the emergency notice on a case of a contagion    

C. The statistical coupon is to be filled in and it is necessary to deliver on a sign (+) *  

D. The statistical coupon for registration of final diagnosis is not necessary   

E. The necessary registration form is not indicated   

 

 146 A 52 year old patient with history of functional Class II angina complains of having intense and prolonged retrosternal pains,  decreased exercise tolerance for 5 days. Angina is less responsive to nitroglycerine. What is the most probable diagnosis?    

A. IHd. Functional Class II angina   

B. IHd. Unstable angina*

C. Cardialgia due to spine problem   

D. Myocarditis   

E. Myocardial dystrophy   

 

 

147 A woman 26 years old has abused alcohol for 7 years. She has psychological dependence on alcohol, but no withdrawal  syndrome. Drinks almost every day approximately 50-100 g of wine. She is in her 4-th week of pregnancy. Primary prevention of  fetal alcohol syndrome requires:    

A. Medical abortion     

B. Treatment of alcoholism and full abstinance from alcohol during all the period of pregnancy*    

C. Decrease of alcohol use    

D. Participation in the A-ANON group     

E. Gyneacological observation    

 

 148 A 22 year old woman complained of right subcostal aching pain, nausea, and decreased appetite. She fell ill 2 months after  appendectomy when jaundice appeared. She was treated in an infectious hospital. 1 year later above mentioned symptoms  developed. On exam: the subicteric sclerae, enlarged firm liver. Your preliminary diagnosis:   

A. Calculous cholecystitis   

B. Chronic viral hepatitis*   

C. Acute viral hepatitis   

D. Gilbert’s disease   

E. Chronic cholangitis   

 

 149 A 33 year old patient was delivered to the infectious diseases department on the 7-th day of disease. He complained about great  weakness, high temperature, pain in the lumbar area and leg muscles, icteritiousness, dark colour of urine, headache. The acute  disease started with chill, body temperature rise up to 40oC, headache, pain in the lumbar area and sural muscles. Icterus  turned up on the 4th day, nasal and scleral haemorrhages came on the 5th day. Fever has lasted for 6 days. Diuresis – 200 ml.  What is the most probable diagnosis?    

A. Sepsis   

B. Yersiniosis   

 

C. Typhoid fever   

D. Virus A hepatitis   

E. Leptospirosis*   

 

 150 A 13 year old girl was admitted to the cardiological department because of pain in the muscles and joints. Examination of her face  revealed an edematic erythema in form of butterfly in the region of nose bridge and cheeks. What is the most probable diagnosis?     

A. Rheumatoid arthritis   

B. Periarteritis nodosa   

C. Systemic lupus erythematosus*   

D. Rheumatism   

E. Dermatomyositis   

 

 151 A 23 year old patient fell ill 3 weeks ago when she noticed a very painful induration in her axillary crease. 4-5 days later it burst  and discharged a lot of pus. After that some new infiltrations appeared around the affected area. The patient has never suffered  from skin diseases before. What is the most probable diagnosis?    

A. Hydradenitis*   

B. Streptococcal impetigo   

C. Furuncle   

D. Mycosis   

E. Herpes zoster   

 

 152 A pregnant woman in her 40th week of pregnancy undergoes obstetric examination: the cervix of uterus is undeveloped. The  oxytocin test is negative. Examination at 32 weeks revealed: AP 140/90 mm Hg, proteinuria 1 g/l, peripheral edemata. Reflexes are  normal. Choose the most correct tactics:    

A. Labour stimulation after preparation*   

B. Absolute bed rest for 1 month   

C. Complex therapy of gestosis for 7 days    

D. Caesarian section immediately   

E. Complex therapy of gestosis for 2 days   

 

 153 A patient has restrained umbilateral hernia complicated by phlegmon hernia, it is necessary to take following actions:    

A. Herniotomy by Grenov   

B. Herniotomy by Lekser   

C. Herniotomy by Mayo    

D. Herniotomy by Sapezhko   

E. Herniotomy by Mayo-Sapezhko*   

 

 154 A 25 year old woman complained of edema on her face and legs, rise of blood pressure up to 160/100 mm Hg and weakness. She  fell ill 3 weeks after recovering from angina. Urinalysis data: protein of 0,5 g/l, erythrocytes of 17-20/field, leukocytes of 2-3/field,  erythrocyte casts. What treatment should be initiated after specifying the diagnosis?    

A. Penicillin OS*   

B. Dipyridamole   

C. Heparin   

D. Ceftriaxone   

E. Ciprofloxacine   

 

 155 A 40 year old patient was bitten by a stray dog for about an hour ago. The bite can be seen on the patient’s left shin in form of a  wound 4х2х0,5 cm large. What kind of aid would be recommended in this case?    

A. Blind suture   

B. Retension sutures   

C. Wound lavage with soapsuds, retension sutures*   

D. Aseptic bandage   

E. Salve bandage   

 

 156 A plot of land with total area of 2,0 hectare was intended for building of a hospital. The maximal capacity of the hospital will be:    

A. 200 beds   

 

B. 100 beds*    

C. Over 1000 beds   

D. 800 beds    

E. 400 beds   

 

 157 A 38 year old man worked at roofing and drain pipes production for 15 years. He seeks medical help for expiratory  breathlessness on exertion, and dry cough. On exam, wheezes above both lungs, grayish warts on fingers are seen. Factory  physician has diagnosed asbestosis. What method is the most important for this diagnosis?   

A. Bronchoscopy   

B. Spirography   

C. Chest X-ray*  

D. Electrocardiography   

E. Blood gas analysis   

 

 158 A patient suffering from gastroesophageal reflux has taken from time to time a certain drug that “reduces acidity” for 5 years. This  drug was recommended by a pharmaceutist. The following side effects are observed: osteoporosis, muscle weakness,  indisposition. What drug has such following effects?    

A. Aluminium-bearing antacid*   

B. Н2-blocker   

C. Metoclopramide   

D. Gastrozepin    

E. Inhibitor of proton pump   

 

 159 A 52 year old patient complains about headache, weakness of his upper left extremity. Neurological symptoms become more  intense during physical stress of the left extremity. Pulsation on the arteries of the left extremity is sharply dampened but it remains  unchanged on the carotid arteries. What is the most probable diagnosis?    

A. Thoracal outlet syndrome   

B. Raynaud’s syndrome   

C. Occlusion of brachiocephalic trunk   

D. Takayasu’s disease   

E. Occlusion of the left subclavicular artery, steal syndrome*   

 

 160 A 42 year old man applied to a hospital 10 minutes after he got stung by a bee and complained about face edema and difficult  respiration. Objectively: Ps- 98 bpm, AP- 130/80 mm Hg. A doctor on duty injected him 1 ml of 1% dimedrol solution intramuscularly  and recommended to apply to his local therapeutist on the next day. What tactics of treatment should be chosen for this patient?    

A. Intravenous introduction of calcium chloride and hospitalization    

B. Intravenous introduction of calcium chloride   

C. Intravenous introduction of prednisolone and hospitalization*    

D. The patient needs no further medical aid   

E. Hospitalization for observation   

 

 161 It was noticed that a 7 year old pupil had been innattentive several times during the lessons. The teacher also noticed that the child  had been smacking his lips and had vacant look. There were no falls and convulsions. During such short periods of absence the  child didn’t react to calling his name. His mother notic such phenomena before but didn’t pay much attention to them thinking that the  child was deep in thought. What type of epileptic attack (according to the standard classification) is it?     

A. Complex partial epilepsy   

B. Simple partial epilepsy   

C. Generalized tonoclonic epilepsy    

D. Jacksonian partial epilepsy   

E. Absentia*    

 

 162 A 37 year old miner has lifted significant loads and afterwards felt pain in the lumbar area irradiating to his left leg. He walks  slowly and carefully. Lumbar lordosis is flattened. There is also left-sided scoliosis and tension of paravertebral muscles. Neri’s  and Dejerine’s symptoms are positive, there is Lasegue’s sign on the left from the angle of 350. What method will help to specify  the diagnosis?    

A. CT of lumbosacral part of vertebral column*    

B. Lumbal puncture   

C. Renal sonography   

D. Descending myelography   

 

E. Electromyography   

 

 163 A 41 year old woman has suffered from nonspecific ulcerative colitis for 5 years. On rectoromanoscopy: evident inflammatory  process of lower intestinal parts, pseudopolyposive changes of mucous membrane. In blood: WBC- 9,8*109/l, RBC-  3,0*1012/l, ESR – 52 mm/hour. What medication provides pathogenetic treatment of this patient?      

A. Motilium   

B. Sulfosalasine*    

C. Vikasolum   

D. Linex   

E. Kreon   

 

 164 A 19 year old patient was admitted to a hospital with acute destructive appendicitis. He suffers from hemophilia of B type. What  antihemophilic medications should be included in pre- and post-operative treatment plan?     

A. Dried plasma   

B. Native plasma   

C. Fresh frozen blood    

D. Fresh frozen plasma*   

E. Cryoprecipitate   

 

 165 Infant is 6,5 months now and is given natural feeding since birth. Body mass was 3,5 kg, with length 52 cm at birth. How many  times per day the supplement (up feeding) should be given?   

A. 4   

B. 1   

C. 0   

D. 3    

E. 2*   

 

 

 166 The results of 5 year monitoring allowed to estimate the level of environmental influence upon health indices of popultaion. What  statistic method should be chosen?    

A. Calculation of dynamic indices   

B. Calculation of correlation coefficient*   

C. Calculation of coefficient of difference validity   

D. Calculation of regression coefficient   

E. Calculation of conformity coefficient   

 

 167 A 10 month old boy has been ill for 5 days after consumption of unboiled milk. Body temperature is 38-39oC, there is vomiting,  liquid stool. The child is pale and inert. His tongue is covered with white deposition. Heart sounds are muffled. Abdomen is  swollen, there is borborygmus in the region of ubbilicus, liver is enlarged by 3 cm. Stool is liquid, dark-green, with admixtures of  mucus, 5 times a day. What is the most probable diagnosis?    

A. Rotaviral infection   

B. Salmonellosis*    

C. Acute shigellosis   

D. Escherichiosis   

E. Staphylococcal enteric infection    

 

 168 A 34 year old female patient has been suffering from anxious depression accompanied by panic attacks for 2 years. She has  been a patient of a psychotherapist. Treatment resulted in incomplete remission. The patient had to break off psychotherapy  because of moving to a new place of residence. Soon after this her condition grew significantly worse, she was almost  permanently anxious, panic attacks turned up 5-6 times a day and were accompanied by palpitation, dyspnea, cold sweat,  thanatophobia. What drug group is the most appropriate for medicamental therapy?      

A. Cardiotonics, respiratory analeptics   

B. Lithium drugs   

C. Antipsychotic neuroleptics   

D. Sedative neuroleptics   

E. Antidepressants*   

 

 

 169 The student has the following devices: Geiger counter, Ebert counter, Krotov’s apparatus, Mischuk device, Ebert device. What  device can he use to assess air germ pollution?   

A. Mischuk’s device   

B. Krotov’s apparatus*   

C. Geiger’s counter    

D. Ebert’s counter   

E. Ebert’s device   

 

 170 A 42 year old woman complains of dyspnea, edema of the legs and tachycardia during minor physical exertion. Heart borders are  displaced to the left and S1 is accentuated, there is diastolic murmur on apex. The liver is enlarged by 5 cm. What is the cause of  heart failure?     

A. Tricuspid regurgitation   

B. Mitral stenosis*   

C. Mitral regurgitation   

D. Tricuspid stenosis   

E. Aortic stenosis   

 

 171 A 75 year old man who has been suffering from diabetes for the last six months was found to be jaundiced. He was  asymptomatic except for weight loss at the rate of 10 pounds in 6 months. Physical examination revealed a hard, globular, right  upper quadrant mass that moves during respiration. A CT scan shows enlargement of the head of the pancreas, with no filling  defects in the liver. The most likely diagnosis is:    

A. Infectious hepatitis   

B. Carcinoma of the head of the pancreas*   

C. Malignant biliary stricture   

D. Metastatic disease of liver   

E. Haemolytic jaundice   

 

 172 Estimation of community health level involved analysis of a report on diseases registered among the population of district under  charge (reporting form 12). What index is calculated on the grounds of this report?    

A. Index of hospitalized morbidity   

B. Common sickness rate*   

C. Index of morbidity with temporary disability   

D. Index of pathological affection   

E. Index of basic non-epidemic morbidity   

 

 173 A 40 year old woman has a self-detected hard breast mass. The procedure of choice for confirming the diagnosis is:   

A. Mammography   

B. Thermography   

C. Aspiration biopsy with cytology   

D. Ultrasonography   

E. Excision biopsy*   

 

 174 A 36 year old female patient complains about general weakness, edemata of her face and hands, rapid fatigability during walking,  difficult diglutition, cardiac irregularities. These symptoms turned up 11 days after holiday at the seaside. Objectively: face  erythema, edema of shin muscles. Heart sounds are muffled, AP is 100/70 mm Hg. In blood: ASAT activity is 0,95  millimole/h*l, ALAT – 1,3 micromole/h*l, aldolase – 9,2 IU/l, creatine phosphokinase – 2,5 micromole Р/g*l. What  method of study would be the most specific?    

A. Muscle biopsy*   

B. ECG   

C. Echocardiogram   

D. Electromyography   

E. Determination of cortisol concentration in blood and urine   

 

 175 A patient complains about strong dyspnea that is getting worse during physical activity. Presentations appeared suddenly 2 hours  ago at work: acute chest pain on the left, cough. The pain was abating, but dyspnea, dizziness, pallor, cold sweat and cyanosis  were progressing. Vesicular respiration is absent, X-ray picture shows a shadow on the left. What pathology might be  suspected?    

A. Pulmonary infarction   

B. Pleuritis   

C. Pulmonary abscess   

D. Left-sided pneumonia   

E. Spontaneous left-sided pneumothorax*   

 

 176 A 28 year old parturient complains about headache, vision impairment, psychic inhibition. Objectively: AP- 200/110 mm Hg, evident  edemata of legs and anterior abdominal wall. Fetus head is in the area of small pelvis. Fetal heartbeats is clear, rhythmic, 190/min.  Internal investigation revealed complete cervical dilatation, fetus head was in the area of small pelvis. What tactics of labor  management should be chosen?     

A. Embryotomy   

B. Forceps operation*   

C. Cesarean    

D. Conservative labor management with episiotomy    

E. Stimulation of labor activity    

 

 177 An outbreak of food poisoning was recorded in an urban settlement. The illness was diagnosed as botulism on the grounds of  clinical presentations. What foodstuffs should be chosen for analysis in the first place in order to confirm the diagnosis?    

A. Boiled meat   

B. Potatoes   

C. Pasteurized milk   

D. Cabbage   

E. Tinned food*   

 

 178 A 3 year old boy fell ill abruptly: fever up to 39oC, weakness, vomitng. Haemorrhagic rash of various size appeared on his  lower limbs within 5 hours. Meningococcemia with infective – toxic shock of the 1 degree was diagnosed. What medications  should be administered?   

A. Ampicillin and immunoglobulin   

B. Chloramphenicol succinate and prednisone*   

C. Penicillin and prednisone   

D. Penicillin and immunoglobulin   

E. Chloramphenicol succinate and interferon   

 

 179 A 5 month old boy was born prematurely, he didn’t suffer from any disease at the infant age and later on. Examination at an  outpatient’s hospital revealed paleness of skin, sleepiness. Blood count: Hb – 95 g/l, erythrocytes – 3,5*1012/l,  reticulocytes – 90/00, colour index – 0,7, osmotic stability of erythrocytes – 0,44-0,33%, serum iron – 4,9 micromole/l. What is  the most probable cause of anemia?    

A. Hemogenesis immaturity   

B. Erythrocyte hemolysis   

C. Iron deficit*   

D. Infectious process   

E. B12 deficit   

 

 180 A patient was delivered to a surgical department after a road accident with a closed trauma of chest and right-sided rib fracture.  The patient was diagnosed with right-sided pneumothorax, it is indicated to perform drainage of pleural cavity. Pleural puncture  should be made in:    

A. In the 2nd intercostal space along the middle clavicular line*   

B. In the point of the greatest dullness on percussion   

C. In the 6th intercostal space along the posterior axillary line   

D. In the projection of pleural sinus   

E. In the 7th intercostal space along the scapular line   

 

 181 A 25 year old patient complains about weakness, dizziness, haemorrhagic skin rash. She has been suffering from this for a  month. Blood count: erythrocytes: 1,0*1012/l, Hb- 37 g/l, colour index – 0,9, leukocytes – 1,2*109/l, thrombocytes –  42*109/l. What diagnostic method will be the most effective?    

A. Spleen biopsy   

B. Liver biopsy   

C. Coagulogram   

D. Abdominal ultrasound   

E. Sternal puncture*   

 

 182 An 8 year old boy suffering from haemophilia was undergoing transfusion of packed red cells. Suddenly he got pain behind the  breastbone and in the lumbar area, dyspnea, cold sweat. Objectively: pale skin, heart rate – 100/min, AP- 60/40 Hg; oliguria, brown  urine. For treatment of this complication the following drug should be administered:    

A. Prednisolone*   

B. Analgine   

C. Lasix   

D. Adrenaline   

E. Aminophylline   

 

 183 A 72 year old female patient has been treated for urolithiasis in the urological department. After atropine injection she got acute  pain in her left eye and abrupt vision impairment. Objectively: visual acuity of the left eye is 0,01, the eye is dense but painful on  palpation, cornea is opaque, there is cyanotic induration of eyeball vessels. What is the most probable diagnosis?     

A. Acute keratitis of the left eye   

B. Acute attack of primary glaucoma of the left eye*    

C. Secondary glaucoma of the left eye   

D. Degeneration of the left eye cornea    

E. Acute iridocyclitis of the left eye   

 

 184 What guarantees against the preconceived attitude to the physician in cases of professional law violations do you know?   

A. Utilisation copy of medical documents    

B. Conduct an inquiry by preliminary investigator of police department    

C. Sanction of public prosecutor, inquiry by preliminary investigator of prosecutor’s office, committee of experts*     

D. Draw up a statement about forensic medical examination    

E. Conduct forensic medical examination by district forensic medicine expert    

 

 185 Immediately after delivery a woman had haemorrhage, blood loss exceeded postpartum haemorrhage rate and was progressing.  There were no symptoms of placenta detachment. What tactics should be chosen?    

A. Uterus tamponade   

B. Instrumental revision of uterine cavity walls   

C. Manual removal of placenta and afterbirth*   

D. Removal of afterbirth by Crede’s method   

E. Intravenous injection of methylergometrine with glucose   

 

 186 As a result of prophylactic medical examination a 35 year old woman was diagnosed with alimentary and constitutive obesity of  the III degree. It is known from her anamnesis that the patient doesn’t observe rules of rational nutrition: she often overeats, the  last food intake is usually 10-15 minutes before going to bed, prefers fattening and rich in carbohydrates food. What is the main  alimentary risk factor of obesity development?     

A. Excess of carbohydrates   

B. Energetic unprofitableness of nutrition*   

C. Lack of cellulose   

D. Violation of dietary pattern   

E. Excess of fats   

 

 187 25 children at the age of 2-3 years who don’t attend any child welfare institutions should be observed by a district pediatrician  within the current year. How many initial visits of this group of children should be planned?   

A. 20   

B. 100   

C. 50*   

D. 40   

E. 200   

 

 188 A 25 year old patient complains of pain in the I finger on his right hand. On examination: the finger is homogeneously hydropic, in  bent position. On attempt to unbend the finger the pain is getting worse. Acute pain appears during the probe in ligament  projection. What decease is the most likely?     

A. Thecal whitlow (ligament panaritium)*    

B. Articular (joint) panaritium    

C. Paronychia    

D. Subcutaneous panaritium    

E. Bone panaritium    

 

 189 A sergeant was injured by a shell splinter in the left subcostal area. He was bandaged with a first-aid pack on a battlefield. The  patient was delivered to the regiment medical aid station. He complains about dizziness, weakness, thirst, abdominal pain. General  condition is grave, the patient is pale. Ps is 120 bpm. Abdomen is soft, painful on palpation. The bandage is well fixed but a little bit  soaked with blood. The patient should be evacuated to the medical battalion with the following transport and in the following turn:     

A. With a passing car in the third turn   

B. With medical vehicle in the second turn   

C. With a passing car in the second turn   

D. With a passing car in the first turn   

E. With medical vehicle in the first turn*   

 

 190 A nine year old child is at a hospital with acute glomerulonephritis. Clinical and laboratory examinations show acute condition.  What nutrients must not be limited during the acute period of glomerulonephritis?   

A. Salt   

B. Carbohydrates*   

C. Proteins   

D. Liquid   

E. Fats   

 

 191 The amount of ultraviolet radiation dose was measured in minutes. What device was applied for measurement of the biodose?    

A. UV-meter   

B. Gorbachev’s biodosimeter*   

C. Actinometer   

D. Catathermometer   

E. Radiometer   

 

 192 A patient with acute purulent otitis media complicated by mastoiditis was admitted to a hospital. Roentgenogram of mastoid  processes showed the shadiowing of the cellular system on the lesion, absence of bone septa was present. What are the  necessary therapeutic actions at the second stage of mastoiditis?    

A. Tympanoplasty   

B. Mastoidotomy*   

C. Radical operation on the middle ear   

D. Cateterization of the Eustachian tube   

E. Paracentesis of the drum    

 

 193 A 30 year old patient complains about inability to become pregnant over 3 years of married life. The patient is of supernutrition  type, she has hair along the median abdominal line, on the internal thigh surface and in the peripapillary area. Menses started at the  age of 16, they are infrequent and non-profuse. US revealed that the uterus was of normal size, ovaries were 4х5х5 cm large  and had a lot of cystic inclusions. What is the most probable diagnosis?    

A. Menstrual irregularity   

B. Chronic oophoritis   

C. Bilateral ovarian tumours   

D. Polycystic ovaries*   

E. Ovarian cystoma   

 

 194 A 63 year old patient complained about pain in the lumbar area. He underwent a course of physiological treatment on account of  radiculitis but this led to no improvement of his condition. R-graphy of spinal column and pelvic bones revealed osteoporosis and  serious bone defects. Blood analysis revealed moderate normochromic anaemia, urine analysis revealed proteinuria. Whole blood  protein made up 10,7 g/l. What disease should be suspected?    

A. Acute radiculitis   

B. Myelomatosis*   

C. Urolithiasis   

D. Metastases in bones   

E. Systemic osteoporosis   

 

 195 During preventive examination a 16 year old patient presented no problems. Objectively: the patient has signs of malnutrition, he is  asthenic, AP is 110/70 mm Hg, Ps is 80 bpm, cardiac border is normal, auscultation above the cardiac apex reveals three sounds,  cardiac murmur is absent. ECG shows no pathological changes, phonocardiogram shows that the third sound comes 0,15 s after  the second one above the apex. How are these changes called?   

A. Atrial gallop rhythm   

B. Protodiastolic gallop rhythm   

C. Fout-ta-ta-rou (reduplication of the 2nd sound)   

D. IV physiological sound   

E. III physiological sound*  

 

 196 A patient has been in a hospital. The beginning of the disease was gradual: nausea, vomiting, dark urine, аcholic stools,  yellowness of the skin and scleras. The liver is protruded by 3 cm. Jaundice progressed on the 14th day of the disease. The liver  diminished in size. What complication of viral hepatitis caused deterioration of the patient’s condition?    

A. Infectious-toxic shock   

B. Meningitis   

C. Cholangitis   

D. Hepatic encephlopathy*   

E. Relapse of viral hepatitis     

 

 197 A 70 year old man is suffering from coronary heart disease. His mood is evidently depressed, anxious. As a result of continious  sleeplessness he has got fears, suicidal thoughts. He would sit for a long time in the same pose, answer after a pause, in a low,  monotonous voice. His face has a look of suffering, pain, fear. What is the main psychopathologic syndrome?      

A. Asthenic syndrome   

B. Phobic syndrome   

C. Depressive syndrome*   

D. Obsessive syndrome   

E. Paranoid syndrome   

 

 198 A 16 year old female patient underwent an operation on account of diffuse toxic goiter of the III-IV degree 12 years ago. Now she  has recurrence of thyrotoxicosis. The patient was offered operative intervention, but it is necessary first to localize the  functioning gland tissue. What method should be applied for this purpose?     

A. Roentgenography of esophagus    

B. Gland scanning*    

C. USI   

D. Puncture aspiration biopsy    

E. Roentgenography of neck    

 

 199 A woman delivered a child. It was her fifth pregnancy but the first delivery. Mother’s blood group is A(II)Rh, newborn’s –  A(II)Rh+. The level of indirect bilirubin in umbilical blood was 58 micromole/l, haemoglobin – 140 g/l, RBC- 3,8*1012/l. In  2 hours the level of indirect bilirubin turned 82 micromole/l. The hemolytic disease of newborn (icteric-anemic type,  Rh-incompatibility) was diagnosed. Choose the therapeutic tactics:    

A. Antibiotics   

B. Replacement blood transfusion (conservative therapy)*   

C. Blood transfusion (conservative therapy)   

D. Symptomatic therapy   

E. Conservative therapy   

 

 200 In order to study impact of microclimate upon the human organism it is necessary to make systematic observation of air  temperature over 3 days. Choose a device that will allow to make the most precise temperature records:    

A. Alcohol thermometer   

B. Mercury thermometer   

C. Thermograph*   

D. August’s psychrometer   

E. Assmann psychrometer   

 

 

ch 16. Gynocological Cancers

Posted: May 19, 2012 in Gynocology

Gynecologic cancer is a group of cancers that affect the tissue and organs of the female reproductive system. Each type of cancer is named after the organ it originates.

Types of gynecologic cancer include:

  1. Cervical
  2. Endometrial
  3. Fallopian Tube
  4. Gestational Trophoblastic Disease
  5. Ovarian
  6. Vulvar
  7. Vaginal

Cervical Cancer :

Cervical cancer is cancer that starts in the cervix, the lower part of the uterus (womb) that opens at the top of the vagina. It’s 12% of  all gynecologic cancers.

  •  Mean age is 47.
  •  20% of cases in women >65yrs
  •  10% of cases in women >75yrs

Types :

  1.       Squamous Cell      85-90%
  2.       Adenocarcinoma   10-15%
  3.       Numerous others   <5%

Etiology :

The primary risk factor for developing cervical cancer is the human papillomavirus (HPV). HPV is a common sexually transmitted infection that is spread through sexual, skin-to-skin contact. The virus can cause changes in cervical cells that could develop into cervical cancer if left undetected, unmonitored, or untreated.

Cervical human papillomavirus (HPV) : HPVs establish productive infections only in keratinocytes of the skin or mucous membranes. While the majority of the known types of HPV cause no symptoms in most people, some types can cause warts (verrucae), while others can – in a minority of cases – lead to cancers of the cervix, vulva, vagina, penis, oropharynx and anus.

  • 80% of sexually active women will acquire HPV.
  • 70% of infections are gone in 1 year and ninety percent in 2 years.
  • However, when the infection persists — in 5% to 10% of infected women — there is high risk of developing precancerous lesions of the cervix, which can progress to invasive cervical cancer.

    HPV With HIV(+) Female Patient.

Human Papillovirus

Cervical Cancer Stages

Endometrial Cancer :

Syn : Endometrial adenocarcinoma; Uterine adenocarcinoma; Uterine cancer; Adenocarcinoma – endometrium; Adenocarcinoma – uterus; Cancer – uterine; Cancer – endometrial; Uterine corpus cancer .

Most common Gynocological Cancer.Endometrial cancer is cancer that starts in the endometrium, the lining of the uterus (womb).Mean age is 61 yrs.

Types :

Endometrial Carcinoma : Most endometrial cancers are carcinomas (usually adenocarcinomas), meaning that they originate from the single layer of epithelial cells that line the endometrium and form the endometrial glands.There are many microscopic subtypes of endometrial carcinoma.

  1. Endometrioid type (most common)
  2. Papillary serous carcinoma (more aggressive)
  3. Clear cell endometrial carcinomas (aggressive and high recurence rate

Endometrial Sarcoma : In contrast to endometrial carcinomas, the uncommon endometrial stromal sarcomas are cancers that originate in the non-glandular connective tissue of the endometrium. Uterine carcinosarcoma, formerly called Malignant mixed müllerian tumor, is a rare uterine cancer that contains cancerous cells of both glandular and sarcomatous appearance – in this case, the cell of origin is unknown.

Fallopian tube cancer :

Primary fallopian tube cancer (PFTC), often just tubal cancer, is a malignant neoplasm that originates from the fallopian tube.ubal cancer is thought to be a relatively rare primary cancer among women accounting for 1 – 2% of all Gynocological Cancers.

Diagnosis : A pelvic examination may detect an adnexal mass. A CA-125 blood test is a nonspecific test that tends to be elevated in patients with tubal cancer. More specific tests are a gynecologic ultrasound examination, a CT scan, or an MRI of the pelvis. Occasionally, an early fallopian tube cancer may be detected serendipitously during pelvic surgery.

Gestational trophoblastic disease (GTD) :

Gestational trophoblastic disease (GTD) is a term used for a group of pregnancy-related tumours.The cells that form gestational trophoblastic tumours are called trophoblasts and come from tissue that grows to form the placenta during pregnancy.

Features :

  1. This tissue may grow at the same rate as a normal pregnancy.
  2. Can produces chorionic gonadotropin, a hormone which is measured to monitor fetal well-being.
  3. Common in child-bearing age, it may rarely occur in postmenopausal women.

The main types of gestational trophoblastic diseases are:

  • Hydatidiform mole ( Benign)
  • Invasive mole (Malignant)
  • Choriocarcinoma (Malignant)
  • Placental-site trophoblastic tumor (Malignant)

Hadatidiform Mole : Hydatidiform mole is an overgrowth of placental tissue or an abnormal growth that develops from a non-viable, fertilized egg at the beginning of a pregnancy. It often is referred to as a molar pregnancy. Instead of the normal embryonic cell division that results in the development of a fetus, the placental material grows uncontrolled and develops into a shapeless mass of watery, small, blister-like sacs (vesicles). The cause of hydatidiform mole is unknown, but is thought to be caused in part by chromosomal abnormalities .

Invasive mole : An invasive mole (formerly known as chorioadenoma destruens) is a hydatidiform mole that has grown into the muscle layer of the uterus. Invasive moles can develop from either complete or partial moles, but complete moles become invasive much more often than do partial moles.

Choriocarcinoma : Choriocarcinoma is a malignant form of GTD. It is much more likely than other types of GTD to grow quickly and spread to organs away from the uterus.Choriocarcinoma most often develops from a complete hydatidiform mole.

Choriocarcinoma

Placental-site trophoblastic tumor :

Placental-site trophoblastic tumor (PSTT) is a very rare form of GTD that develops where the placenta attaches to the lining of the uterus. This tumor most often develops after a normal pregnancy or abortion, but it may also develop after a complete or partial mole is removed.

Ovarian Cancer :

Ovarian cancer

Ovarian cancer is cancer that starts in the ovaries. The ovaries are the female reproductive organs that produce eggs.Ovarian cancer affects females of any ages.Cancer should be suspected in any woman between 40 and 80 with persistent gastrointestinal symptoms cannot be diagnosed.Diagnosis is difficult.  70% of diagnoses are stage III or IV.

Types :

  1.       Epithelial—About 85-90% (>50 yrs)
  2.       Germ cell—10-15%  (<20yrs)
  3.       Gonadal stroma—5-10%
  4.       Mesenchymal

Symptoms :

Ovarian cancer symptoms are often vague.

Woman should see herr doctor if she has the following symptoms on a daily basis for more than a few weeks:

  1. Bloating or swollen belly area
  2. Difficulty eating or feeling full quickly
  3. Pelvic or lower abdominal pain; the area may feel “heavy” to you (pelvic heaviness)

Other symptoms are also seen with ovarian cancer. However, these symptoms are also common in women who do not have cancer:

  1. Abnormal menstrual cycles
  2. Digestive symptoms:
  3. Constipation
  4. Increased gas
  5. Indigestion
  6. Lack of appetite
  7. Nausea and vomiting
  8. Unexplained back pain that worsens over time
  9. Vaginal bleeding that occurs in between periods
  10. Weight gain or loss
  11. Other symptoms that can occur with this disease:
  12. Excessive hair growth that is coarse and dark
  13. Sudden urge to urinate
  14. Needing to urinate more often than usual

Vaginal cancer :

Vaginal cancer is any type of cancer that forms in the tissues of the vagina. Primary vaginal cancer is rare in the general population of women and is usually a squamous carcinoma. Metastases are more common.

Feature :

  1. Common at +50 Years
  2. Can affects at any age ,even in Infancy.
  3. 5 – 10% of patients have no symptoms.

Types :

  1. Squamous cell carcinoma (Vaginal squamous cell carcinoma arises from the thin, flat squamous cells that line the vagina. This is the most common type of vaginal cancer. It is found most often in women aged 60 or older.)

Adenocarcinoma (aginal adenocarcinoma arises from the glandular (secretory) cells in the lining of the vagina that produce some vaginal fluids. Adenocarcinoma is more likely than squamous cell cancer to spread to the lungs and lymph nodes. It is found most often in women aged 30 or younger.)

Vaginal Cancer

Vulvar cancer :

Vulvar cancer is a rare type of cancer. It forms in a woman’s external genitals, called the vulva. The cancer usually develops slowly over several years. First, precancerous cells grow on vulvar skin. This is called vulvar intraepithelial neoplasia (VIN), or dysplasia. Not all VIN cases turn into cancer, but it is best to treat it early.

Often, vulvar cancer doesn’t cause early symptoms. However, see your doctor for testing if you notice

  • A lump in the vulva
  • Vulvar itching or tenderness
  • Bleeding that is not your period

Being older and having a human papillomavirus infection are risk factors for vulvar cancer. Treatment varies, depending on your overall health and how advanced the cancer is. It might include laser therapy, surgery, radiation or chemotherapy.

Types :

  1. Squamous cell carcinoma.
  2. Melanoma
  3. Basal cell carcinoma
  4.  Adenocarcinoma
  5.  Sarcoma.

Patient with vulvar cancer after 4 weeks ( A) and 6 weeks ( B) of chemoradiation.

ch 15. Uterine fibroids

Posted: May 17, 2012 in Gynocology

Uterine fibroids ;Leiomyoma ; Fibromyoma ; Myoma ; Fibroids :  A leiomyoma  is a benign smooth muscle neoplasm that is very rarely (0.1%) premalignant. They can occur in any organ, but the most common forms occur in the uterus, small bowel and the esophagus.

Etiology :

  1. Etiology is unknown.
  2. Originates from Single Muscle Cell (Could be Embryonic Cells or Sooth Muscle Cell from blood vessels)
  3. Estogens : no evidence that it is a causative factor , it has been implicated in growth of myomas
  4. Myoma contains estrogen receptors in higher concentration than surrounding myometrium,so Estrogen helps Myoma to be enlarged.
  5. Progestrone helps Myoma for further cell division (Mitosis).
Features :
  1. Occurring frequently at 50 yr. and very rare before 20 yr.
  2. 25% in white women
  3. 50% in black women

Types: They are classified by anatomic locations. Most frequently,

  1. Subserous (beneath the peritoneum)
  2. Intramural (within the uterine wall)
  3. Submucous (beneath the endometrium)

a = Subserosal fibroids
b = Intramural fibroids
c = Submucosal fibroid
d = Pedunculated submucosal fibroid
e = Fibroid in statu nascendi

Microscopic Stracture of Myoma :

  1.  Nonstriated muscle fibers are localized  in different directions
  2. Pseudocapsule of areolar tissue (a type of loose connective tissue esp. made of collagens) & compressed myometrium
  3. Arteries are less dense than myometrium & do not have a regular pattern of distribution
  4. 1-2 major vesseles are found at the base or pedicle

Symptoms :

Depends on size and location

■Asymptomatic – small (and some large ones) – detected on routine examination

■Menorrhagia – if submucous. May lead to anaemia, with symptoms associated with this.

■Crampy pains may result from contraction of the uterus.

■Pressure symptoms from large fibroids

  • Dysuria
  • Constipation or backache
  • Pelvic pain and sometimes, impossible intercourse from cervical myoma
  • Subfertility and Recurrent miscarriage from submucous fibroids

Differential Diagnosis : Exclude other causes of abnormal bleeding

  1. Endometrial hyperplasia
  2. Endometrial or tubal Cancer
  3. Uterine sarcoma
  4. Ovarian Cancer
  5. Polyps
  6. Adenomyosis
  7. DUB (Dysfunctional Uterine Bleeding)
  8. Endometriosis
Exogenouse estrogens Endometrial biopsy or D&C is essential in the evaluation of abnormal bleeding to exclude endometrial Cancer
Complications :
For Pregnant Women :
  1. Uterine inertia
  2. Malpresentation
  3. Obstruction of the birth canal
  4. Cervical or isthmeic myoma
  5. Postpartum Hemorrhage

For Non-Pregnant Women :

  1. Heavy bleeding with anemia is the most common
  2. Urinary or bowel obstruction from large parasitic myoma is much less common
  3. Malignant transformation is rare
  4. Ureteral injury or ligation is a recognized complication of surgery for Cervical  Myoma
Treatments :

Treatment for the symptoms of fibroids may include:

  1. Birth control pills (oral contraceptives) to help control heavy periods
  2. Intrauterine devices (IUDs) that release the hormone progestin to help reduce heavy bleeding and pain
  3. Iron supplements to prevent or treat anemia due to heavy periods
  4. Nonsteroidal anti-inflammatory drugs (NSAIDs) such as ibuprofen or naprosyn for cramps or pain
  5. Short-term hormonal therapy injections to help shrink the fibroids

Surgery and procedures used to treat fibroids include:

  1. Hysteroscopic resection of fibroids: Women who have fibroids growing inside the uterine cavity may need this outpatient procedure to remove the fibroid tumors.
  2. Uterine artery embolization: This procedure stops the blood supply to the fibroid, causing it to die and shrink. Women who may want to become pregnant in the future should discuss this procedure with their health care provider.
  3. Myomectomy: This surgery removes the fibroids. It is often the chosen treatment for women who want to have children, because it usually can preserve fertility. More fibroids can develop after a myomectomy.
  4. Hysterectomy: This invasive surgery may be an option if medicines do not work and other surgeries and procedures are not an option.

ch 14. Endometriosis

Posted: May 17, 2012 in Gynocology
  • Every month, a woman’s ovaries produce hormones that tell the cells lining the uterus (womb) to swell and get thicker. The body removes these extra cells from the womb lining (endometrium) when she gets her period.
  • If these cells (called endometrial cells) implant and grow outside the uterus, endometriosis results.Women with endometriosis typically have tissue implants on the ovaries, bowel, rectum, bladder, lining of the pelvic area and other Ectopic Locations.

Etiology :

  1. The cause of endometriosis is unknown.
  2. Retrograde menstruation :One theory is that the endometrial cells shed when you get your period travel backwards through the fallopian tubes into the pelvis, where they implant and grow. This is called .
  3. Aging : Endometriosis is typically diagnosed between ages 25 – 35, the condition probably begins about the time that regular menstruation begins.
  4. Genetics : Changes in Chromosome 10 can create Endometriosis.Any past family history of Endometriosis.
  5. Biomarkers : The one biomarker that has been used in clinical practice over the last 20 years is CA-125. However, its performance in diagnosing endometriosis is low, even though it shows some promise in detecting more severe disease.

    Retrigrade Menstuation

    Can for Endometrial Cyst or Chocolate cyst as a further complication.

Stages of Endometriosis :

Stage 1 (Minimal): Just a few endometrial implant; mostly found in the cul-de-sac and pelvic area.
Stage 2 (Mild): Moderate levels of endometriosis to mild levels that not only affect the above areas but can now affect the ovaries.
Stage 3 (Moderate): Moderate amount of disease and in extensive places around the pelvic cavity, with adhesions.
Stage 4 (Severe) : Extensive endomtrial implants sprinkled all throughout the pelvic cavity with adhesions; higher probability of infertility.

Symptoms :

  1. Dysmenorrhea – painful, sometimes disabling cramps during menses; pain may get worse over time (progressive pain), also lower back pains linked to the pelvis
  2. Chronic pelvic pain – typically accompanied by lower back pain or abdominal pain
  3. Dyspareunia – painful sex
  4. Dysuria – urinary urgency, frequency, and sometimes painful voiding

Diagnosis :

  1. Laparoscopy:  A surgical procedure where a camera is used to look inside the abdominal cavity
  2. Ultrasound and magnetic resonance imaging (MRI) : Use of imaging tests may identify endometriotic cysts or larger endometriotic areas. It also may identify free fluid often within the Recto-uterine pouch.
  3.  Biopsy : The diagnosis is based if Endometrium like tissues can be found on ectopic Locations.

ch. 13 Infertility

Posted: May 17, 2012 in Gynocology

Infertility means She/He cannot make a baby (conceive).

Types :

  • Primary infertility refers to couples who have not become pregnant after at least 1 year of unprotected sex (intercourse).
  • Secondary infertility refers to couples who have been pregnant at least once, but never again.
Requirements for Normal Reproduction :
1)Release of a normal preovulatory oocyte.
2)Production of adequate spermatozoa.
3)Normal transport of the gametes to the ampullary portion of the fallopian tube (where fertilization occurs).
4)Subsequent transport of the cleaving embryo into the endometrial cavity for implantation and development.

Etiology :

Female infertility :

  1. A fertilized egg or embryo does not survive once it sticks to the lining of the womb (uterus)
  2. The fertilized egg does not attach to the lining of the uterus
  3. The eggs cannot move from the ovaries to the womb
  4. The ovaries have problems producing eggs

Female infertility may be caused by:

  1. Autoimmune disorders, such as antiphospholipid syndrome (APS)
  2. Cancer or tumor
  3. Clotting disorders
  4. Diabetes
  5. Growths (such as fibroids or polyps) in the uterus and cervix
  6. Birth defects that affect the reproductive tract
  7. Excessive exercising
  8. Eating disorders or poor nutrition
  9. Use of certain medications, including chemotherapy drugs
  10. Drinking too much alcohol
  11. Obesity
  12. Older age
  13. Ovarian cysts and polycystic ovary syndrome (PCOS)
  14. Pelvic infection or pelvic inflammatory disease (PID)
  15. Scarring from sexually transmitted infection or endometriosis
  16. Thyroid disease
  17. Too little or too much hormones
Male infertility :
  1. A decrease in sperm count
  2. Sperm being blocked from being released
  3. Sperm that do not work properly

Male infertility can be caused by:

  1. Environmental pollutants
  2. Being in high heat for prolonged periods
  3. Birth defects
  4. Heavy use of alcohol, marijuana, or cocaine
  5. Too little or too much hormones
  6. Impotence
  7. Infection
  8. Older age
  9. Cancer treatments, including chemotherapy and radiation
  10. Scarring from sexually transmitted diseases, injury, or surgery
  11. Retrograde ejaculation
  12. Smoking
  13. Use of certain drugs, such as cimetidine, spironolactone, and nitrofurantoin

 Tests for diagnosis Female Infertility :

  1. Transvaginal Ultrasonography (TVUS)
  2. Hysterosalpingography (HSG)
  3. Hysteroscopy
  4. Laparoscopy
  5. History of Past Diseases

Tests for diagnosis Male Infertility :

  1. Scrotal Ultrasonography
  2. Transrectal Ultrasonography (TRUS)
  3. Complete Blood Count (CBC)
  4. FSH, Testosterone
  5. GC/Chlam, UA
  6. Renal and Liver Function
  7. Semen analysis
  8. Postejaculatory urinalysis
  9. Past sexually transmitted infection (STI).
  10. Varicocele  (widening of the Varicose veins along the cord that holds up a man’s testicles)
  11. Hernia
  12. History of Past Diseases

Infertility Treatment :

  • Treatment of infertility depends on the cause, diagnosis, duration of infertility, age of the partners and many personal preferences.
  • Some causes of infertility cannot be corrected.
  • A woman can still become pregnant with assisted reproductive technology or other procedures to restore fertility.
Increase frequency of intercourse: Two to three times a week of intercourse may improve fertility.

Sperm survive in the female reproductive tract for up to 72 hours, and an egg can be fertilized for up to 24 hours after ovulation.

General sexual problems:  Addressing impotence or premature ejaculation can improve fertility.
Treatment for these problems often is with medication or behavioral approaches.

 Lack of sperm: surgery or hormones to correct the problem or use of assisted reproductive technology is sometimes possible.

Varicocele can often be surgically corrected.

Stimulating ovulation with fertility drugs: Fertility drugs are the main treatment for women who are infertile due to ovulation disorders.These medications regulate or induce ovulation, and work like natural hormones to trigger ovulation such as: Follicle-stimulating hormone (FSH) and Luteinizing hormone (LH).

Commonly used fertility drugs include:

  1.  Clomiphene citrate
  2. Human menopausal gonadotropin, or hMG (Repronex)
  3. Follicle-stimulating hormone, or FSH (Gonal-F, Follistim, Bravelle)
  4. Human chorionic gonadotropin, or HCG (Ovidrel, Pregnyl)
  5. Gonadotropin-releasing hormone (Gn-RH) analogs
  6. Letrozole (Femara)
 
In Vitro Fertilization
Normal Menses : 
  • Flow lasts 2-7 days
  • Cycle 21-35 days in length
  • Total menstrual blood loss 20-60 mL
Other Causes of Vaginal Bleeding :
  1. Pregnancy related causes
  2. Medications
  3. Anatomic causes
  4. Infectious disease
  5. Endocrine abnormalities: Thyroid, DM
  6. Bleeding disorders
  7. Endometrial hyperplasia
  8. Neoplasms

Descriptive Term

Bleeding pattern

Menorrhagia Regular cycles, prolonged duration, excessive flow
Metrorrhagia Irregular cycles
Menometorrhagia Irregular, prolonged, excessive
Hypermenorrhea Regular, normal duration, excessive flow
Polymenorrhea Frequent cycles
Oligomenorrhea Infrequent cycles